SlideShare a Scribd company logo
ENGINEERING MATHEMATICS -IV
SUBJECT CODE: BSH251
(Regulation 2016-17)
Common to all branches of B.E
UNIT-I
LAPLACE TRANSFORM
Engineering Maths - IV
Laplace Transform Page 1
Engineering Maths - IV
UNIT-V
LAPLACE TRANSFORM
1. INTRODUCTION
A transformation is an operation which converts a mathematical expression to a different but
equivalent form. The well known transformation logarithms reduce multiplication and division to a simpler
process of addition subtraction.
The Laplace transform is a powerful mathematical technique which solves linear equations
with given initial conditions by using algebra methods. The Laplace transform can also be used to solve
systems of differential equations, Partial differential equations and integral equations. In this chapter, we
will discuss about the definition, properties of Laplace transform and derive the transforms of some
functions which usually occur in the solution of linear differential equations.
2. LAPLACE TRANSFORM
Let 𝑓(𝑡) be a function of t defined for all 𝑡 ≥ 0 .then the Laplace transform of𝑓(𝑡), denoted by
𝐿[ 𝑓(𝑡)] is defined by
𝐿[ 𝑓(𝑡)] = ∫0
∞
𝑒−𝑠𝑡 𝑓(𝑡)𝑑𝑡
𝑡→∞
Provided that the integral exists, “s” is a parameter which may be real or complex. Clearly 𝐿[ 𝑓(𝑡)]is a
function of s and is briefly written as 𝐹(𝑠) (𝑖. 𝑒. ) 𝐿[ 𝑓(𝑡)] = 𝐹(𝑠)
Piecewise continuous function
A function 𝑓(𝑡) is said to be piecewise continuous is an interval 𝑎 ≤ 𝑡 ≤ 𝑏, if the interval can be sub
divided into a finite number of intervals in each of which the function is continuous and has finite right and
left hand limits.
Exponential order
A function 𝑓(𝑡) is said to be exponential order if lim 𝑒−𝑠𝑡𝑓(𝑡) is a finite quantity, where 𝑠 >
0(exists).
Example: 5. 1 Show that the function 𝒇(𝒕) = 𝒆𝒕𝟑
is not of exponential order.
Solution:
lim 𝑒−𝑠𝑡 𝑒𝑡3
=lim 𝑒−𝑠𝑡+𝑡3
= lim 𝑒𝑡3−𝑠𝑡
𝑡→∞ 𝑡→∞ 𝑡→∞
= 𝑒∞ = ∞, not a finite quantity.
Hence 𝑓(𝑡) = 𝑒𝑡3
is not of exponential order.
Sufficient conditions for the existence of the Laplace transform
The Laplace transform of 𝑓(𝑡) exists if
i)
ii)
𝑓(𝑡) is piecewise continuous in the interval 𝑎 ≤ 𝑡 ≤ 𝑏
𝑓(𝑡) is of exponential order.
Laplace Transform Page 2
Engineering Maths - IV
Note: The above conditions are only sufficient conditions and not a necessary condition.
Example: 5.2 Prove that Laplace transform of 𝒆𝒕𝟐
does not exist.
Solution:
lim 𝑒−𝑠𝑡 𝑒𝑡2
=lim 𝑒−𝑠𝑡+𝑡2
= lim 𝑒𝑡2−𝑠𝑡
𝑡→∞ 𝑡→∞ 𝑡→∞
= 𝑒∞ = ∞ ,not a finite quantity.
∴ 𝑒𝑡2
is not of exponential order.
Hence Laplace transform of 𝑒𝑡2
does not exist.
5.3 PROPERTIES OF LAPLACE TRANSFORM
Property: 1 Linear property
𝑳[𝒂𝒇(𝒕) ± 𝒃𝒈(𝒕)] = 𝒂𝑳[𝒇(𝒕)] ± 𝒃𝑳[𝒈(𝒕)] , where a and b are constants.
Proof:
𝐿[𝑎𝑓(𝑡) ± 𝑏𝑔(𝑡)] = ∫
∞
[𝑎𝑓(𝑡) ± 𝑏𝑔(𝑡)] 𝑒−𝑠𝑡 𝑑𝑡
0
=𝑎 ∫0
∞
𝑔(𝑡) 𝑒−𝑠𝑡𝑑𝑡
∞
𝑓(𝑡)𝑒−𝑠𝑡𝑑𝑡 ± 𝑏 ∫
0
𝐿[𝑎𝑓(𝑡) ± 𝑏𝑔(𝑡)] = 𝑎 𝐿[𝑓(𝑡)] ± 𝑏 𝐿[𝑔(𝑡)]
Property: 2 Change of scale property.
𝒂 𝒂
If 𝑳[𝒇(𝒕)] = 𝑭(𝒔), then 𝑳[𝒇(𝒂𝒕)] = 𝟏
𝑭 (𝒔
) ;𝒂 > 0
Proof:
Given 𝐿[𝑓(𝑡)] = 𝐹(𝑠)
∞
𝑒−𝑠𝑡 𝑓(𝑡) 𝑑𝑡 = 𝐹(𝑠) ⋯ ⋯ (1)
∴ ∫0
By the definition of Laplace transform, we have
𝐿[𝑓(𝑎𝑡)] = ∫0
∞
𝑒−𝑠𝑡 𝑓(𝑎𝑡) 𝑑𝑡 ⋯ ⋯ (2)
Put at= 𝑥 𝑖𝑒. , 𝑡 = 𝒙
⇒ 𝑑𝑡 = 𝑑𝑥
𝒂 𝑎
−𝒔𝒙
∞ 𝑑𝑥
(2) ⇒ 𝐿[𝑓(𝑎𝑡)] = ∫0 𝑒 𝒂 𝑓(𝑥) 𝑎
−𝒔𝒙
𝟏 ∞
= ∫ 𝑒 𝒂 𝑓(𝑥)𝑑𝑥
𝒂 0
Replace 𝑥 by t,
−𝑠
𝑡
1 ∞
𝐿[𝑓(𝑎𝑡)] = ∫ 𝑒 𝑎 𝑓(𝑡)𝑑𝑡
𝑎 0
𝟏 𝒔
𝒂 𝒂
𝑳[𝒇(𝒂𝒕)] = 𝑭 ( ) ;𝒂 > 0
Property: 3 First shifting property.
𝑳[𝒆−𝒂𝒕𝒇(𝒕)] = 𝑭(𝒔 + 𝒂)
𝑳[𝒆𝒂𝒕𝒇(𝒕)] = 𝑭(𝒔 − 𝒂)
If 𝑳[𝒇(𝒕)] = 𝑭(𝒔), then i)
ii)
Proof:
(i) 𝐿[𝑒−𝑎𝑡𝑓(𝑡)] = 𝐹(𝑠 + 𝑎)
Laplace Transform Page 3
Given 𝐿[𝑓(𝑡)] = 𝐹(𝑠)
∞
𝑒−𝑠𝑡 𝑓(𝑡) 𝑑𝑡 = 𝐹(𝑠) ⋯ (1)
∴ ∫0
By the definition of Laplace transform, we have
∞
𝑒−𝑠𝑡 𝑒−𝑎𝑡𝑓(𝑡) 𝑑𝑡
𝐿[𝑒−𝑎𝑡𝑓(𝑎𝑡)] = ∫
0
∞
𝑒−(𝑠+𝑎)𝑡 𝑓(𝑡) 𝑑𝑡
= ∫0
= 𝐹(𝑠 + 𝑎) by (1)
∞
𝑒−𝑠𝑡 𝑒𝑎𝑡𝑓(𝑡) 𝑑𝑡
(ii) 𝐿[𝑒𝑎𝑡𝑓(𝑎𝑡)] = ∫
0
∞
𝑒−(𝑠−𝑎)𝑡 𝑓(𝑡) 𝑑𝑡
= ∫0
= 𝐹(𝑠 − 𝑎) by (1)
Property: 4 Laplace transforms of derivatives 𝑳[𝒇′(𝒕)] = 𝒔𝑳[𝒇(𝒕)] − 𝒇(𝟎)
Proof:
∞ ∞
𝐿[𝑓′(𝑡)] = ∫ 𝑒−𝑠𝑡 𝑓′(𝑡) 𝑑𝑡 = ∫ 𝑢𝑑𝑣
0 0
0
= [𝑢𝑣]∞ − ∫ 𝑢𝑑𝑣
0
∞
𝑓(𝑡) (−𝑠)𝑒−𝑠𝑡 𝑑𝑡
= [𝑒−𝑠𝑡 𝑓(𝑡)]∞ − ∫
0
= 0 − 𝑓(0) + 𝑠𝐿[𝑓(𝑡)]
= 𝑠𝐿[𝑓(𝑡)] − 𝑓(0)
𝑳[𝒇′(𝒕)] = 𝒔𝑳[𝒇(𝒕)] − 𝒇(𝟎)
Property: 5 Laplace transform of derivative of order n
𝑳[𝒇𝒏(𝒕)] = 𝒔𝒏𝑳[𝒇(𝒕)] − 𝒔𝒏−𝟏 𝒇(𝟎) − 𝒔𝒏−𝟐𝒇′(𝟎)⋯ − 𝒔𝒏−𝟑 𝒇′′(𝟎) − ⋯ 𝒇𝒏−𝟏 (𝟎)
Proof:
We know that 𝐿[𝑓′(𝑡)] = 𝑠𝐿[𝑓(𝑡)] − 𝑓(0)⋯ ⋯ (1)
𝐿[𝑓𝑛(𝑡)] = 𝐿[[𝑓′(𝑡)]′]
= 𝑠𝐿[𝑓′(𝑡)] − 𝑓′(0)
= 𝑠[𝑠𝐿[𝑓(𝑡)] − 𝑓(0)] − 𝑓′(0)
= 𝑠2𝐿[𝑓(𝑡)] − 𝑠𝑓(0) − 𝑓′(0)
Similarly, 𝐿[𝑓′′′(𝑡)] = 𝑠3𝐿[𝑓(𝑡)] − 𝑠2𝑓(0) − 𝑠𝑓′(0) − 𝑓′′(0)
In general, 𝑳[𝒇𝒏(𝒕)] = 𝒔𝒏𝑳[𝒇(𝒕)] − 𝒔𝒏−𝟏 𝒇(𝟎) − 𝒔𝒏−𝟐𝒇′(𝟎) ⋯ − 𝒔𝒏−𝟑 𝒇′′(𝟎) − ⋯ 𝒇𝒏−𝟏 (𝟎)
Laplace transform of integrals
𝒕 𝑭(𝒔)
∫
𝟎 𝒇(𝒕)𝒅𝒕] =
𝒔
Theorem: 1 If𝑳[𝒇(𝒕)] = 𝑭(𝒔), then 𝑳 [
Proof:
𝑡
𝑓(𝑡)𝑑𝑡
Let 𝑔(𝑡) = ∫0
∴ 𝑔′(𝑡) = 𝑓(𝑡)
0
𝑓(𝑡)𝑑𝑡 = 0
And 𝑔(0) = ∫0
𝑢 = 𝑒−𝑠𝑡
∴ 𝑑𝑢 = −𝑠𝑒−𝑠𝑡𝑑𝑡
𝑑𝑣 = 𝑓′(𝑡)𝑑𝑡
∴ 𝑣 = ∫ 𝑓′(𝑡)𝑑𝑡
= 𝑓(𝑡)
Engineering Maths - IV
Laplace Transform Page 4
Engineering Maths - IV
Now𝐿[𝑔′(𝑡)] = 𝐿[𝑓(𝑡)]
𝑠𝐿[𝑔(𝑡)] − 𝑔(0) = 𝐿[𝑓(𝑡)]
𝑠𝐿[𝑔(𝑡)] = 𝐿[𝑓(𝑡)] ∴ 𝑔(0) = 0
𝐿[𝑔(𝑡)] = 𝐿[𝑓(𝑡)]
𝑠
𝒕 𝑭(𝒔)
∴ 𝑳 [∫
𝟎 𝒇(𝒕)𝒅𝒕] =
𝒔
𝒅𝒔
Theorem: 2 If𝑳[𝒇(𝒕)] = 𝑭(𝒔), then 𝑳[𝒕𝒇(𝒕)] = − 𝒅
𝑭(𝒔)
Proof:
Given 𝐿[𝑓(𝑡)] = 𝐹(𝑠)
∴ ∫
∞
𝑒−𝑠𝑡 𝑓(𝑡) 𝑑𝑡 = 𝐹(𝑠) ⋯ ⋯ (1)
0
Differentiating (1) with respect to s, we get
𝑑 ∞
𝑒−𝑠𝑡 𝑓(𝑡) 𝑑𝑡 = 𝑑
𝐹(𝑠)
∫
𝑑𝑠 0 𝑑𝑠
𝑑𝑠
∞ 𝜕
(𝑒−𝑠𝑡 )𝑓(𝑡) 𝑑𝑡 = 𝑑
𝐹(𝑠)
∫0 𝜕𝑠
𝑑𝑠
∞ 𝑑
(−𝑡)𝑒−𝑠𝑡 𝑓(𝑡) 𝑑𝑡 = 𝐹(𝑠)
∫0
𝒅𝒔
∞ 𝒅
𝑒−𝑠𝑡 𝑓(𝑡) 𝑑𝑡 = 𝐹(𝑠)
− ∫
0
𝑑𝑠
−𝐿[𝑡𝑓(𝑡)] = 𝑑
𝐹(𝑠)
𝒅𝒔
∴ 𝑳[𝒕𝒇(𝒕)] = − 𝒅
𝑭(𝒔)
𝒅𝒔𝒏
𝒏
Note: In general 𝑳[𝒕𝒏𝒇(𝒕)] = (−𝟏)𝒏 𝒅
𝑭(𝒔)
Example: 5.3 If 𝑳[𝒇(𝒕)] =
𝒔𝟐−𝒔+𝟏
(𝟐𝒔+𝟏)𝟐(𝒔−𝟏)
then find 𝑳[𝒇(𝟐𝒕)].
Solution:
Given 𝐿[𝑓(𝑡)] =
𝑠2−𝑠+1
(2𝑠+1)2(𝑠−1)
= 𝐹(𝑠)
2 2
𝐿[𝑓(2𝑡)] = 1
𝐹 (𝑠
)
=
1
2
𝑠 2 𝑠
( ) − +1
2
2 2
2
𝑠 𝑠
2
(2 +1) ( −1)
= 1
𝑠2 𝑠+4]
[ 4
− 2
𝑠−2
=
2 (𝑠+1)2 ( )
2
𝑠2−2𝑠+1
4(𝑠+1)2(𝑠−2)
Laplace transform of some Standard functions
Result: 1 Prove that 𝑳[𝒕𝒏] = 𝚪(𝒏+𝟏)
𝒔𝒏+𝟏
Proof:
Laplace Transform Page 5
∞
We know that 𝐿[𝑓(𝑡)] = ∫ 𝑒−𝑠𝑡 𝑓(𝑡) 𝑑𝑡
0
∞
𝑒−𝑠𝑡 𝑡𝑛 𝑑𝑡
𝐿[𝑡𝑛] = ∫
0
[ 𝑛] −𝑢
∞ 𝑢 𝑛 𝑑𝑢
𝐿 𝑡 = ∫0 𝑒 (𝑠
)
𝑠
𝑠𝑛+1
∞ 𝑢𝑛
𝑒−𝑢
= ∫0 𝑑𝑢
=
1 ∞
𝑒−𝑢𝑢𝑛 𝑑𝑢
𝑠𝑛+1 ∫0
∴ 𝑳[𝒕𝒏] = 𝚪(𝒏+𝟏)
𝒔𝒏+𝟏
∞ −𝑢 𝑛
∵ ∫0 𝑒 𝑢 𝑑𝑢
Note: If n is an integer, then Γ(𝑛 + 1) = 𝑛!
n!
𝑠𝑛+1
∴ 𝐿[𝑡𝑛] = if n is an integer
If 𝑛 = 0 , then 𝐿[1] = 1
𝑠
If 𝑛 = 1 , then 𝐿[𝑡] = 1
𝑠2
Similarly 𝐿[𝑡2] = 2!
𝑠3
𝐿[𝑡3] = 3!
𝑠4
𝟏
𝒔−𝒂
Result: 2 Prove that 𝑳(𝒆𝒂𝒕) = , 𝒔 > 𝒂
Proof:
We know that 𝐿[𝑓(𝑡)] = ∫
∞
𝑒−𝑠𝑡 𝑓(𝑡) 𝑑𝑡
0
∞
𝑒−𝑠𝑡𝑒𝑎𝑡 𝑑𝑡
∴ 𝐿(𝑒𝑎𝑡) = ∫
0
∞
𝑒−𝑡(𝑠−𝑎) 𝑓(𝑡) 𝑑𝑡
= ∫0
= [ ]
−(𝑠−𝑎) 0
𝑒−𝑡(𝑠−𝑎) ∞
𝑠−𝑎
= − [0 − ( 1
)]
∴ 𝐿(𝑒𝑎𝑡) =
1
𝑠−𝑎
𝟏
𝒔+𝒂
Result: 3 Prove that 𝑳(𝒆−𝒂𝒕) = , 𝒔 > 𝒂
Proof:
We know that 𝐿[𝑓(𝑡)] = ∫
∞
𝑒−𝑠𝑡 𝑓(𝑡) 𝑑𝑡
0
∞
𝑒−𝑠𝑡𝑒−𝑎𝑡 𝑑𝑡
∴ 𝐿(𝑒−𝑎𝑡) = ∫
0
∞
𝑒−𝑡(𝑠+𝑎) 𝑓(𝑡) 𝑑𝑡
= ∫0
= [ ]
−(𝑠+𝑎) 0
𝑒−𝑡(𝑠+𝑎) ∞
𝑠+𝑎
= − [0 − ( 1
)]
∴ 𝐿(𝑒𝑎𝑡) =
1
𝑠+𝑎
Let 𝑠𝑡 = 𝑢 ⋯ ⋯ (1)
𝑡 = 𝑢
𝑠
𝑑𝑡 = 𝑑𝑢
𝑠
When 𝑡 → 0(1) => 𝑢 → 0
,
𝑡 → ∞,(1) => 𝑢 → ∞
Engineering Maths - IV
Laplace Transform Page 6
𝒂
𝒔𝟐+𝒂𝟐
Result: 4 Prove that 𝑳[𝒔𝒊𝒏𝒂𝒕] =
Proof:
∞
𝑒−𝑠𝑡 𝑓(𝑡) 𝑑𝑡
We know that 𝐿[𝑓(𝑡)] = ∫0
∞
𝑒−𝑠𝑡 𝑠𝑖𝑛𝑎𝑡 𝑑𝑡
𝐿[𝑠𝑖𝑛𝑎𝑡] = ∫0
∴ 𝐿[𝑠𝑖𝑛𝑎𝑡] =
𝒂
𝒔𝟐+𝒂𝟐
, 𝑠 > |𝑎| [∵ ∫0
∞
𝑒−𝑎𝑡 𝑠𝑖𝑛𝑏𝑡 𝑑𝑡 =
𝒃
𝒂𝟐+𝒃𝟐
]
𝒔
𝒔𝟐+𝒂𝟐
Result: 5 Prove that 𝑳[𝒄𝒐𝒔𝒂𝒕] =
Proof:
∞
𝑒−𝑠𝑡 𝑓(𝑡) 𝑑𝑡
We know that 𝐿[𝑓(𝑡)] = ∫0
∞
𝑒−𝑠𝑡 𝑐𝑜𝑠𝑎𝑡 𝑑𝑡
𝐿[𝑐𝑜𝑠𝑎𝑡] = ∫0
∴ 𝐿[𝑐𝑜𝑠𝑎𝑡] =
𝑠
𝑠2+𝑎2
, 𝑠 > |𝑎| ∵ ∫0
∞
𝑒−𝑎𝑡 𝑐𝑜𝑠𝑏𝑡 𝑑𝑡 =
𝒂
𝒂𝟐+𝒃𝟐
𝒂
𝒔𝟐−𝒂𝟐
Result: 6 Prove that 𝑳[𝒔𝒊𝒏𝒉𝒂𝒕] =
Proof:
, 𝒔 > |𝒂|
2
𝑎𝑡 −𝑎𝑡
We have 𝐿[𝑠𝑖𝑛ℎ𝑎𝑡] = 𝐿 [𝑒 −𝑒
]
2
= 1
[𝐿(𝑒𝑎𝑡) − 𝐿(𝑒−𝑎𝑡)]
−
2 𝒔−𝒂 𝒔+𝒂
= 1
[ 𝟏 𝟏
]
= 1
[𝒔+𝒂−𝒔+𝒂
]
2 𝒔𝟐−𝒂𝟐
𝟐𝒂
2 𝒔𝟐−𝒂𝟐
= 1
[ ]
∴ 𝐿[𝑠𝑖𝑛ℎ𝑎𝑡] =
𝒂
𝒔𝟐−𝒂𝟐
, 𝑠 > |𝑎|
𝒔
𝒔𝟐−𝒂𝟐
Result: 7 Prove that 𝑳[𝒄𝒐𝒔𝒉𝒂𝒕] =
Proof:
, 𝒔 > |𝒂|
2
𝑎𝑡 −𝑎𝑡
We have 𝐿[𝑐𝑜𝑠ℎ𝑎𝑡] = 𝐿 [𝑒 +𝑒
]
2
= 1
[𝐿(𝑒𝑎𝑡) + 𝐿(𝑒−𝑎𝑡)]
+
2 𝒔−𝒂 𝒔+𝒂
= 1
[ 𝟏 𝟏
]
= 1
[𝒔+𝒂+𝒔−𝒂
]
2 𝒔𝟐−𝒂𝟐
𝟐𝒔
2 𝒔𝟐−𝒂𝟐
= 1
[ ]
∴ 𝐿[𝑐𝑜𝑠ℎ𝑎𝑡] =
𝒔
𝒔𝟐−𝒂𝟐
, 𝑠 > |𝑎|
𝟏
Example: 5.4 Find 𝑳 [𝒕𝟐]
Solution:
We have 𝐿[𝑡𝑛] = Γ(𝑛+1)
𝑠𝑛+1
Engineering Maths - IV
Laplace Transform Page 7
Put 𝑛 = 𝟏
𝟐
𝟏 Γ 𝟏
∴ 𝐿 [𝑡𝟐] = 𝟐
( +1)
𝟏
𝑠𝟐
+1
∵ Γ(𝑛 + 1) = 𝑛Γ𝑛
=
𝟏
Γ 𝟏
𝟐 𝟐
( )
𝟏
𝑠𝟐
+1 𝟐
∵ Γ (𝟏
) = √𝜋
= √𝜋
𝟑
2𝑠𝟐
𝟏
∴ 𝐿 [𝑡𝟐] =
√𝜋
2𝑠√𝑠
− 𝟏
Example: 5.5 Find the Laplace transform of 𝒕 𝟐 or
𝟏
√𝑡
Solution:
We have 𝐿[𝑡𝑛] = Γ(𝑛+1)
𝑠𝑛+1
Put 𝑛 = − 𝟏
𝟐
∴ 𝐿 [𝑡−
𝟏 𝟏
𝟐] = 𝟐
Γ(− +1)
𝟏
𝑠 𝟐
− +1
∵ Γ(𝑛 + 1) = 𝑛Γ𝑛
Γ 𝟏
= 𝟐
( )
𝟏
𝑠𝟐
𝟏
𝟐
∵ Γ ( ) = √𝜋
= √𝜋
√𝑠
∴ 𝐿 𝟏 𝜋
[ ] = √
√𝑡 𝑠
FORMULA
𝑳[𝒇(𝒕)] = 𝑭(𝒔) 𝑳[𝒇(𝒕)] = 𝑭(𝒔)
𝑳[𝟏] = 𝟏
𝒔
𝑳[𝒕] = 𝟏
𝒔𝟐
𝑳[𝒕𝒏] = 𝚪(𝒏+𝟏)
if n is not an integer
𝒔𝒏+𝟏
𝑳[𝒕𝒏] = 𝐧!
if n is an integer
𝒔𝒏+𝟏
𝑳(𝒆𝒂𝒕) = 𝟏
𝒔−𝒂
𝑳(𝒆𝒂𝒕) = 𝟏
𝒔+𝒂
𝒂
𝑳[𝒔𝒊𝒏𝒂𝒕] =
𝒔𝟐 + 𝒂𝟐
𝒔
𝑳[𝒄𝒐𝒔𝒂𝒕] =
𝒔𝟐 + 𝒂𝟐
𝒔
𝑳[𝒄𝒐𝒔𝒉𝒂𝒕] =
𝒔𝟐 − 𝒂𝟐
𝒂
𝑳[𝒔𝒊𝒏𝒉𝒂𝒕] =
𝒔𝟐 − 𝒂𝟐
Problems using Linear property
Example: 5.6 Find the Laplace transform for the following
i. 𝟑𝒕𝟐 + 𝟐𝒕 + 𝟏
ii. (𝒕 + 𝟐)𝟑
iii. 𝒂𝒕
v. 𝒔𝒊𝒏√𝟐 𝒕
vi. 𝒔𝒊𝒏(𝒂𝒕 + 𝒃)
vii. 𝒄𝒐𝒔𝟑𝟐𝒕
ix. 𝒔𝒊𝒏𝟐𝒕
x. 𝒄𝒐𝒔𝟐𝟐𝒕
xi. 𝒄𝒐𝒔𝟓𝒕𝒄𝒐𝒔𝟒𝒕
Engineering Maths - IV
Laplace Transform Page 8
iv. 𝒆𝟐𝒕+𝟑
viii. 𝒔𝒊𝒏𝟑𝒕
Engineering Maths - IV
Solution:
(i) Given 𝑓(𝑡) = 3𝑡2 + 2𝑡 + 1
𝐿[𝑓(𝑡)] = 𝐿[3𝑡2 + 2𝑡 + 1]
= 𝐿[3𝑡2] + 𝐿[2𝑡] + 𝐿[1]
= 𝐿[3𝑡2] + 𝐿[2𝑡] + 𝐿[1]
= 3𝐿[𝑡2] + 2𝐿[𝑡] + 𝐿[1]
𝑠3 𝑠2
= 3 2
+ 2 1
+ 1
𝑠
𝑠3 𝑠2
∴ 𝐿[3𝑡2 + 2𝑡 + 1] = 6
+ 2
+ 1
𝑠
𝑠4 𝑠3 𝑠2
(ii) Given 𝑓(𝑡) = (𝑡 + 2)3 = 𝑡3 + 3𝑡2(2) + 3𝑡22 + 23
𝐿[𝑓(𝑡)] = 𝐿[𝑡3 + 3𝑡2(2) + 3𝑡22 + 23]
= 𝐿[𝑡3] + 𝐿[6𝑡2] + 𝐿[12𝑡] + 𝐿[8]
= 𝐿[𝑡3] + 6𝐿[𝑡2] + 12𝐿[𝑡] + 8𝐿[1]
= 6
+ 12
+ 12
+ 12
𝑠
(𝑖𝑖𝑖) Given 𝑓(𝑡) = 𝑎𝑡
𝐿[𝑓(𝑡)] = 𝐿[𝑎𝑡 ] = 𝐿[𝑒𝑡𝑙𝑜𝑔𝑎 ]
𝐿[𝑎𝑡] =
1
𝑠−𝑙𝑜𝑔
𝑎
(iv)Given 𝑓(𝑡) = 𝑒2𝑡+3
𝐿[𝑓(𝑡)] = 𝐿[𝑒2𝑡+3] = 𝐿[𝑒2𝑡 . 𝑒3]
= 𝑒3𝐿[𝑒2𝑡]
𝑠−2
1
= 𝑒3 [ ]
𝑠−2
1
∴ 𝐿[𝑒2𝑡+3] = 𝑒3 [ ]
(v) 𝐿[𝑠𝑖𝑛√2𝑡] = √2
𝑠2+2
(vi)Given 𝑓(𝑡) = sin(𝑎𝑡 + 𝑏) = 𝑠𝑖𝑛𝑎𝑡𝑐𝑜𝑠𝑏 + 𝑐𝑜𝑠𝑎𝑡𝑠𝑖𝑛𝑏
𝐿[𝑓(𝑡)] = 𝐿[sin(𝑎𝑡 + 𝑏)]
= 𝐿[𝑠𝑖𝑛𝑎𝑡𝑐𝑜𝑠𝑏 + 𝑐𝑜𝑠𝑎𝑡𝑠𝑖𝑛𝑏]
= 𝑐𝑜𝑠𝑏 𝐿[sin𝑎𝑡] + 𝑠𝑖𝑛𝑏 𝐿[cos𝑎𝑡]
𝐿[sin(𝑎𝑡 + 𝑏)] = 𝑐𝑜𝑠𝑏 + 𝑠𝑖𝑛𝑏
𝑠 𝑠
𝑠2+𝑎2 𝑠2+𝑎2
4
(vii) Given 𝑓(𝑡) = cos3 2𝑡 = 1
[3𝑐𝑜𝑠2𝑡 + 𝑐𝑜𝑠6𝑡]
4
𝐿[𝑓(𝑡)] = 1
𝐿[3𝑐𝑜𝑠2𝑡 + 𝑐𝑜𝑠6𝑡]
4
= 1
[3𝐿(𝑐𝑜𝑠2𝑡) + 𝐿(𝑐𝑜𝑠6𝑡)]
= 1
[3 +
𝑠 𝑠
4 𝑠2+4 𝑠2+36
]
3𝑐𝑜𝑠𝜃 + 𝑐𝑜𝑠3𝜃
∵ cos3 𝜃 =
4
Laplace Transform Page 9
𝐿[cos3 2𝑡] = 1
[3 +
𝑠 𝑠
4 𝑠2+4 𝑠2+36
]
4
(viii) Given 𝑓(𝑡) = sin3 𝑡 = 1
[3𝑠𝑖𝑛𝑡 − 𝑠𝑖𝑛3𝑡]
4
𝐿[𝑓(𝑡)] = 1
𝐿[3𝑠𝑖𝑛𝑡 − 𝑠𝑖𝑛3𝑡]
4
= 1
[3𝐿(𝑠𝑖𝑛𝑡) − 𝐿(𝑠𝑖𝑛3𝑡)]
= 1
[3 −
1 3
4 𝑠2+1 𝑠2+9
]
𝐿[sin3 𝑡] = 3
[ −
1 1
4 𝑠2+1 𝑠2+9
]
(ix) Given 𝑓(𝑡) = sin2 𝑡 = 1−𝑐𝑜𝑠2𝑡
2
2
𝐿[𝑓(𝑡)] = 𝐿 [1−𝑐𝑜𝑠2𝑡
]
2
= 1
[𝐿(1) − 𝐿(𝑐𝑜𝑠2𝑡)]
= 1
[1
−
2 𝑠 𝑠2+4
𝑠
]
𝐿[cos2 2𝑡] = 1
[1
−
2 𝑠 𝑠2+4
𝑠
]
(x) Given 𝑓(𝑡) = cos2 2𝑡 = 1+𝑐𝑜𝑠4𝑡
2
2
𝐿[𝑓(𝑡)] = 𝐿 [1+𝑐𝑜𝑠4𝑡
]
2
= 1
[𝐿(1) + 𝐿(𝑐𝑜𝑠4𝑡)]
= 1
[1
+
2 𝑠 𝑠2+16
𝑠
]
𝐿[cos2 2𝑡] = 1
[1
+
2 𝑠 𝑠2+16
𝑠
]
(xi) Given 𝑓(𝑡) = 𝑐𝑜𝑠5𝑡𝑐𝑜𝑠4𝑡
𝐿[𝑓(𝑡)] = 𝐿[𝑐𝑜𝑠5𝑡𝑐𝑜𝑠4𝑡]
2
= 1
[𝐿(𝑐𝑜𝑠9𝑡) + 𝐿(𝑐𝑜𝑠𝑡)]
= 1
[
𝑠 𝑠
2 𝑠2+81 𝑠2+1
+ ]
Problems using First Shifting theorem
𝑳[𝒆−𝒂𝒕𝒇(𝒕)] = 𝑳[𝒇(𝒕)]𝒔→𝒔+𝒂
𝑳[𝒆𝒂𝒕𝒇(𝒕)] = 𝑳[𝒇(𝒕)]𝒔→𝒔−𝒂
Example: 5.7 Find the Laplace transform for the following:
i.
ii.
𝒕𝒆−𝟑𝒕
𝒕𝟑𝒆𝟐𝒕
iii. 𝒆𝟒𝒕𝒔𝒊𝒏𝟐𝒕
iv. 𝒆−𝟓𝒕𝒄𝒐𝒔𝟑𝒕
v. 𝒔𝒊𝒏𝒉𝟐𝒕𝒄𝒐𝒔𝟑𝒕
vii. 𝒕𝟐𝟐𝒕
viii. 𝒕𝟑𝟐−𝒕
ix. 𝒆−𝟐𝒕𝒔𝒊𝒏𝟑𝒕𝒄𝒐𝒔𝟐𝒕
x. 𝒆−𝟑𝒕𝒄𝒐𝒔𝟒𝒕𝒄𝒐𝒔𝟐𝒕
xi. 𝒆𝟒𝒕𝒄𝒐𝒔𝟑𝒕𝒔𝒊𝒏𝟐𝒕
Engineering Maths - IV
Laplace Transform
Page
10
vi. cosh3tsin2t
(i) 𝒕𝒆−𝟑𝒕
𝐿[𝑡𝑒−3𝑡] = 𝐿[𝑡]𝑠→𝑠+3
= ( 1
)
𝑠2
𝑠→𝑠+3
∵ 𝐿(𝑡) = 1
𝑠2
∴ 𝐿[𝑡𝑒−3𝑡] =
1
(𝑠+3)2
(ii) 𝒕𝟑𝒆𝟐𝒕
𝐿[𝑡3𝑒2𝑡] = 𝐿[𝑡3]𝑠→𝑠−2
= (3!
)
𝑠4
𝑠→𝑠−2
∵ 𝐿(𝑡) =
3!
𝑠3+1
∴ 𝐿[𝑡3𝑒2𝑡] =
6
(𝑠−2)4
(iii) 𝒆𝟒𝒕𝒔𝒊𝒏𝟐𝒕
𝐿[𝑒4𝑡𝑠𝑖𝑛2𝑡] = 𝐿[𝑠𝑖𝑛2𝑡]𝑠→𝑠−4
2
= ( )
=
𝑠2+22
𝑠→𝑠−4
2
=
(𝑠−4)2+4
2
𝑠2−8𝑠+16+4
∴ 𝐿[𝑒4𝑡𝑠𝑖𝑛2𝑡] =
2
𝑠2−8𝑠+20
(iv) 𝑳[𝒆−𝟓𝒕𝒄𝒐𝒔𝟑𝒕]
𝐿[𝑒−5𝑡𝑐𝑜𝑠3𝑡] = 𝐿[𝑐𝑜𝑠3𝑡]𝑠→𝑠+5
𝑠
= ( )
=
𝑠2+32
𝑠→𝑠+5
𝑠+5
=
(𝑠+5)2+9
𝑠+5
𝑠2+10𝑠+25+9
𝑠+5
𝑠2+10𝑠+34
∴ 𝐿[𝑒−5𝑡𝑐𝑜𝑠3𝑡] =
(v) 𝑳[𝒔𝒊𝒏𝒉𝟐𝒕𝒄𝒐𝒔𝟑𝒕]
2
2𝑡 −2𝑡
𝐿[𝑠𝑖𝑛ℎ2𝑡𝑐𝑜𝑠3𝑡] = 𝐿 [(𝑒 −𝑒
) 𝑐𝑜𝑠3𝑡]
2
= 1
[𝐿(𝑒2𝑡𝑐𝑜𝑠3𝑡) − 𝐿(𝑒−2𝑡𝑐𝑜𝑠3𝑡)]
2 𝑠→𝑠−2
= 1
[𝐿(𝑐𝑜𝑠3𝑡) − 𝐿(𝑐𝑜𝑠3𝑡)𝑠→𝑠+2]
2
= 1
[( )
𝑠 𝑠
− ( )
𝑠2+32
𝑠→𝑠−2 𝑠2+32
𝑠→𝑠+2
]
∴ 𝐿[𝑠𝑖𝑛ℎ2𝑡𝑐𝑜𝑠3𝑡] = 1
[
𝑠−2 𝑠+2
2 (𝑠−2)2+9 (𝑠+2)2+9
− ]
(vi) 𝑳[𝒄𝒐𝒔𝒉𝟑𝒕𝒔𝒊𝒏𝟐𝒕]
2
3𝑡 −3𝑡
𝐿[𝑐𝑜𝑠ℎ3𝑡𝑠𝑖𝑛2𝑡] = 𝐿 [(𝑒 +𝑒
) 𝑠𝑖𝑛2𝑡]
Engineering Maths - IV
Laplace Transform
Page
11
2
= 1
[𝐿(𝑒3𝑡𝑠𝑖𝑛2𝑡) + 𝐿(𝑒−3𝑡𝑠𝑖𝑛2𝑡)]
2 𝑠→𝑠−3
= 1
[𝐿(𝑠𝑖𝑛2𝑡) + 𝐿(𝑠𝑖𝑛2𝑡)𝑠→𝑠+3]
2
= 1
[( )
2 2
+ ( )
𝑠2+22
𝑠→𝑠−3 𝑠2+22
𝑠→𝑠+3
]
∴ 𝐿[𝑐𝑜𝑠ℎ3𝑡𝑠𝑖𝑛2𝑡] = 1
[
2 2
2 (𝑠−3)2+4 (𝑠+3)2+4
+ ]
(vii) 𝒕𝟐𝟐𝒕
𝐿[𝑡22𝑡] = 𝐿[𝑡2𝑒𝑙𝑜𝑔2𝑡
]
= 𝐿[𝑡2𝑒𝑡𝑙𝑜𝑔2] = 𝐿[𝑡2]𝑠→𝑠−𝑙𝑜𝑔2
= (2!
)
𝑠3
𝑠→𝑠−𝑙𝑜𝑔2
2
=
(𝑠−𝑙𝑜𝑔2)3
∴ 𝐿[𝑡22𝑡] =
2
(𝑠−𝑙𝑜𝑔2)3
(viii) 𝒕𝟑𝟐−𝒕
𝐿[𝑡32−𝑡] = 𝐿[𝑡3𝑒𝑙𝑜𝑔2−𝑡
]
= 𝐿[𝑡3𝑒−𝑡𝑙𝑜𝑔2] = 𝐿[𝑡3]𝑠→𝑠+𝑙𝑜𝑔2
= (3!
)
=
𝑠4
𝑠→𝑠+𝑙𝑜𝑔2
6
∴ 𝐿[𝑡32−𝑡] =
(𝑠+𝑙𝑜𝑔2)4
6
(𝑠+𝑙𝑜𝑔2)4
(ix) 𝑳[𝒆−𝟐𝒕𝒔𝒊𝒏𝟑𝒕𝒄𝒐𝒔𝟐𝒕]
𝐿[𝑒−2𝑡𝑠𝑖𝑛3𝑡𝑐𝑜𝑠2𝑡] = 𝐿[𝑠𝑖𝑛3𝑡𝑐𝑜𝑠2𝑡]𝑠→𝑠+2
2
= 1
𝐿[sin(3𝑡 + 2𝑡) + sin(3𝑡 − 2𝑡)]𝑠→𝑠+2
2
= 1
𝐿[sin 5𝑡 + sin 𝑡]𝑠→𝑠+2
2
= 1
[L(sin 5𝑡) + 𝐿(sin 𝑡)]𝑠→𝑠+2
= 1
[
2 𝑠2+52
5 1
+ ]
𝑠2+12
𝑠→𝑠+2
= 1
[ +
5 1
2 (𝑠+2)2+25 (𝑠+2)2+1
]
∴ 𝐿[𝑒−2𝑡𝑠𝑖𝑛3𝑡𝑐𝑜𝑠2𝑡] == 1
[ +
5 1
2 (𝑠+2)2+25 (𝑠+2)2+1
]
(x) 𝑳[𝒆−𝟑𝒕𝒄𝒐𝒔𝟒𝒕𝒄𝒐𝒔𝟐𝒕]
𝐿[𝑒−3𝑡𝑐𝑜𝑠4𝑡𝑐𝑜𝑠2𝑡] = 𝐿[𝑐𝑜𝑠4𝑡𝑐𝑜𝑠2𝑡]𝑠→𝑠+3
2
= 1
𝐿[cos(4𝑡 + 2𝑡) + cos(4𝑡 − 2𝑡)]𝑠→𝑠+3
2
= 1
𝐿[cos6t + cos2𝑡]𝑠→𝑠+3
Engineering Maths - IV
Laplace Transform
Page
12
Engineering Mathematics - II
2
= 1
[L(cos6𝑡) + 𝐿(cos2 𝑡)]𝑠→𝑠+3
= 1
[
2 𝑠2+62
𝑠 𝑠
+ ]
𝑠2+22
𝑠→𝑠+3
= 1
[ +
𝑠+3 𝑠+3
2 (𝑠+3)2+36 (𝑠+3)2+4
]
∴ 𝐿[𝑒−3𝑡𝑐𝑜𝑠4𝑡𝑐𝑜𝑠2𝑡] = 1
[
𝑠+3 𝑠+3
2 (𝑠+3)2+36 (𝑠+3)2+4
+ ]
(xi) 𝑳[𝒆𝟒𝒕𝒄𝒐𝒔𝟑𝒕𝒔𝒊𝒏𝟐𝒕]
𝐿[𝑒4𝑡𝑐𝑜𝑠3𝑡𝑠𝑖𝑛2𝑡] = 𝐿[𝑐𝑜𝑠3𝑡𝑠𝑖𝑛2𝑡]𝑠→𝑠−4
2
= 1
𝐿[sin(3𝑡 + 2𝑡) − sin(3𝑡 − 2𝑡)]𝑠→𝑠−4
2
= 1
𝐿[sin 5𝑡 − sin 𝑡]𝑠→𝑠−4
2
= 1
[L(sin 5𝑡) − 𝐿(sin 𝑡)]𝑠→𝑠−4
= 1
[
2 𝑠2+52
5 1
− ]
𝑠2+12
𝑠→𝑠−4
= 1
[ +
5 1
2 (𝑠−4)2+25 (𝑠−4)2+1
]
∴ 𝐿[𝑒4𝑡𝑐𝑜𝑠3𝑡𝑠𝑖𝑛2𝑡] = 1
[
5 1
2 (𝑠−4)2+25 (𝑠−4)2+1
+ ]
Exercise: 5.1
Find the Laplace transform for the following
1. cos2 3𝑡 Ans: 𝟏
[ 𝟑𝒔
+
𝒔
𝟒 𝒔𝟐+𝟗 𝒔𝟐+𝟖𝟏
]
2. 𝑠𝑖𝑛3𝑡𝑐𝑜𝑠4𝑡 Ans: 𝟏
[
𝟕 𝟏
− ]
3. 𝑡𝑒2𝑡 Ans:
𝟒 𝒔𝟐+𝟒𝟗 𝒔𝟐+𝟏
𝟏
4. 𝑡4𝑒−3𝑡
Ans:
(𝒔−𝟐)𝟐
𝟒!
5. 𝑒4𝑡𝑠𝑖𝑛2𝑡 Ans:
(𝒔−𝟑)𝟓
𝟐
6. 𝑒−5𝑡𝑐𝑜𝑠3𝑡 Ans:
(𝒔−𝟒)𝟐+𝟒
𝒔+𝟓
7. 𝑡33𝑡 Ans:
(𝒔+𝟓)𝟐+𝟗
𝟑!
8. 𝑡54−𝑡 Ans:
(𝒔−𝒍𝒐𝒈𝟑)𝟒
𝟓!
(𝒔+𝒍𝒐𝒈𝟒)𝟔
9. 𝑒−2𝑡𝑠𝑖𝑛3𝑡𝑐𝑜𝑠2𝑡 Ans: +
𝟓 𝟏
(𝒔+𝟐)𝟐+𝟐𝟓 (𝒔+𝟐)𝟐+𝟏
10. 𝑒−3𝑡𝑐𝑜𝑠4𝑡𝑐𝑜𝑠2𝑡 Ans: +
𝒔+𝟑 𝒔+𝟑
(𝒔+𝟑)𝟐+𝟑𝟔 (𝒔+𝟑)𝟐+𝟒
11. 𝑠𝑖𝑛ℎ𝑡𝑠𝑖𝑛4𝑡 Ans: −
𝟒 𝟒
(𝒔−𝟏)𝟐+𝟏𝟔 (𝒔+𝟏)𝟐+𝟏𝟔
12. 𝑐𝑜𝑠ℎ2𝑡𝑐𝑜𝑠2𝑡 Ans: 1
[ −
𝑠−2 𝑠+2
2 (𝑠−2)2+4 (𝑠+2)2+4
]
Laplace Transform
Page
13
Engineering Maths - IV
5.4 LAPLACE TRANSFORM OF DERIVATIVES AND INTEGRALS
Problems using the formula
𝑳[𝒕𝒇(𝒕)] = −𝒅
𝑳[𝒇(𝒕)]
𝒅𝒔
Example: 5.8 Find the Laplace transform for 𝒕𝒔𝒊𝒏𝟒𝒕
Solution:
𝑑𝑠
𝐿[𝑡𝑠𝑖𝑛4𝑡] = −𝑑
𝐿[𝑡𝑠𝑖𝑛4𝑡]
= −𝑑
[ 4
𝑑𝑠 𝑠2+4
]
2
= −[(𝑠 +16)0−4(2𝑠)]
(𝑠2+16)2
∴ 𝐿[𝑡𝑠𝑖𝑛4𝑡] =
8𝑠
(𝑠2+16)2
Example: 5.9 Find 𝑳[𝒕𝒔𝒊𝒏𝟐𝒕]
Solution:
𝐿[𝑡𝑠𝑖𝑛2𝑡] = −𝑑
𝐿[sin2 𝑡] = −𝑑
𝐿 [(1−𝑐𝑜𝑠2𝑡)
]
𝑑𝑠 𝑑𝑠 2
2 𝑑𝑠
= − 1 𝑑
[𝐿(1) − 𝐿(𝑐𝑜𝑠2𝑡)]
𝒔
2 𝑑𝑠 𝑠 𝒔𝟐+𝟒
= − 1 𝑑
[1
− ]
𝟐 𝟐
= − 1 𝑑
[𝒔 +𝟒−𝒔
]
2 𝑑𝑠 𝒔(𝒔𝟐+𝟒)
= − 1 𝑑
[
2 𝑑𝑠 𝒔(𝒔𝟐+𝟒)
𝟒
]
= − 4 𝑑
[
2 𝑑𝑠 𝒔(𝒔𝟐+𝟒)
𝟏
]
𝟐
= −2 [𝟎−(𝟑𝒔 +𝟒)
]
(𝒔𝟑+𝟒𝒔)
𝟐
𝟐
∴ 𝐿[𝑡𝑠𝑖𝑛2𝑡] = 𝟐(𝟑𝒔 +𝟒)
(𝒔𝟑+𝟒𝒔)
𝟐
Example: 5.10 Find 𝑳[𝒕𝒄𝒐𝒔𝟐𝟐𝒕]
Solution:
𝐿[𝑐𝑜𝑠22𝑡] = −𝑑
𝐿[cos2 2𝑡] = −𝑑
𝐿 [(1+𝑐𝑜𝑠4𝑡)
]
𝑑𝑠 𝑑𝑠 2
2 𝑑𝑠
= − 1 𝑑
[𝐿(1) + 𝐿(𝑐𝑜𝑠4𝑡)]
= − 1 𝑑
[1
+
2 𝑑𝑠 𝑠 𝒔𝟐+𝟏𝟔
𝒔
]
𝑠2
2 (𝒔𝟐+𝟏𝟔)𝟐
𝟐
= − 1
[− 1
+ (𝒔 +𝟏𝟔)𝟏−𝒔.𝟐𝒔
]
𝑠2
2 (𝒔𝟐+𝟏𝟔)𝟐
𝟐 𝟐
= − 1
[− 1
+ 𝒔 +𝟏𝟔−𝟐𝒔
]
∴ 𝐿[𝑐𝑜𝑠22𝑡] = 1
[ 1
−
2 𝑠2 (𝒔𝟐+𝟏𝟔)𝟐
𝟏𝟔−𝒔𝟐
]
Laplace Transform
Page
14
Engineering Maths - IV
Example: 5.11 Find the Laplace transform for 𝒕𝒔𝒊𝒏𝒉𝟐𝒕
Solution:
𝑑𝑠
𝐿[𝑠𝑖𝑛ℎ2𝑡] = −𝑑
𝐿[𝑠𝑖𝑛ℎ2𝑡]
= −𝑑
[ 2
𝑑𝑠 𝑠2−4
]
2
= −[(𝑠 −4)0−2(2𝑠)]
(𝑠2−4)2
∴ 𝐿[𝑡𝑠𝑖𝑛ℎ2𝑡] =
4𝑠
(𝑠2−4)2
Example: 5.12 Find the Laplace transform for 𝒇(𝒕) = 𝒔𝒊𝒏𝒂𝒕 − 𝒂𝒕𝒄𝒐𝒔𝒂𝒕
Solution:
𝐿[𝑠𝑖𝑛𝑎𝑡 − 𝑎𝑡𝑐𝑜𝑠𝑎𝑡] = L(sin 𝑎𝑡) − 𝑎 L(tcosat)
=
𝑎
𝑑𝑠
− 𝑎 (−𝑑
𝐿[𝑐𝑜𝑠𝑎𝑡])
=
𝑠2+𝑎2
𝑎 𝑠
𝑠2+𝑎2 𝑑𝑠 𝑠2+𝑎2
+ 𝑎 𝑑
[ ]
=
=
2 2
𝑎
+ 𝑎 [(𝑠 +𝑎 )1−𝑠(2𝑠)
]
𝑠2+𝑎2 (𝑠2+𝑎2)2
𝑎
+ 𝑎 [𝑠2+𝑎2−𝑠2
]
𝑠2+𝑎2 (𝑠2+𝑎2)2
=
𝑎
𝑠2+𝑎2 (𝑠2+𝑎2)2
2 2
+ 𝑎 [ 𝑎 −𝑠
]
2 2 2 2
= 𝑎(𝑠 +𝑎 )+𝑎(𝑎 −𝑠 )
(𝑠2+𝑎2)2
2 3 3 2
= 𝑎𝑠 +𝑎 +𝑎 −𝑎𝑠 )
(𝑠2+𝑎2)2
∴ 𝐿[𝑠𝑖𝑛𝑎𝑡 − 𝑎𝑡𝑐𝑜𝑠𝑎𝑡] =
2𝑎3
(𝑠2+𝑎2)2
Example: 5.13 Find the Laplace transform for the following
(i) 𝒕𝒆−𝟑𝒕𝒔𝒊𝒏𝟐𝒕 (ii) 𝒕𝒆−𝒕𝒄𝒐𝒔𝒂𝒕 9iii) 𝒕𝒔𝒊𝒏𝒉𝒕𝒄𝒐𝒔𝟐𝒕
Solution:
𝑑𝑠
(i) 𝐿[𝑡𝑒−3𝑡𝑠𝑖𝑛2𝑡] = 𝐿[𝑡𝑠𝑖𝑛2𝑡] = −𝑑
𝐿[𝑠𝑖𝑛2𝑡]
𝑠→𝑠+3 𝑠→𝑠+3
𝑑𝑠
= −𝑑
( 2
)
𝑠2+22
𝑠→𝑠+3
(𝑠2+4)2
2
= [(𝑠 +4)0−2(2𝑠)
]
𝑠→𝑠+3
= [
4𝑠
]
(𝑠2+4)2
𝑠→𝑠+3
∴ 𝐿[𝑡𝑒−3𝑡 𝑠𝑖𝑛2𝑡] =
4(𝑠+3)
((𝑠+3)2+4 )2
𝑠→𝑠+1 𝑑𝑠
(ii) 𝐿[𝑡𝑒−𝑡𝑐𝑜𝑠𝑎𝑡] = 𝐿[𝑡𝑐𝑜𝑠𝑎𝑡] = −𝑑
𝐿[𝑐𝑜𝑠𝑎𝑡]𝑠→𝑠+1
𝑑𝑠
= −𝑑
( 𝑠
)
𝑠2+𝑎2
𝑠→𝑠+1
Laplace Transform
Page
15
(𝑠2+𝑎2)2
Engineering Maths - IV
2 2
= − [(𝑠 +𝑎 )1−𝑠(2𝑠)
]
𝑠→𝑠+1
2 2
= − [ 𝑎 −𝑠
]
(𝑠2+𝑎2)2
𝑠→𝑠+1
2 2
= [ 𝑠 −𝑎
]
(𝑠2+𝑎2)2
𝑠→𝑠+1
∴ 𝐿[𝑡𝑒−𝑡𝑐𝑜𝑠𝑎𝑡] =
(𝑠+1)2−𝑎2
((𝑠+1)2+𝑎2 )2
(iii) 𝐿[𝑡𝑠𝑖𝑛ℎ𝑡𝑐𝑜𝑠2𝑡]
2
𝑡 −𝑡
𝐿[𝑡𝑠𝑖𝑛ℎ𝑡𝑐𝑜𝑠2𝑡] = 𝐿 [𝑡 (𝑒 −𝑒
) 𝑐𝑜𝑠2𝑡]
2
= 1
[𝐿(𝑡𝑒𝑡𝑐𝑜𝑠2𝑡) − 𝐿(𝑡𝑒−𝑡𝑐𝑜𝑠2𝑡)]
2 𝑑𝑠 𝑑𝑠
= 1
[−𝑑
𝐿[𝑐𝑜𝑠2𝑡] + 𝑑
𝐿[𝑐𝑜𝑠2𝑡]
𝑠→𝑠−1 𝑠→𝑠+1]
2 𝑑𝑠
= 1
[−𝑑
( 𝑠
𝑠2+4 𝑠→𝑠−1 𝑑𝑠
𝑠
𝑠2+4 𝑠→𝑠+1
) + 𝑑
( ) ]
2 (𝑠2+4)2
2
= 1
[− [(𝑠 +4)1−𝑠(2𝑠)
]
𝑠→𝑠−1 (𝑠2+4)2
2
+ [(𝑠 +4)1−𝑠(2𝑠)
]
𝑠→𝑠+1
]
2
= 1
[− [ 4−𝑠
]
2
+ [ 4−𝑠
]
2 (𝑠2+4)2
𝑠→𝑠−1 (𝑠2+4)2
𝑠→𝑠+1
]
2
∴ 𝐿[𝑡𝑠𝑖𝑛ℎ𝑡𝑐𝑜𝑠2𝑡] = 1
[ (𝑠−1) −4
+
4−(𝑠+1)2
2 ((𝑠−1)2+4 )2 ((𝑠+1)2+4 )2
]
Problems using the formula
𝟐
𝑳[𝒕𝟐𝒇(𝒕)] = 𝒅
𝑳[𝒇(𝒕)]
𝒅𝒔𝟐
Example: 5.14 Find the Laplace transform for (i)𝒕𝟐𝒔𝒊𝒏𝒕 (ii) 𝒕𝟐𝒄𝒐𝒔𝟐𝒕
Solution:
𝑑𝑠2
2
(i) 𝐿[𝑡2𝑠𝑖𝑛𝑡] = 𝑑
𝐿[𝑠𝑖𝑛𝑡]
1
𝑑𝑠2 𝑠2+1
2
= 𝑑
[ ]
2
= 𝑑
([(𝑠 +1)0−1(2𝑠)]
)
𝑑𝑠 (𝑠2+1)2
= 𝑑
(
𝑑𝑠 (𝑠2+1)2
−2𝑠
)
= −2 𝑑
(
𝑑𝑠 (𝑠2+1)2
𝑠
)
=
2
2 2
−2[(𝑠 +1) (1)−𝑠(2)(𝑠 +1)(2𝑠)]
(𝑠2+1)4
2 2 2
= −2(𝑠 +1)[(𝑠 +1)−4𝑠 ]
(𝑠2+1)4
2
= −2[1−3𝑠 ]
(𝑠2+1)3
2
∴ 𝐿[𝑡2𝑠𝑖𝑛𝑡] = 6𝑠 −2
(𝑠2+1)3
Laplace Transform
Page
16
𝑑𝑠2
2
(ii) 𝐿[𝑡2𝑐𝑜𝑠2𝑡] = 𝑑
𝐿[𝑐𝑜𝑠2𝑡]
𝑠
2
= 𝑑
[ ]
= (
𝑑𝑠2 𝑠2+4
2
𝑑 [(𝑠 +4)1−𝑠(2𝑠)]
𝑑𝑠 (𝑠2+4)2
)
2
= 𝑑
( 4−𝑠
)
𝑑𝑠 (𝑠2+4)2
=
2 2
( 2 2
[(𝑠 +4) −2𝑠)−(4−𝑠 )2(𝑠 +4)(2𝑠)]
(𝑠2+4)4
2 2 2
= 2𝑠(𝑠 +4)[(𝑠 +4)(−1)−(4−𝑠 )2]
(𝑠2+4)4
2
= 2𝑠[𝑠 −12]
(𝑠2+4)3
2
∴ 𝐿[𝑡2 cos2𝑡] = 2𝑠[𝑠 −12]
(𝑠2+4)3
Example: 5.15 Find the Laplace transform for (i)𝒕𝟐𝒆−𝟐𝒕𝒄𝒐𝒔𝒕 (ii) 𝒕𝟐𝒆𝟒𝒕𝒔𝒊𝒏𝟑𝒕
Solution:
(i) 𝐿[𝑡2𝑒−2𝑡𝑐𝑜𝑠𝑡] = 𝐿[𝑡2𝑐𝑜𝑠𝑡]𝑠→𝑠+2 𝑑𝑠2
2
= 𝑑
𝐿[𝑐𝑜𝑠𝑡]𝑠→𝑠+2
𝑑𝑠2
𝑠
2
= 𝑑
( )
𝑠2+1 𝑠→𝑠+2
𝑑𝑠 (𝑠2+1)2
2
= 𝑑
[(𝑠 +1)1−𝑠(2𝑠)
]
𝑠→𝑠+2
2
= 𝑑
[ 1−𝑠
]
𝑑𝑠 (𝑠2+1)2
𝑠→𝑠+2
= [
2 2
( 2 2
[(𝑠 +1) −2𝑠)−(1−𝑠 )2(𝑠 +1)(2𝑠)]
(𝑠2+1)4
]
𝑠→𝑠+2
(𝑠2+1)4
2 2
= (𝑠2 + 1)[[(𝑠 +1)(−2𝑠)−4𝑠(1−𝑠 )]
]
𝑠→𝑠+2
(𝑠2+1)3
3 3
= [−2𝑠 −2𝑠−4𝑠+4𝑠
]
𝑠→𝑠+2
(𝑠2+1)3
3
= [2𝑠 −6𝑠
]
𝑠→𝑠+2
∴
3
𝐿[𝑡2𝑒−2𝑡𝑐𝑜𝑠𝑡] = 2(𝑠+2) −6(𝑠+2)
((𝑠+2)2+1 )3
(ii)𝐿[𝑡2𝑒4𝑡𝑠𝑖𝑛3𝑡] = 𝐿[𝑡2𝑠𝑖𝑛3𝑡] 𝑠→𝑠−4 𝑑𝑠2
2
= 𝑑
𝐿[𝑠𝑖𝑛3𝑡]𝑠→𝑠−4
𝑑𝑠2
3
2
= 𝑑
( )
𝑠2+9 𝑠→𝑠−4
𝑑𝑠 (𝑠2+9)2
2
= 𝑑
[(𝑠 +9)0−3(2𝑠)
]
= 𝑑
[
−6𝑠
]
𝑑𝑠 (𝑠2+9)2
𝑠→𝑠−4
𝑠→𝑠−4
= −6 𝑑
[
𝑠
]
𝑑𝑠 (𝑠2+9)2
𝑠→𝑠−4
= −6 [
2
2
[(𝑠 +9) ( ) 2
1 −(𝑠)2(𝑠 +9)(2𝑠)]
(𝑠2+9)4
]
𝑠→𝑠−4
Engineering Maths - IV
Laplace Transform
Page
17
(𝑠2+9)4
Engineering Mathematics - II
2 2
= −6(𝑠2 + 9)[[(𝑠 +9)−4𝑠 ]
]
𝑠→𝑠−4
(𝑠2+9)3
2
= −6 [ 9−3𝑠
]
𝑠→𝑠−4
(𝑠2+9)3
2
= [18𝑠 −54
]
𝑠→𝑠−4
2
∴ 𝐿[𝑡2𝑒4𝑡𝑠𝑖𝑛3𝑡] = 18(𝑠−4) −54
((𝑠−4)2+9 )3
Exercise: 5.2
Find the Laplace transform for the following
2𝑎𝑠
(𝑠2+𝑎2)2
𝑠2−𝑎2
(𝑠2+𝑎2)2
6(𝑠+4)
(𝑠+4)2+9
8𝑠
(𝑠2+64)2
−
4𝑠
(𝑠2+16)2
(𝑠−2)2−4
((𝑠+4)2+4)2
1. 𝑡𝑠𝑖𝑛𝑎𝑡 Ans:
2. 𝑡𝑐𝑜𝑠𝑎𝑡 Ans:
3. 𝑡𝑒−4𝑡𝑠𝑖𝑛3𝑡 Ans:
4. 𝑡𝑐𝑜𝑠2𝑡𝑠𝑖𝑛6𝑡 Ans:
5. 𝑡𝑒−2𝑡𝑐𝑜𝑠2𝑡 Ans:
Problems using the formula
𝑳 [𝒇(𝒕)
] =
𝒕
∞
∫𝒔 𝑳[𝒇(𝒕)]𝒅𝒔
𝑡→0 𝑡
This formula is valid if lim 𝑓(𝑡)
is finite.
The following formula is very useful in this section
𝑠
𝑑𝑠
∫ = 𝑙𝑜𝑔𝑠
𝑠+𝑎
𝑑𝑠
∫ = log(𝑠 + 𝑎)
𝑠 𝑑𝑠 1
∫ = log(𝑠2 + 𝑎2)
𝑠2+𝑎2 2
𝑎 𝑑𝑠 𝑠
∫ = tan−1
𝑠2+𝑎2 𝑎
𝒕
Example: 5.16 Find 𝑳 [𝒄𝒐𝒔𝒂𝒕
]
𝑡→0 𝑡 0 0
Solution:
lim 𝑐𝑜𝑠𝑎𝑡
= 𝑐𝑜𝑠𝑎(0)
= 1
= ∞
∴ Laplace transform does not exists.
𝒕
Example: 5.17 Find 𝑳 [𝒔𝒊𝒏𝒂𝒕
]
𝑡→0
Solution:
lim 𝑠𝑖𝑛𝑎𝑡
= 𝑠𝑖𝑛𝑎(0)
= 0
𝑡 0 0
= lim𝑎𝑐𝑜𝑠𝑎𝑡
𝑡→0
(by applying L−Hospital rule)
Laplace Transform
Page
18
Engineering Maths - IV
lim𝑎𝑐𝑜𝑠𝑎𝑡 = 𝑎𝑐𝑜𝑠0 = 𝑎, finite quantity.
𝑡→0
Hence Laplace transform exists
𝑡
𝐿 [𝑠𝑖𝑛𝑎𝑡
] =
∞
𝐿[(𝑠𝑖𝑛𝑎𝑡)]𝑑𝑠
∫𝑠
∞ 𝑎
= ∫𝑠 𝑠2+𝑎2
𝑑𝑠
= [tan ]
𝑎 𝑠
−1 𝑠 ∞
𝑎
= [tan−1∞ − tan−1 𝑠
]
= [𝜋
− tan−1 𝑠
]
2 𝑎
∴ 𝐿 [𝑠𝑖𝑛𝑎𝑡
] = 𝑐𝑜t−1 𝑠
𝑡 𝑎
𝒕
𝟑
Example: 5.18 Find 𝑳 [𝒔𝒊𝒏 𝒕
]
Solution:
𝑡→0 𝑡→0
𝒔𝒊𝒏𝟑𝒕
= 𝟑𝒔𝒊𝒏𝒕−𝒔𝒊𝒏𝟑𝒕
𝒕 𝟒𝒕
𝒔𝒊𝒏𝟑𝒕 𝟑𝒔𝒊𝒏𝒕−𝒔𝒊𝒏𝟑𝒕
lim = lim
𝒕 𝟒𝒕
= 0−0
= 0
0 0
(by applying L−Hospital rule)
𝑡→0 4𝑡
= lim 3𝑠𝑖𝑛𝑡−𝑠𝑖𝑛3𝑡
= 0
𝑡
Hence Laplace transform exists
3
𝐿 [𝑠𝑖𝑛 𝑡
] = 𝐿 [3𝑠𝑖𝑛𝑡 −𝑠𝑖𝑛3𝑡
]
= 1
4𝑡
∞
𝐿[(3𝑠𝑖𝑛𝑡 − 𝑠𝑖𝑛3𝑡)]𝑑𝑠
∫
4 𝑠
𝑠2+1 𝑠2+9
= 1
∫
∞
(3 1
− 3
) 𝑑𝑠
4 𝑠
1
[ −1 −1 𝑠
= 3tan 𝑠 − tan ]
4 3 𝑠
∞
= 1
[3(tan−1 ∞ − tan−1 𝑠) − (tan−1 ∞ − tan−1 𝑠
)]
4 3
= 1
[(𝜋
− tan−1 𝑠) − (𝜋
− tan−1 𝑠
) ]
4 2 2 3
= 1
[cot−1 𝑠 − 𝑐𝑜t−1 𝑠
]
4 3
𝒕
Example: 5.19 Find 𝑳 [𝒆−𝟐𝒕 𝒔𝒊𝒏𝟐𝒕𝒄𝒐𝒔𝟑𝒕
]
𝒕 𝒕
Solution:
𝐿 [𝑒−2𝑡 𝒔𝒊𝒏𝟐𝒕𝒄𝒐𝒔𝟑𝒕
] = 𝐿 [𝒔𝒊𝒏𝟐𝒕𝒄𝒐𝒔𝟑𝒕
]
𝑠→𝑠+2
= 1
[∫
∞
𝐿(sin(3𝑡 + 2𝑡) − sin(3𝑡 − 2𝑡))𝑑𝑠]
2 𝑠 𝑠→𝑠+2
= 1
[∫
∞
𝐿((sin 5𝑡) − 𝐿(sin 𝑡))𝑑𝑠]
2 𝑠 𝑠→𝑠+2
Laplace Transform
Page
19
−
2 𝑠2+52 𝑠2+12
1
∞
[ 5
= 1
[ ∫
𝑠
]]
𝑠→𝑠+2
2 5
−1 𝑠 −1
𝑠
∞
= 1
[[tan −tan 𝑠] ]
𝑠→𝑠+2
= 1
[[(tan−1 ∞ − tan−1 𝑠
) − (tan−1 ∞ − tan−1 𝑠)] ]
2 5 𝑠→𝑠+2
2 2 5 2
= 1
[(𝜋
− tan−1 𝑠
) − (𝜋
− tan−1 𝑠) ]
𝑠→𝑠+2
= 1
[cot−1 𝑠
− 𝑐𝑜t−1 𝑠 ]
2 5 𝑠→𝑠+2
= 1
[cot−1 (𝑠+2)
− 𝑐𝑜t−1(𝑠 + 2)]
2 5
−𝒂𝒕 −𝒃𝒕
Example: 5.20 Find the Laplace transform for 𝒆 −𝒆
𝒕
Solution:
𝑡→0 𝑡 𝑡→0 0 0 0
−𝑎𝑡 −𝑏𝑡 0 0
lim𝑒 −𝑒
= lim 𝑒 −𝑒
= 1−1
= 0
(use L− Hospital rule)
𝑡→0
−𝑎𝑡 −𝑏𝑡
= lim −𝑎𝑒 +𝑏𝑒
1
= −𝑎 + 𝑏 = 𝑏 − 𝑎 = a finite quantity
Hence Laplace transform exists.
𝑡
−𝑎𝑡 −𝑏𝑡
𝐿 [𝑒 −𝑒
] =
∞ −𝑎𝑡 −𝑏𝑡
∫𝑠 𝐿[𝑒 − 𝑒 ]𝑑𝑠
∞ −𝑎𝑡 −𝑏𝑡
= ∫𝑠
[𝐿(𝑒 ) − 𝐿(𝑒 )]𝑑𝑠
𝑠+𝑎 𝑠+𝑏
∞ 1 1
= ∫
𝑠 ( − ) 𝑑𝑠
𝑠
= [log(𝑠 + 𝑎) − log(𝑠 + 𝑏)]∞
= [𝑙𝑜𝑔 ]
𝑠+𝑏 𝑠
𝑠+𝑎 ∞
= [𝑙𝑜𝑔
𝑎
𝑠
𝑠(1+ )
𝑏 ]
𝑠(1+𝑠
)
𝑠
∞
= 𝑙𝑜𝑔1 − 𝑙𝑜𝑔 𝑠+𝑎
= 0 − 𝑙𝑜𝑔 𝑠+𝑎
𝑠+𝑏 𝑠+𝑏
∵ 𝑙𝑜𝑔1 = 0
= 𝑙𝑜𝑔 𝑠+𝑎
𝑠+𝑏
Example: 5.21 Find the Laplace transform of
Solution:
𝟏−𝒄𝒐𝒔𝒕
𝒕
𝑡→0
lim 𝟏−𝒄𝒐𝒔𝒕
= 0
𝒕 0 𝑡→0 𝟏 1
lim 𝒔𝒊𝒏𝒕
= 0
= 0 (use L− Hospital rule)
𝑡
𝐿 [1−𝑐𝑜𝑠𝑡
] exists.
𝐿 [1−𝑐𝑜𝑠𝑡
] =
𝑡
∞
∫𝑠 𝐿[(1 − 𝑐𝑜𝑠𝑡)]𝑑𝑠
𝑠
𝑠 𝑠2+1
∞ 1
( − ) 𝑑𝑠
= ∫𝑠
Engineering Maths - IV
Laplace Transform
Page
20
1
2
2
= [log𝑠 − log(𝑠 + 1)]
𝑠
∞
𝑠
∞
= [log𝑠 − log√𝑠2 + 1]
𝑠
= [𝑙𝑜𝑔 ]
√𝑠2+1 𝑠
∞
= 0 − 𝑙𝑜𝑔
𝑠
√𝑠2+1
= 𝑙𝑜𝑔 √𝑠2+1
𝑠
Example: 5.22 Find the Laplace transform for 𝒄𝒐𝒔𝒂𝒕−𝒄𝒐𝒔𝒃𝒕
𝒕
Solution:
𝑡→0
lim 𝑐𝑜𝑠𝑎𝑡−𝑐𝑜𝑠𝑏𝑡
= 1−1
=
0 𝑡 0 0
(use L− Hospital rule)
𝑡→0 1
= lim −𝑎𝑠𝑖𝑛𝑎𝑡+𝑏𝑠𝑖𝑛𝑏𝑡
= 0 = a finite quantity
Hence Laplace transform exists.
𝑡
𝐿 [𝑐𝑜𝑠𝑎𝑡−𝑐𝑜𝑠𝑏𝑡
]
=
∞
∫𝑠 𝐿[𝑐𝑜𝑠𝑎𝑡 − 𝑐𝑜𝑠𝑏𝑡]𝑑𝑠
∞
[𝐿(𝑐𝑜𝑠𝑎𝑡) − 𝐿(𝑐𝑜𝑠𝑏𝑡)]𝑑𝑠
= ∫𝑠
( −
𝑠2+𝑎2 𝑠2+𝑏2
𝑠
) 𝑑𝑠
∞ 𝑠
= ∫𝑠
[1
2
( 2 2) 1
2
2 2
= log 𝑠 + 𝑎 − log(𝑠 + 𝑏 )]
𝑠
∞
= [𝑙𝑜𝑔 ]
2 𝑠2+𝑏2
𝑠
1 𝑠2+𝑎2 ∞
2
2 𝑎2
𝑠2
𝑠2(1+𝑏2
)
𝑠 (1+ )
= 1
[𝑙𝑜𝑔 𝑠2
]
𝑠
∞
2
𝑎2
𝑏2
(1+ )
= 1
[𝑙𝑜𝑔 𝑠2
]
(1+ )
𝑠2
𝑠
∞
2 2 2 2
= 1
[𝑙𝑜𝑔1 − 𝑙𝑜𝑔 𝑠 +𝑎
] = − 1
[𝑙𝑜𝑔 𝑠 +𝑎
]
2 𝑠2+𝑏2 2 𝑠2+𝑏2
[∵ 𝑙𝑜𝑔1 = 0]
2 2
= 1
[𝑙𝑜𝑔 𝑠 +𝑏
]
2 𝑠2+𝑎2
Example: 5.23 Find the Laplace transform of
Solution:
𝒔𝒊𝒏𝟐𝒕
𝒕
𝑡→0
𝑠𝑖𝑛2𝑡
= 1−𝑐𝑜𝑠2𝑡
𝑡 2𝑡
lim 𝟏−𝒄𝒐𝒔𝟐𝒕
= 0
𝟐𝒕 0
𝑡→0 𝟐 1
lim 𝟐𝒔𝒊𝒏𝟐𝒕
= 0
= 0 (use L− Hospital rule)
Laplace transform exists.
Engineering Maths - IV
Laplace Transform
Page
21
𝑡 2𝑡
2
𝐿 [𝑠𝑖𝑛 𝑡
] = 𝐿 [1−𝑐𝑜𝑠2𝑡
] = 1 ∞
∫ 𝐿[(1 − 𝑐𝑜𝑠2𝑡)]𝑑𝑠
2 𝑠
= 1 ∞
[𝐿(1) − 𝐿(𝑐𝑜𝑠2𝑡)]𝑑𝑠
∫
2 𝑠
𝑠 𝑠2+4
= 1 ∞
(1
− 𝑠
) 𝑑𝑠
∫
2 𝑠
1
[ 1
2 2
2
= log𝑠 − log(𝑠 + 4)]
𝑠
∞
2 𝑠
= 1
[log𝑠 − log√𝑠2 + 4]
∞
2
1
[
= 𝑙𝑜𝑔
𝑠
2
√𝑠 +4
∞
]
𝑠
2
= 1
[0 − 𝑙𝑜𝑔
𝑠
√𝑠2+4
]
= 1
𝑙𝑜𝑔 √𝑠2+4
2 𝑠
Example: 5.24 Find the Laplace transform for 𝒔𝒊𝒏𝟐𝒕𝒔𝒊𝒏𝟓𝒕
𝒕
Solution:
𝑡
𝐿 [𝑠𝑖𝑛2𝑡𝑠𝑖𝑛5𝑡
] =
∞
𝐿[𝑠𝑖𝑛2𝑡𝑠𝑖𝑛5𝑡]𝑑𝑠
∫𝑠
2
∞ 1
[𝐿(cos(−3𝑡) − 𝐿(𝑐𝑜𝑠7𝑡)]𝑑𝑠
= ∫𝑠
= 1 ∞
∫ [𝐿(cos(3𝑡) − 𝐿(𝑐𝑜𝑠7𝑡)]𝑑𝑠
2 𝑠
[∵ cos(−𝜃) = 𝑐𝑜𝑠𝜃]
𝑠
𝑠2+9 𝑠2+49
( − ) 𝑑𝑠
= 1 ∞ 𝑠
∫
2 𝑠
1
[1
2 2
( 2 )
1
2
2
= log 𝑠 + 9 − log(𝑠 + 49)]
𝑠
∞
= [𝑙𝑜𝑔 ]
4 𝑠2+49 𝑠
1 𝑠2+9 ∞
1
4
= [𝑙𝑜𝑔
2 9
𝑠2
𝑠 (1+ )
2 49
𝑠2
𝑠 (1+ )
]
𝑠
∞
9
𝑠2
4 (1+49
)
= 1
[𝑙𝑜𝑔
(1+𝑠2)
]
𝑠
∞
2 2
= 1
[𝑙𝑜𝑔1 − 𝑙𝑜𝑔 𝑠 +9
] = − 1
[𝑙𝑜𝑔 𝑠 +9
]
4 𝑠2+49 4 𝑠2+49
[∵ 𝑙𝑜𝑔1 = 0]
2
= 1
[𝑙𝑜𝑔 𝑠 +49
]
4 𝑠2+9
Problems using 𝑳 [
𝒕 1
𝑠
∫𝟎 𝒇(𝒕)𝒅𝒕] = 𝑳[𝒇(𝒕)]
Example: 5.25 Find the Laplace transform for (i)
𝒕 −𝟐𝒕
∫𝟎 𝒆 𝒅𝒕 (ii)
𝒕
∫𝟎𝒄𝒐𝒔𝟐𝒕𝒅𝒕
(iii) ∫𝟎
𝒕
𝒕𝒔𝒊𝒏𝟑𝒕𝒅𝒕 (iv) 𝒕
𝒕
∫
𝟎 𝒄𝒐𝒔𝒕𝒅𝒕
Solution:
(i) 𝐿 [∫
𝑡
𝑒−2𝑡𝑑𝑡] = 1
𝐿[𝑒−2𝑡] = 1
( 1
)
0 𝑠 𝑠 𝑠+2
Engineering Maths - IV
Laplace Transform
Page
22
∴ 𝐿 [∫
0
𝑡
𝑒−2𝑡𝑑𝑡] =
1
𝑠(𝑠+2)
(ii) 𝐿 [∫
𝑡
𝑐𝑜𝑠2𝑡𝑑𝑡] = 1
𝐿[𝑐𝑜𝑠2𝑡] = 1
(
0 𝑠 𝑠2+4
𝑠
)
∴ 𝐿 [∫
0
𝑠
𝑡
𝑐𝑜𝑠2𝑡𝑑𝑡] =
1
𝑠2+4
(iii) 𝐿 [
𝑠
𝑡 1
∫0 𝑡𝑠𝑖𝑛3𝑡𝑑𝑡] = 𝐿[𝑡𝑠𝑖𝑛3𝑡]
𝑠 𝑑𝑠
= 1
[−𝑑
[𝐿[𝑠𝑖𝑛3𝑡]]]
= −1
[ 𝑑
[ 3
𝑠 𝑑𝑠 𝑠2+9
]]
= −1
[ −6𝑠
𝑠 (𝑠2+9)2
]
𝑡
∴ 𝐿 [∫
0 𝑡𝑠𝑖𝑛3𝑡𝑑𝑡] =
6
(𝑠2+9)2
(iv) 𝐿 [𝑡
𝑑𝑠
𝑡 −𝑑 𝑡
∫0 𝑐𝑜𝑠𝑡𝑑𝑡] = 𝐿 [∫0𝑐𝑜𝑠𝑡𝑑𝑡]
= −𝑑
[1
( 𝑠
𝑑𝑠 𝑠 𝑠2+1
)]
1
𝑑𝑠 𝑠2+1
= − 𝑑
[ ]
−2𝑠
= − [ ]
∴ 𝐿 [∫
0
𝑡
𝑡𝑠𝑖𝑛3𝑡𝑑𝑡] =
(𝑠2+1)2
2𝑠
(𝑠2+1)2
𝒕
𝒕𝒄𝒐𝒔𝟒𝒕𝒅𝒕
Example: 5.26 Find the Laplace transform for 𝒆−𝒕 ∫
𝟎
Solution:
𝐿 [𝑒−𝑡 ∫
0
𝑡
𝑡𝑐𝑜𝑠4𝑡𝑑𝑡] = 𝐿 [
𝑠 𝑑𝑠
𝑠→𝑠+1 𝑠→𝑠+1
= − (1 𝑑
𝑡 −1 𝑑
∫0 𝑡𝑐𝑜𝑠4𝑡𝑑𝑡] = [ 𝐿(𝑐𝑜𝑠4𝑡)]
𝑠
)
𝑠 𝑑𝑠 𝑠2+16 𝑠→𝑠+1
𝑠 (𝑠2+16)2
2
= [−1 (𝑠 +16)1−𝑠(2𝑠)
]
𝑠→𝑠+1
𝑠 (𝑠2+16)2
2 2
= [−1 (𝑠 +16−2𝑠 )
]
𝑠→𝑠+1
2
= [−1 (−𝑠 +16)
]
𝑠 (𝑠2+16)2
𝑠→𝑠+1
2
= [1 (𝑠 −16)
]
𝑠 (𝑠2+16)2
𝑠→𝑠+1
∴ 𝐿 [𝑒−𝑡 ∫
0
𝑡
𝑡𝑐𝑜𝑠4𝑡𝑑𝑡] =
𝑠+1 ((𝑠+1)2+16 )2
2
1
[ (𝑠+1) −16
]
𝒕
𝒕 𝒔𝒊𝒏𝒕
𝒅𝒕
Example: 5.27 Find the Laplace transform of 𝒆−𝒕 ∫
𝟎
Solution:
𝐿 [𝑒−𝑡 ∫
0 𝑡 𝑡
∫0
𝑡 𝑠𝑖𝑛𝑡
𝑑𝑡] = 𝐿 [ 𝑡 𝑠𝑖𝑛𝑡
𝑑𝑡]
𝑠→𝑠+1
Engineering Maths - IV
Laplace Transform
Page
23
Engineering Mathematics - II
𝑠 𝑡
= [1
𝐿 (𝑠𝑖𝑛𝑡
)]
𝑠→𝑠+1
= [1 ∫
𝑠 𝑠
∞
𝐿(𝑠𝑖𝑛𝑡)𝑑𝑠]
𝑠→𝑠+1
∞
1
= [1
∫
𝑠 𝑠
]
𝑠2+1 𝑠→𝑠+1
𝑠 𝑠
= [1
[tan−1 𝑠 ]∞ ]
𝑠→𝑠+1
𝑠
= [1
(tan−1 ∞ − tan−1 𝑠) ]
𝑠→𝑠+1
𝑠 2
= [1
(𝜋
− tan−1 𝑠) ]
𝑠→𝑠+1
𝑠
= [1
cot−1 𝑠 ]
𝑠→𝑠+1
𝑡
∴ 𝐿 [𝑒−𝑡 ∫
0
𝑡 𝑠𝑖𝑛𝑡
𝑑𝑡] = 1
𝑠+1
𝑐𝑜t−1(𝑠 + 1)
Exercise: 5.3
Find the Laplace transform of
1. 𝑠𝑖𝑛
𝑡 𝑡
Ans: cot−1 𝑠
2
2. 𝑒−2𝑡 𝑠𝑖𝑛𝑡
𝑡
Ans: 𝑐𝑜t−1(𝑠 + 2)
3. 𝑠𝑖𝑛𝑎𝑡−𝑠𝑖𝑛𝑏𝑡
𝑡
Ans:
−𝑎𝑡
4. 𝑒 −𝑐𝑜𝑠𝑏𝑡
𝑡
Ans:
cot−1 𝑠
− cot−1 𝑠
𝑎 𝑏
𝑙𝑜𝑔 √𝑠2+𝑏2
𝑠+𝑎
−𝑡
5. 1−𝑒
𝑡
Ans: 𝑙𝑜𝑔 s+1
𝑠
𝑡
𝑡 𝑠𝑖𝑛𝑡
𝑑𝑡
6. 𝑒−𝑡 ∫
0
Ans:
1
𝑠+1
𝑐𝑜t−1(𝑠 + 1)
7. 𝑒−𝑡 ∫
𝑡
𝑡𝑐𝑜𝑠𝑡𝑑𝑡
0
Ans: [
1 𝑠2+2𝑠
𝑠+1 (𝑠2+2𝑠+2 )2
]
𝑡
𝑡𝑒−𝑡𝑠𝑖𝑛𝑡𝑑𝑡
8. 𝑒−𝑡 ∫
0 𝑠 𝑠2+2𝑠+2
Ans: 1
[ 2(𝑠+1)
]
∞
𝑓(𝑡)𝑒−𝑠𝑡𝑑𝑡 = 𝐿[𝑓(𝑡)]
Evaluation of integrals using Laplace transform
Note: (i) ∫
0
(ii) ∫0
∞
𝑓(𝑡)𝑒−𝑎𝑡 𝑑𝑡 = [𝐿[𝑓(𝑡)]]
𝑠=𝑎
(iii) ∫0
∞
𝑓(𝑡)𝑑𝑡 = [𝐿[𝑓(𝑡)]]
𝑠=0
𝒔𝟐+𝟒
Example: 5.28 If 𝑳[𝒇(𝒕)] = 𝒔+𝟐
, then find the value of
∞
∫𝟎 𝒇(𝒕)𝒅𝒕
Solution:
Given 𝐿[𝑓(𝑡)] = 𝑠+2
𝑠2+4
We know that ∫0
∞
𝑓(𝑡)𝑑𝑡 = [𝐿[𝑓(𝑡)]]
𝑠=0
𝑠+2
= [ ] = 2
𝑠2+4 𝑠=0 4
Laplace Transform
Page
24
2
∞ 1
∫0 𝑓(𝑡)𝑑𝑡 =
𝒔𝟐−𝟗
∞
𝒆−𝟐𝒕𝒇(𝒕)𝒅𝒕
Example: 5.29 If 𝑳[𝒇(𝒕)] = 𝟓𝒔+𝟒
, then find the value of ∫
𝟎
Solution:
Given 𝐿[𝑓(𝑡)] = 5𝑠+4
𝑠2−9
We know that ∫0
∞
𝑒−2𝑡 𝑓(𝑡)𝑑𝑡 = [𝐿[𝑓(𝑡)]]
𝑠=2
= [5𝑠+4
] = 14
𝑠2−9 𝑠=2 −5
5
∞ −14
𝑒−2𝑡𝑓(𝑡)𝑑𝑡 =
∴ ∫0
Example: 5.30 Find the values of the following integrals using Laplace transforms:
(i) ∫𝟎
∞
𝒕𝒆−𝟐𝒕𝒄𝒐𝒔𝟐𝒕𝒅𝒕 (ii) ∫𝟎
∞
𝒕𝟐𝒆−𝒕𝒔𝒊𝒏𝒕𝒅𝒕 (iii)
𝒕
∞ 𝒆−𝒕−𝒆−𝟐𝒕
( ) 𝒅𝒕
∫𝟎
𝒕
(iv) ∫𝟎
∞ 𝟏−𝒄𝒐𝒔𝒕
( ) 𝒆−𝒕𝒅𝒕 (v)
𝒕
∞ 𝒆−𝒂𝒕−𝒄𝒐𝒔𝒃𝒕
∫
𝟎 ( ) 𝒅𝒕
Solution:
(i) ∫
∞
𝑡𝑒−2𝑡𝑐𝑜𝑠2𝑡𝑑𝑡 = 𝐿[𝑡𝑐𝑜𝑠2𝑡]
0 𝑠=2 𝑑𝑠
= [−𝑑
𝐿(𝑐𝑜𝑠2𝑡)]
𝑠=2
= −𝑑
( 𝑠
)
𝑑𝑠 𝑠2+4 𝑠=2
(𝑠2+4)2
2
= − [ (𝑠 +4)1−𝑠(2𝑠)
]
𝑠=2
2
= − [ (4−𝑠 )
]
(𝑠2+4)2
𝑠=2
= − (4−4)
= 0
(4+4)2
(ii) 𝑠=1 𝑑𝑠2
∞ 2 −𝑡 2 𝑑2
∫0 𝑡 𝑒 𝑠𝑖𝑛𝑡𝑑𝑡 = 𝐿[𝑡 𝑠𝑖𝑛𝑡] = 𝐿[𝑠𝑖𝑛𝑡]𝑠=1
1
2
= 𝑑
( )
𝑑𝑠2 𝑠2+1 𝑠=1
= 𝑑
[ −1(2𝑠)
]
𝑑𝑠 (𝑠2+1)2
𝑠=1
= −2 𝑑
[
𝑠
]
𝑑𝑠 (𝑠2+1)2
𝑠=1
2
[(𝑠 +1)
= −2 [
2
2
(1)−𝑠.2(𝑠 +1)(2𝑠))]
(𝑠2+1)4
]
𝑠=1
= −2 [
[(𝑠2+1)[(𝑠2+1)−4𝑠2)]]
(𝑠2+1)4
]
𝑠=1
(𝑠2+1)3
2
= −2 [(1−3𝑠 )
]
𝑠=1
(𝑠2+1)3
3
= [ 6𝑠 −2
]
𝑠=1
= 4
= 1
(iii) ∫0 𝑡 𝑡
∞ 𝑒−𝑡−𝑒−2𝑡 −𝑡 −2𝑡
( ) 𝑑𝑡 = 𝐿 [𝑒 −𝑒
]
𝑠=0
𝑠=0
8 2
= ∫
∞
[𝐿[𝑒−𝑡 − 𝑒−2𝑡]𝑑𝑠]
𝑠
Engineering Maths - IV
Laplace Transform
Page
25
𝑠=0
∞
= ∫ [[𝐿(𝑒−𝑡) − 𝐿(𝑒−2𝑡)]𝑑𝑠]
𝑠
𝑠+1 𝑠+2
− 1
) 𝑑𝑠]
𝑠=0
∞
[( 1
= ∫𝑠
= {[log(𝑠 + 1) − log(𝑠 + 2)]∞}
𝑠 𝑠
=0
𝑠+2 𝑠
𝑠+1 ∞
= {[𝑙𝑜𝑔 ] }
𝑠=0
= {𝑙𝑜𝑔 𝑠
𝑠(1+ )
2
𝑠(1+𝑠
)
𝑠
1 ∞
}
𝑠=0
= [0 − 𝑙𝑜𝑔 𝑠+1
]
𝑠+2 𝑠=0
∵ 𝑙𝑜𝑔1 = 0
= [𝑙𝑜𝑔 𝑠+2
] = 𝑙𝑜𝑔2
𝑠+1 𝑠=0
(iv)
𝑡
∞ 1−𝑐𝑜𝑠𝑡 −𝑡
∫0 ( ) 𝑒 𝑑𝑡
∫0 𝑡 𝑡
∞ 1−𝑐𝑜𝑠𝑡 1−𝑐𝑜𝑠𝑡
( ) 𝑒−𝑡𝑑𝑡 = 𝐿 [ ]
𝑆=1
𝑆=1
∞
[𝐿[(1 − 𝑐𝑜𝑠𝑡)]𝑑𝑠]
= ∫𝑠
𝑆=1
∞
[[𝐿(1) − 𝐿(𝑐𝑜𝑠𝑡)]𝑑𝑠]
= ∫𝑠
𝑠 𝑠2+1 𝑆=1
∞ 1 𝑠
= ∫
𝑠 [( − ) 𝑑𝑠]
2 𝑠
∞
= {[log𝑠 − 1
log(𝑠2 + 1)] }
𝑆=1
𝑠
∞
= {[log𝑠 − log√𝑠2 + 1] }
𝑆=1
= {[𝑙𝑜𝑔
𝑠
√𝑠2+1 𝑠
∞
] }
𝑆=1
= [0 − 𝑙𝑜𝑔
𝑠
]
√𝑠2+1 𝑠=1
𝑠
= [𝑙𝑜𝑔 √𝑠2+1
]
𝑠=1
= 𝑙𝑜𝑔√2
𝑡
−𝑎𝑡
(v)
∞
(𝑒 −𝑐𝑜𝑠𝑏𝑡
) 𝑑𝑡
∫0
𝑡 𝑡
∞ 𝑒−𝑎𝑡−𝑐𝑜𝑠𝑏𝑡 𝑒−𝑎𝑡−𝑐𝑜𝑠𝑏𝑡
∫0 ( ) 𝑑𝑡 = 𝐿 [ ]
𝑆=0
𝑆=0
∞
[𝐿[(𝑒−𝑎𝑡 − 𝑐𝑜𝑠𝑏𝑡)]𝑑𝑠]
= ∫𝑠
𝑆=0
∞
[[𝐿(𝑒−𝑎𝑡) − 𝐿(𝑐𝑜𝑠𝑏𝑡)]𝑑𝑠]
= ∫𝑠
−
𝑠+𝑎 𝑠2+𝑏2
𝑠
) 𝑑𝑠]
𝑆=0
∞
[( 1
= ∫𝑠
1
2
2 2
= {[log(𝑠 + 𝑎) − log(𝑠 + 𝑏 )]
𝑠
∞
}
𝑆=0
2
𝑠
2
∞
= {[log(𝑠 + 𝑎) − log√𝑠 + 𝑏 ] }
𝑆=0
𝑠+𝑎
𝑠
∞
= {[𝑙𝑜𝑔 ] }
√𝑠2+𝑏2
𝑆=0
Engineering Maths - IV
Laplace Transform
Page
26
= [0 − 𝑙𝑜𝑔
𝑠+𝑎
]
√𝑠2+𝑏2
𝑠=0
𝑠+𝑎
= [𝑙𝑜𝑔 √𝑠2+𝑏2
]
𝑠=0
√𝑏2
= 𝑙𝑜𝑔
𝑎
= 𝑙𝑜𝑔 𝑏
𝑎
Exercise: 5.4
Find the values of the following integrals using Laplace transforms
1. ∫0
∞
𝑡𝑒−2𝑡𝑐𝑜𝑠𝑡𝑑𝑡 Ans:
3
25
13
250
2. ∫0
∞
𝑡𝑒−3𝑡𝑠𝑖𝑛𝑡𝑑𝑡 Ans:
𝑡
3. ∫0
∞ 𝑒−𝑎𝑡−𝑒−𝑏𝑡
( ) 𝑑𝑡 Ans: 𝑙𝑜𝑔 𝑏
𝑎
𝑡
4. ∫0
∞ sin2 𝑡
𝑒−2𝑡 𝑑𝑡 Ans:
4
1
𝑙𝑜𝑔2
5. ∫0
∞ 𝑐𝑜𝑠𝑎𝑡−𝑐𝑜𝑠𝑏𝑡
( ) 𝑑𝑡 Ans: 𝑙𝑜𝑔 𝑎
𝑡 𝑏
∞
𝒇(𝒕)𝒆−𝒔𝒕𝒅𝒕 = 𝑳[𝒇(𝒕)]
Laplace transform of Piecewise continuous functions
∫𝟎
Example: 5.31 Find the Laplace transform of 𝒇(𝒕) = {
𝒆−𝒕; 𝟎 < 𝑡 < 𝜋
𝟎 ; 𝒕 > 𝜋
Solution:
∞
𝑓(𝑡)𝑒−𝑠𝑡𝑑𝑡
𝐿[𝑓(𝑡)] = ∫0
= ∫0
𝜋
𝑒−𝑠𝑡𝑒−𝑡𝑑𝑡 +
∞
𝑒−𝑠𝑡 0𝑑𝑡
∫𝜋
𝜋
𝑒−(𝑠+1)𝑡𝑑𝑡
= ∫0
= [ ] =
𝑒−(𝑠+1)𝑡 𝜋 𝑒−(𝑠+1)𝜋−𝑒0
−(𝑠+1) 0 −(𝑠+1)
−(𝑠+1)𝜋
∴ 𝐿[𝑓(𝑡)] = 1−𝑒
−(𝑠+1)
Example: 5.32 Find the Laplace transform of 𝒇(𝒕) = {
𝒔𝒊𝒏𝒕; 𝟎 < 𝑡 < 𝜋
𝟎 ; 𝒕 > 𝜋
Solution:
𝐿[𝑓(𝑡)] =
∞ −𝑠𝑡
∫0 𝑓(𝑡)𝑒 𝑑𝑡
= ∫𝟎
∞
𝑒−𝑠𝑡0𝑑𝑡
𝝅
𝑒−𝑠𝑡𝑠𝑖𝑛𝑡𝑑𝑡 + ∫
𝜋
𝜋
𝑒−𝑠𝑡𝑠𝑖𝑛𝑡𝑑𝑡
= ∫0
𝒆−𝒔𝒕
𝟎
𝝅 𝑒−𝑠𝜋
[
= [ (−𝑠𝑠𝑖𝑛𝑡 − 𝑐𝑜𝑠𝑡)] = −𝑠𝑠𝑖𝑛𝜋 − 𝑐𝑜𝑠𝜋] −
(−𝒔)𝟐+𝟏 𝑠2+1
𝑒0
2
𝑠 +1
[−𝑠𝑠𝑖𝑛0 − 𝑐𝑜𝑠0]
1
−𝑠𝜋 −𝑠𝜋
= 𝑒
(0 + 1) − (−1) = 𝑒 +1
𝑠2+1 𝑠2+1 𝑠2+1
Engineering Maths - IV
Laplace Transform
Page
27
−𝒔𝝅
∴ 𝐿[𝑓(𝑡)] = 𝒆 +𝟏
𝒔𝟐+𝟏
( )
Example: 5.33 Find the Laplace transform of 𝒇 𝒕 = {
𝒕;𝟎 < 𝑡 < 1
𝟎 ; 𝒕 > 1
Solution:
∞
𝑓(𝑡)𝑒−𝑠𝑡𝑑𝑡
𝐿[𝑓(𝑡)] = ∫0
1 −𝑠𝑡
= ∫0 𝑒 𝑡𝑑𝑡 +
∞ −𝑠𝑡
∫1 𝑒 0𝑑𝑡
1
𝑡𝑒−𝑠𝑡𝑑𝑡
= ∫0
0
𝑒−𝑠𝑡 𝑒−𝑠𝑡 1 𝒆−𝒔 𝒆−𝒔 𝟏
= [𝑡 − (1) ] = − − 0 +
−𝑠 (−𝑠)2 −𝒔 𝒔𝟐 𝒔𝟐
𝒔
−𝒔 −𝒔
∴ 𝐿[𝑓(𝑡)] = − 𝒆
− 𝒆
+ 𝟏
𝒔𝟐 𝒔𝟐
Exercise: 5.5
( )
1. Find the Laplace transform of 𝑓 𝑡 = {
0; 0 < 𝑡 < 2
3 ; 𝑡 > 2
Ans:
𝟑𝒆−𝟐𝒔
𝒔
( )
2. Find the Laplace transform of 𝑓 𝑡 = {
𝑒𝑡 ; 0 < 𝑡 < 1
0 ; 𝑡 > 1
Ans:
𝟏−𝒆−(𝒔−𝟏)
𝒔−𝟏
( )
3. Find the Laplace transform of 𝑓 𝑡 = {
1; 0 < 𝑡 < 1
0 ; 𝑡 > 1
Ans:
𝟏−𝒆−𝒔
𝒔
Unit step function
The unit step function 𝑈(𝑡 − 𝑎) is defined as 𝑈(𝑡 − 𝑎) = {0; 𝑡 < 𝑎
0 ; 𝑡 > 𝑎
Example: 5.34 Find the Laplace transform of unit step functions.
Solution:
∞
𝑈(𝑡 − 𝑎)𝑒−𝑠𝑡𝑑𝑡
𝐿[𝑈(𝑡 − 𝑎)] = ∫0
∞
𝑒−𝑠𝑡𝑑𝑡
∞
(1)𝑒−𝑠𝑡𝑑𝑡 = ∫
𝑎
𝑎
= ∫00𝑑𝑡 + ∫𝑎
= [
−𝑠
𝑒−𝑠𝑡 ∞
] = 0 −
𝑎
𝑒−𝑠𝑎
−𝑠
=
𝑒−𝑠𝑎
𝑠
−𝑠𝑎
𝐿[𝑈(𝑡 − 𝑎)] = 𝑒
𝑠
Second Shifting theorem
Statement: If 𝐿[𝑓(𝑡)] = 𝐹(𝑠), then 𝐿[𝑓(𝑡 − 𝑎)𝑈(𝑡 − 𝑎)] = 𝑒−𝑎𝑠𝐹(𝑠)
Proof:
𝑈(𝑡 − 𝑎)𝑓(𝑡 − 𝑎) = {
0; 𝑡 < 𝑎
𝑓(𝑡 − 𝑎) ; 𝑡 > 𝑎
By the definition of Laplace transform,
∞
𝑈(𝑡 − 𝑎)𝑓(𝑡 − 𝑎)𝑒−𝑠𝑡𝑑𝑡
𝐿[𝑈(𝑡 − 𝑎)𝑓(𝑡 − 𝑎)] = ∫0
∞
𝑓(𝑡 − 𝑎)𝑒−𝑠𝑡𝑑𝑡
𝑎
= ∫00𝑑𝑡 + ∫𝑎
∞
𝑒−𝑠(𝑎+𝑥)𝑓(𝑥)𝑑𝑥
𝐿[𝑈(𝑡 − 𝑎)𝑓(𝑡 − 𝑎)] = ∫0
Engineering Maths - IV
Laplace Transform
Page
28
∞
= ∫ 𝑒−𝑠𝑎𝑒−𝑠𝑥𝑓(𝑥)𝑑𝑥
0
= 𝑒−𝑠𝑎
∞
𝑒−𝑠𝑥𝑓(𝑥)𝑑𝑥
∫0
∞
𝑒−𝑠𝑡𝑓(𝑡)𝑑𝑡
Replace x by t
𝐿[𝑈(𝑡 − 𝑎)𝑓(𝑡 − 𝑎)] = 𝑒−𝑠𝑎 ∫
0
= 𝑒−𝑠𝑎𝐿[𝑓(𝑡)] = 𝑒−𝑠𝑎𝐹(𝑠)
𝐿[𝑈(𝑡 − 𝑎)𝑓(𝑡 − 𝑎)] = 𝑒−𝑠𝑎𝐹(𝑠)
5.5 PERIODIC FUNCTIONS
Definition: A function 𝑓(𝑡) is said to be periodic if 𝑓(𝑡 + 𝑇) = 𝑓(𝑡) for all values of t and for certain values
of T. The smallest value of T for which 𝑓(𝑡 + 𝑇) = 𝑓(𝑡) for all t is called periodic function.
Example:
𝑠𝑖𝑛𝑡 = sin(𝑡 + 2𝜋) = sin(𝑡 + 4𝜋) ⋯
∴ 𝑠𝑖𝑛𝑡 is periodic function with period 2𝜋.
Let 𝒇(𝒕) be a periodic function with period T. Then
𝟏
𝑳[𝒇(𝒕)] =
𝟏 − 𝒆−𝒔𝑻
𝑻
∫ 𝒆−𝒔𝒕𝒇(𝒕)𝒅𝒕
𝟎
Problems on Laplace transform of Periodic function
Example: 5.35 Find the Laplace transform of 𝒇(𝒕) = {
𝒔𝒊𝒏𝑚𝒕; 𝟎 < 𝑡 < 𝝅
𝑚
𝟎 ; 𝝅
< 𝑡 < 𝟐𝝅
𝑚 𝑚
𝑚
𝒇(𝒕 + 𝟐𝝅
) = 𝒇(𝒕)
Solution:
The given function is a periodic function with period 𝑇 = 2𝜋
𝜔
𝐿[𝑓(𝑡)] =
1 𝑇
𝑒−𝑠𝑡𝑓(𝑡)𝑑𝑡
1−𝑒−𝑠𝑇 ∫0
=
1
−𝟐𝝅𝒔
1−𝑒 𝑚
𝝅
𝑚
0
𝟐𝝅
−𝑠𝑡 𝑚 −𝑠𝑡
𝝅
𝑚
[∫ 𝑠𝑖𝑛𝜔𝑡𝑒 𝑑𝑡 + ∫ 𝑒 (0)𝑑𝑡]
=
1
1−𝑒 𝜔
𝜋
∫𝜔 𝑠𝑖𝑛𝜔𝑡𝑒−𝑠𝑡𝑑𝑡
−2𝜋𝑠 0
=
1
−2𝜋 𝑠
1−𝑒 𝜔
𝑒−𝑠𝑡
[ (−𝑠𝑠𝑖𝑛𝜔𝑡 − 𝜔𝑐𝑜𝑠𝜔𝑡)]
(−𝑠)2+𝜔2
0
𝜋
𝜔
=
1−𝑒 𝜔
−𝑠𝜋
1 𝑒 𝜔
−2𝜋𝑠 { 𝑠2+𝜔2
[−𝑠𝑠𝑖𝑛𝜋 − 𝜔𝑐𝑜𝑠𝜋] +
𝜔
𝑠2+𝜔2
}
=
1−𝑒 𝜔
−𝑠𝜋
𝑠2+𝜔2
1 𝑒 𝜔 𝜔+𝜔
−2𝜋𝑠 [ ]
=
1
12−(𝑒 𝜔 )
−𝜋𝑠 2 [
−𝑠𝜋
𝜔(𝑒 𝜔 +1)
𝑠2+𝜔2
]
Let 𝑡 − 𝑎 = 𝑥 ⋯ (1)
𝑡 = 𝑎 + 𝑥
𝑑𝑡 = 𝑑𝑥
When 𝑡 = 𝑎, (1) => 𝑥 = 0
When 𝑡 = ∞, (1) => 𝑥 = ∞
Engineering Maths - IV
Laplace Transform
Page
29
=
1
−𝜋 𝑠
(1−𝑒 𝜔 )(1+𝑒 𝜔 )
−𝜋𝑠 [
−𝑠𝜋
𝜔(𝑒 𝜔 +1)
𝑠2+𝜔2
]
∴ 𝐿[𝑓(𝑡)] =
𝜔
−𝜋𝑠
(1−𝑒 𝜔 )(𝑠2+𝜔2)
Example: 5.36 Find the Laplace transform of 𝒇(𝒕) = {
𝑬;𝟎 ≤ 𝒕 ≤ 𝒂
−𝑬 ;𝒂 ≤ 𝒕 ≤ 𝟐𝒂
given that 𝒇(𝒕 + 𝟐𝒂) = 𝒇(𝒕).
Solution:
The given function is a periodic function with period 𝑇 = 2𝑎
𝐿[𝑓(𝑡)] =
1 𝑇
𝑒−𝑠𝑡𝑓(𝑡)𝑑𝑡
1−𝑒−𝑠𝑇 ∫0
=
1 2𝑎
𝑒−𝑠𝑡𝑓(𝑡)𝑑𝑡
1−𝑒−2𝑎𝑠 ∫0
=
1
1−𝑒−2𝑎𝑠 ∫0
[
𝑎
𝐸𝑒−𝑠𝑡 𝑑𝑡 + ∫
𝑎
2𝑎
−𝐸𝑒−𝑠𝑡𝑑𝑡]
=
1
1−𝑒−2𝑎𝑠 [𝐸 ∫0
𝑎
𝑒−𝑠𝑡𝑑𝑡 − 𝐸 ∫
𝑎
2𝑎
𝑒−𝑠𝑡𝑑𝑡]
𝐸
= [[ ]
1−𝑒−2𝑎𝑠 −𝑠 0 𝑎
𝑒−𝑠𝑡 𝑎 𝑒−𝑠𝑡 2𝑎
− [ ] ]
−𝑠
=
𝐸
=
1−𝑒−2𝑎𝑠
𝐸
1−𝑒−2𝑎𝑠 𝑠
−𝑎𝑠 −2𝑎𝑠 −𝑎𝑠
[𝑒
+ 1
− 𝑒
− 𝑒
]
−𝑠 𝑠 𝑠 𝑠
−𝑎𝑠 −2𝑎𝑠
[1−2𝑒 +𝑒
]
= 12−(𝑒−𝑎𝑠)2 𝑠
−𝑎𝑠 2
𝐸
[(1−𝑒 )
]
=
𝐸
(1−𝑒−𝑎𝑠)(1+𝑒−𝑎𝑠) 𝑠
−𝑎𝑠 2
[(1−𝑒 )
]
−𝑎𝑠
= 𝐸 (1−𝑒 )
𝑠 (1+𝑒−𝑎𝑠)
∴ 𝐿[𝑓(𝑡)] = 𝐸
tanh (𝑎𝑠
)
𝑠 2
Example: 5.37 Find the Laplace transform of 𝒇(𝒕) = {
𝟏;𝟎 ≤ 𝒕 ≤ 𝒂
𝟐
𝒂
𝟐
−𝟏 ; ≤ 𝒕 ≤ 𝒂
given that 𝒇(𝒕 + 𝒂) = 𝒇(𝒕).
Solution:
The given function is a periodic function with period 𝑇 = 𝑎
𝐿[𝑓(𝑡)] =
1 𝑇
𝑒−𝑠𝑡𝑓(𝑡)𝑑𝑡
1−𝑒−𝑠𝑇 ∫0
=
1 𝑎
𝑒−𝑠𝑡𝑓(𝑡)𝑑𝑡
1−𝑒−𝑎𝑠 ∫0
=
1
1−𝑒−𝑎𝑠
𝑎
0
𝑎
2
[∫2(1)𝑒−𝑠𝑡𝑑𝑡 + ∫𝑎(−1)𝑒−𝑠𝑡𝑑𝑡]
=
1
1−𝑒−𝑎𝑠
𝑎
0
𝑎
2
[∫2 𝑒−𝑠𝑡𝑑𝑡 − ∫𝑎 𝑒−𝑠𝑡𝑑𝑡]
=
1
1−𝑒−𝑎𝑠
0
𝑎
−𝑠𝑡
−𝑠 −𝑠 𝑎
2
−𝑠𝑡 2𝑎
[[𝑒
]
2
− [𝑒
] ]
Engineering Maths - IV
Laplace Transform
Page
30
= 1−𝑒−𝑎𝑠
−𝑠𝑎 −𝑠𝑎
1
[𝑒 2
+ 1
+ 𝑒−𝑎𝑠
− 𝑒 2
]
−𝑠 𝑠 𝑠 𝑠
= 1−𝑒−𝑎𝑠
−𝑠𝑎
𝑠
−𝑎𝑠
1
[1−2𝑒 2 +𝑒
]
=
1
12−(𝑒 2 )
−𝑠𝑎 2 [
−𝑠𝑎 2
(1−𝑒 2 )
𝑠
]
=
1
−𝑠𝑎
(1−𝑒 2 )(1+𝑒 2 )
−𝑠𝑎 [
−𝑠𝑎 2
(1−𝑒 2 )
𝑠
]
=
𝑠
−𝑠𝑎
1 (1−𝑒 2 )
−𝑠𝑎
(1+𝑒 2 )
(1−𝑒−2𝑥)
[∵ 𝑡𝑎𝑛ℎ𝑥 = ]
(1+𝑒−2𝑥)
∴ 𝐿[𝑓(𝑡)] = 1
tanh (𝑎𝑠
)
𝑠 4
Example: 5.38 Find the Laplace transform of 𝒇(𝒕) = {
𝒕;𝟎 ≤ 𝒕 ≤ 𝒂
𝟐𝒂 − 𝒕 ;𝒂 ≤ 𝒕 ≤ 𝟐𝒂
given that
𝒇(𝒕 + 𝟐𝒂) = 𝒇(𝒕).
Solution:
The given function is a periodic function with period 𝑇 = 2𝑎
𝐿[𝑓(𝑡)] =
1 𝑇
𝑒−𝑠𝑡𝑓(𝑡)𝑑𝑡
1−𝑒−𝑠𝑇 ∫0
=
1 2𝑎
𝑒−𝑠𝑡𝑓(𝑡)𝑑𝑡
1−𝑒−2𝑎𝑠 ∫0
=
1
1−𝑒−2𝑎𝑠 ∫0
[
𝑎
𝑡𝑒−𝑠𝑡𝑑𝑡 + ∫
𝑎
2𝑎
(2𝑎 − 𝑡)𝑒−𝑠𝑡𝑑𝑡]
1
𝑎
𝑒−𝑠𝑡 𝑒−𝑠𝑡 𝑒−𝑠𝑡 𝑒−𝑠𝑡
0 𝑎
2𝑎
= [[𝑡 ( ) − ( )] − [(2𝑎 − 𝑡) ( ) − (−1)( )] ]
1−𝑒−2𝑎𝑠 −𝑠 (−𝑠)2 −𝑠 (−𝑠)2
=
𝑠 𝑠
𝑠2 𝑠2 𝑠2 𝑠2
1
[−𝑎𝑒−𝑎𝑠
− 𝑒−𝑎𝑠
+ 1
+ 𝑒−2𝑎𝑠
+ 𝑎𝑒−𝑎𝑠
− 𝑒−𝑎𝑠
]
= 1−𝑒−2𝑎𝑠 𝑠2
1−𝑒−2𝑎𝑠
1
[1−2𝑒−𝑎𝑠+𝑒−2𝑎𝑠
]
= 12−(𝑒−𝑎𝑠)2 𝑠2
−𝑎𝑠 2
1
[(1−𝑒 )
]
=
1
(1−𝑒−𝑎𝑠)(1+𝑒−𝑎𝑠) 𝑠2
−𝑎𝑠 2
[(1−𝑒 )
]
−𝑎𝑠
= 1 (1−𝑒 )
𝑠2 (1+𝑒−𝑎 𝑠)
𝑠2 2
= 1
tanh (𝑎𝑠
)
Exercise: 5.6
1. Find the Laplace transform of
𝑓(𝑡) = {
1; 0 ≤ 𝑡 ≤ 𝑎
2
2
−1 ;𝑎
≤ 𝑡 ≤ 𝑎
given that 𝑓(𝑡 + 𝑎) = 𝑓(𝑡). Ans: 𝑘
tanh (𝑎𝑠
)
𝑠 2
Engineering Maths - IV
Laplace Transform
Page
31
Engineering Maths - IV
2. Find the Laplace transform of
𝑓(𝑡) = {
𝑡; 0 ≤ 𝑡 ≤ 𝑎
2𝑎 − 𝑡 ;𝑎 ≤ 𝑡 ≤ 2𝑎
given that 𝑓(𝑡 + 2𝑎) = 𝑓(𝑡). Ans: 𝑠2 2
1
tanh (𝑠𝜋
)
3. Find the Laplace transform of
𝑎
𝑡
; 0 ≤ 𝑡 ≤ 𝑎
2𝑎−𝑡
;𝑎 ≤ 𝑡 ≤ 2𝑎
𝑓(𝑡) = { given that 𝑓(𝑡 + 2𝑎) = 𝑓(𝑡).
𝑎
Find the Laplace transform of
Ans:
1
tanh (𝑠𝑎
)
𝑎𝑠2 2
4.
𝑓(𝑡)
𝑠𝑖𝑛𝑡;0 < 𝑡 < 𝜋
= { 0 ;𝜋 < 𝑡 < 2𝜋
𝑓(𝑡 + 2𝜋) = 𝑓(𝑡) Ans:
1
(1−𝑒−𝜋𝑠)(𝑠2+1)
5.6 INVERSE LAPLACE TRANSFORM
Definition
If the Laplace transform of a function 𝑓(𝑡) is 𝐹(𝑠)𝑖𝑒. , 𝐿[𝑓(𝑡)] = 𝐹(𝑠),then𝑓(𝑡) is called an
inverse Laplace transform of 𝐹(𝑠) and we write symbolically𝑓(𝑡) = 𝐿−1[𝐹(𝑠)], where 𝐿−1 is called the
inverse Laplace transform operator.
Inverse Laplace transform of elementary functions
𝑳[𝒇(𝒕)] = 𝑭(𝒔) 𝑳−𝟏[𝑭(𝒔)] = 𝒇(𝒕)
𝟏
𝑳[𝟏] =
𝒔
𝟏
𝐿−1 [ ] = 𝟏
𝒔
𝟏
𝑳[𝒕] =
𝒔𝟐
𝟏
𝐿−1 [ ] = 𝒕
𝒔𝟐
𝒏!
𝑳[𝒕𝒏] = 𝒊𝒇 𝒏 𝒊𝒔 𝒂𝒏 𝒊𝒏𝒕𝒆𝒈𝒆𝒓
𝒔𝒏+𝟏
𝒏!
𝐿−1 [ ] = 𝒕𝒏
𝒔𝒏+𝟏
𝟏 𝒕𝒏
𝐿−1 [ ] =
𝒔𝒏+𝟏 𝒏!
𝟏
𝑳[𝒆𝒂𝒕] =
𝒔 − 𝒂
𝟏
𝐿−1 [ ] = 𝒆𝒂𝒕
𝒔 − 𝒂
𝟏
𝑳[𝒆−𝒂𝒕] =
𝒔 + 𝒂
𝟏
𝐿−1 [ ] = 𝒆−𝒂𝒕
𝒔 + 𝒂
𝒂
𝑳[𝒔𝒊𝒏𝒂𝒕] =
𝒔𝟐 + 𝒂𝟐
𝟏 𝒔𝒊𝒏𝒂𝒕
𝐿−1 [ ] =
𝒔𝟐 + 𝒂𝟐 𝒂
𝒔
𝑳[𝒄𝒐𝒔𝒂𝒕] =
𝒔𝟐 + 𝒂𝟐
𝒔
𝐿−1 [ ] = 𝒄𝒐𝒔𝒂𝒕
𝒔𝟐 + 𝒂𝟐
Laplace Transform
Page
32
𝒂
𝑳[𝒔𝒊𝒏𝒉𝒂𝒕] =
𝒔𝟐 − 𝒂𝟐
𝟏 𝒔𝒊𝒏𝒉𝒂𝒕
𝐿−1 [ ] =
𝒔𝟐 − 𝒂𝟐 𝒂
𝒔
𝑳[𝒄𝒐𝒔𝒂𝒕] =
𝒔𝟐 − 𝒂𝟐
𝒔
𝐿−1 [ ] = 𝒄𝒐𝒔𝒉𝒂𝒕
𝒔𝟐 − 𝒂𝟐
Result on inverse Laplace transform
Result: 1 Linear property
𝑳[𝒇(𝒕)] = 𝑭(𝒔) and 𝑳[𝒈(𝒕)] = 𝑮(𝒔) ,then 𝑳−𝟏[𝒂𝑭(𝒔) ± 𝒃𝑮(𝒔)] = 𝒂𝑳−𝟏[𝑭(𝒔)] ± 𝒃𝑳−𝟏[𝑮(𝒔)]
Where a and b are constants.
Proof:
We know that 𝐿[𝑎𝐹(𝑠) ± 𝑏𝐺(𝑠)] = 𝑎𝐿[𝐹(𝑠)] ± 𝑏𝐿[𝐺(𝑠)]
= 𝑎 𝐹(𝑠) ± 𝑏 𝐺(𝑠)
(𝑖.𝑒. )𝑎 𝐹(𝑠) ± 𝑏 𝐺(𝑠) = 𝐿[𝑎𝑓(𝑡) ± 𝑏𝑔(𝑡)]
Operating 𝐿−1 on both sides, we get
𝐿−1[𝑎𝐹(𝑠) ± 𝑏𝐺(𝑠)] = 𝑎𝑓(𝑡) ± 𝑏𝑔(𝑡)
𝐿−1[𝑎𝐹(𝑠) ± 𝑏𝐺(𝑠)] = 𝑎𝐿−1[𝐹(𝑠)] ± 𝑏𝐿−1[𝐺(𝑠)]
Result: 2 First shifting property
(i) 𝑳−𝟏 [𝑭(𝒔 + 𝒂) = 𝒆−𝒂𝒕𝑳−𝟏[𝑭(𝒔)]]
(ii) 𝑳−𝟏 [𝑭(𝒔 − 𝒂) = 𝒆𝒂𝒕𝑳−𝟏[𝑭(𝒔)]]
Proof:
Let 𝐿[𝑒−𝑎𝑡𝑓(𝑡)] = 𝐹[𝑠 + 𝑎]
Operating 𝐿−1 on both sides, we get
𝑒−𝑎𝑡𝑓(𝑡) = 𝐿−1[𝐹[𝑠 + 𝑎]]
𝐿−1[𝐹[𝑠 + 𝑎]] = 𝑒−𝑎𝑡𝐿−1[𝐹(𝑠)]
Result: 3 Multiplication by s.
𝒅𝒕
If 𝑳−𝟏[𝑭(𝒔)] = 𝒇(𝒕) and 𝒇(𝟎) = 𝟎, then 𝑳−𝟏[𝒔𝑭(𝒔)] = 𝒅
𝑳−𝟏[𝑭(𝒔)]
Proof:
We know that 𝐿[𝑓′(𝑡)] = 𝑠𝐿[𝑓(𝑡)] − 𝑓(0) = 𝑠𝐹(𝑠)
Operating 𝐿−1 on both sides, we get
𝑓′(𝑡) = 𝐿−1[𝑠𝐹(𝑠)]
𝑑𝑡
𝑑
𝑓(𝑡) = 𝐿−1[𝑠𝐹(𝑠)]
𝑑𝑡
𝑑
𝐿−1[𝐹(𝑠)] = 𝐿−1[𝑠𝐹(𝑠)]
𝑑𝑡
∴ 𝐿−1[𝑠𝐹(𝑠)] = 𝑑
𝐿−1[𝐹(𝑠)]
∵ 𝑓(𝑡) = 𝐿−1[𝐹(𝑠)]
∵ 𝑔(𝑡) = 𝐿−1[𝐺(𝑠)]
Engineering Maths - IV
Laplace Transform
Page
33
Engineering Maths - IV
Result: 4 Division by s.
𝑳−𝟏 [𝑭(𝒔)
] =
𝒔
𝒕
𝐿−1[𝐹(𝑠)]𝑑𝑡
∫𝟎
Proof:
We know that 𝐿 [∫
𝑡
𝑓 (𝑡)𝑑𝑡] = 1
𝐿[𝑓(𝑡)] = 1
𝐹(𝑠)
0 𝑠 𝑠
Operating 𝐿−1 on both sides ,we get
∫0 𝑠
𝑡 1
𝑓 (𝑡)𝑑𝑡 = 𝐿−1 [ 𝐹(𝑠)]
𝑠
𝑡 −1 −1 1
∫0 𝐿 [𝐹(𝑠)] 𝑑𝑡 = 𝐿 [ 𝐹(𝑠)]
𝑠
∴ 𝐿−1 [𝐹(𝑠)
] = ∫
0
𝑡
𝐿−1[𝐹(𝑠)] 𝑑𝑡
Result: 5 Inverse Laplace transform of derivative
𝑳−𝟏[𝑭(𝒔)] = −𝟏
𝑳−𝟏 [ 𝒅
𝑭(𝒔)]
𝒕 𝒅𝒔
Proof:
We know that 𝐿[𝑡𝑓(𝑡)] = −𝑑
𝐿[𝑓(𝑡)] = −𝑑
𝐹(𝑠)
𝑑𝑠 𝑑𝑠
Operating 𝐿−1 on both sides ,we get
𝑑𝑠
𝑑
𝑡𝑓(𝑡) = −𝐿−1 [ 𝐹(𝑠)]
𝐿−1[𝐹(𝑠)] = −1
𝐿−1 [ 𝑑
𝐹(𝑠)]
𝑡 𝑑𝑠
𝑓(𝑡) = −1
𝐿−1 [ 𝑑
𝐹(𝑠)]
𝑡 𝑑𝑠
𝐿−1[𝐹(𝑠)] = −1
𝐿−1 [ 𝑑
𝐹(𝑠)]
𝑡 𝑑𝑠
∞
𝐹(𝑠)𝑑𝑠]
Result: 6 Inverse Laplace transform of integral
𝑳−𝟏[𝑭(𝒔)] = 𝒕𝐿−1[∫
𝑠
Proof:
We know that 𝐿 [𝑓(𝑡)
] =
𝑡
∫𝑠
∞
𝐿(𝑓(𝑡)) 𝑑𝑠
= ∫𝑠
∞
𝐹(𝑠) 𝑑𝑠
Operating 𝐿−1 on both sides, we get
𝑡
𝑓(𝑡)
= 𝐿−1[∫𝑠
∞
𝐹(𝑠) 𝑑𝑠]
𝑓(𝑡) = 𝑡𝐿−1[∫
𝑠
∞
𝐹(𝑠) 𝑑𝑠]
𝐿−1[𝐹(𝑠)] = 𝑡𝐿−1[∫
𝑠
∞
𝐹(𝑠) 𝑑𝑠]
Problems under inverse Laplace transform of elementary functions
Example: 5.39 Find the inverse Laplace for the following
(i)
𝟏 𝟏
𝟐𝒔+𝟑 𝟒𝒔𝟐+𝟗
(ii) (iii)
𝒔𝟑−𝟑𝒔𝟐+𝟕
𝒔𝟒
(iv)
𝟑𝒔+𝟓
𝒔𝟐+𝟑𝟔
Solution:
Laplace Transform
Page
34
(i) 𝐿−1 𝟏
𝟐𝒔+𝟑
[ ] = 𝐿−1 [
𝟏
𝟑
𝟐
𝟐[𝒔+ ]
]
=
𝟏
𝟐
−𝟑𝒕
𝑒− 𝟐
(ii) 𝐿−1 [
𝟏
𝟒𝒔𝟐+𝟗
] = 𝐿−1 [
𝟏
𝟐 𝟗
𝟒[𝒔 +𝟒
]
]
𝟒
= 𝟏
𝐿−1 [
𝟏
𝟐 𝟗
𝟒
[𝒔 + ]
]
4 3⁄2 2
= 1 1
𝑠𝑖𝑛 3
𝑡
6 2
= 1
𝑠𝑖𝑛 3
𝑡
3 2 3 2
(iii) 𝐿−1 [𝑠 −3𝑠 +7
] = 𝐿−1 [𝑠
− 3𝑠
+ 7
]
𝑠4 𝑠4 𝑠4 𝑠4
𝑠 𝑠2 𝑠4
1 1
= 𝐿−1 [1
] − 3𝐿−1 [ ] + 7𝐿−1 [ ]
𝑠4
3 2 3
𝐿−1 [𝑠 −3𝑠 +7
] = 1 − 3𝑡 + 7𝑡
3!
3𝑠+5 𝑠 1
(iv) 𝐿−1 [ ] = 3𝐿−1 [ ] + 5𝐿−1 [ ]
𝑠2+36 𝑠2+36 𝑠2+36
𝐿−1 [ 3𝑠+5
] = 3𝑐𝑜𝑠6𝑡 + 5𝑠𝑖𝑛6𝑡
𝑠2+36 6
Inverse Laplace transform using First shifting theorem
𝑳−𝟏[𝑭(𝒔 + 𝒂)] = 𝒆−𝒂𝒕𝑳−𝟏[𝑭(𝒔)]
Example: 5.40 Find the inverse Laplace transform for the following:
(i)
𝟏 𝟏
(𝒔+𝟐)𝟐
𝟏
(ii) (iii)
𝟏
(iv)
(𝒔−𝟑)𝟒
𝒔+𝟐
𝟏
𝒔𝟐−𝟐𝒔+𝟐
𝒔
(v)
𝒔𝟐−𝟒𝒔+𝟏𝟑
𝒔
(vi) (vii)
(𝒔+𝟑)𝟐+𝟗
𝒔+𝟐
(viii)
(𝒔+𝟐)𝟐+𝟐𝟓
𝒔
𝒔𝟐+𝟒𝒔+𝟐𝟎
𝟐𝒔+𝟑
(𝒔+𝟑)𝟐
𝒔
(ix)
(𝒔−𝟒)𝟑
(x)
𝒔𝟐−𝟐𝒔+𝟐
(xi)
𝒔𝟐+𝟔𝒔+𝟐𝟓
(xii)
𝒔𝟐+𝟔𝒔−𝟕
Solution:
(i) 𝐿−1 [ 1
(𝑠+2)2 𝑠2
] = 𝑒−2𝑡 𝐿−1 [ 1
] = 𝑒−2𝑡 𝑡
(ii) 𝐿−1 [ 1
𝑠4
3
] = 𝑒3𝑡𝐿−1 [ 1
] = 𝑒−2𝑡 𝑡
3!
(iii) 𝐿−1 [
(𝑠−3)4
1
(𝑠+3)2+9
] = 𝑒−3𝑡𝐿−1 [
𝑠2+9
1
] = 𝑒−3𝑡 𝑠𝑖𝑛3𝑡
3
(iv) 𝐿−1 [ 1
] = 𝐿−1 [
1
(𝑠−1)2+1
1
𝑠2+1
] = 𝑒𝑡𝐿−1 [ ] = 𝑒𝑡𝑠𝑖𝑛𝑡
(v) 𝐿−1 [
𝑠2−2𝑠+2
1
] = 𝐿−1 [
1
(𝑠−2)2+9
] = 𝑒2𝑡𝐿−1 [
𝑠2+9
1
] = 𝑒2𝑡 𝑠𝑖𝑛3𝑡
3
(vi) 𝐿−1 [
𝑠2−4𝑠+13
𝑠+2
(𝑠+2)2+25
] = 𝑒−2𝑡𝐿−1 [ 𝑠
𝑠2+25
] = 𝑒−2𝑡𝑐𝑜𝑠5𝑡
(vii) 𝐿−1 [ 𝑠+2
𝑠2+4𝑠+20
] = 𝐿−1 [
𝑠+2
(𝑠+2)2+16
]
Engineering Maths - IV
Laplace Transform
Page
35
𝑠2+16
𝑠
= 𝑒−2𝑡𝐿−1 [ ] = 𝑒−2𝑡𝑐𝑜𝑠4𝑡
(viii) 𝐿−1 [ 𝑠
] = 𝐿−1 [𝑠+3−3
]
(𝑠+3)2 (𝑠+3)2
𝑠2
𝑠+3 3
= 𝐿−1 [ ] − 𝐿−1 [ ]
(𝑠+3)2 (𝑠+3)2
= 𝐿−1 1 1
[ ] − 3𝐿−1 [ ]
𝑠+3 (𝑠+3)2
= 𝑒−3𝑡 − 3𝑒−3𝑡𝐿−1 1
[ ]
= 𝑒−3𝑡 − 3𝑒−3𝑡𝑡
(ix) 𝐿−1 [ 𝑠
] = 𝐿−1 [𝑠−4+4
]
(𝑠−4)3 (𝑠−4)3
𝑠−4 4
= 𝐿−1 [ ] + 𝐿−1 [ ]
(𝑠−4)3 (𝑠−4)3
= 𝐿−1 1 1
[ ] + 4𝐿−1 [ ]
(𝑠−4)2 (𝑠−4)3
= 𝑒4𝑡𝐿−1 1 1
[ ] + 4𝑒4𝑡𝐿−1 [ ]
𝑠2 𝑠3
2
= 𝑒4𝑡𝑡 + 4𝑒4𝑡 𝑡
2!
= 𝑒4𝑡𝑡 + 2𝑒4𝑡𝑡2
(x) 𝐿−1 [ 𝑠
𝑠2−2𝑠+2
] = 𝐿−1 [
(𝑠−1)2+1 (𝑠−1)2+1
𝑠
] = 𝐿−1 [ 𝑠−1+1
]
𝑠−1 1
= 𝐿−1 [ ] + 𝐿−1 [ ]
(𝑠−1)2+1 (𝑠−1)2+1
𝐿−1 [ 𝑠
𝑠 1
= 𝑒𝑡𝐿−1 [ ] + 𝑒𝑡𝐿−1 [ ]
𝑠2+1 𝑠2+1
] = 𝑒𝑡𝑐𝑜𝑠𝑡 + 𝑒𝑡𝑠𝑖𝑛𝑡
(xi) 𝐿−1 [
𝑠2−2𝑠+2
2𝑠+3
𝑠2+6𝑠+25
] = 𝐿−1 [
2𝑠+3
] = 𝐿−1 [2(𝑠+3−3)+3
]
(𝑠+3)2+16 (𝑠+3)2+16
𝑠2+16
= 𝐿−1 [2(𝑠+3)−6+3
]
(𝑠+3)2+16
= 𝑒−3𝑡𝐿−1 2𝑠−3
[ ]
𝑠 1
= 𝑒−3𝑡 [2𝐿−1 [ ] − 3𝐿−1 [ ]]
𝑠2+16 𝑠2+16
𝐿−1 [ 2𝑠+3
𝑠2+6𝑠+25 4
] = 𝑒−3𝑡 (2𝑐𝑜𝑠4𝑡 − 3𝑠𝑖𝑛4𝑡
)
(xii) 𝐿−1 [ 𝑠
𝑠2+6𝑠−7
] = 𝐿−1 [
𝑠
(𝑠+3)2−16
] = 𝐿−1 [
𝑠+3−3
(𝑠+3)2−16
]
𝑠−3
= 𝑒−3𝑡𝐿−1 [ ]
𝑠2−16
𝑠2−16
𝑠
= 𝑒−3𝑡𝐿−1 [ ] − 3𝑒−3𝑡𝐿−1 [
1
𝑠2−16
]
𝐿−1 [ 𝑠
𝑠2+6𝑠−7 4
] = 𝑒−3𝑡 [𝑐𝑜𝑠ℎ4𝑡 − 3𝑠𝑖𝑛ℎ4𝑡
]
Exercise: 5.7
Find the inverse Laplace transform for the following:
Engineering Maths - IV
Laplace Transform
Page
36
1. 2𝑠−3
𝑠2+52
Ans: 2𝑐𝑜𝑠5𝑡 − 3𝑠𝑖𝑛5𝑡
5
2. 3𝑠+5
Ans: 3𝑐𝑜𝑠4𝑡 + 5𝑠𝑖𝑛4𝑡
4
3.
𝑠2+16
1
6 2
Ans: 1
𝑠𝑖𝑛 3
𝑡
4.
4𝑠2+9
1
(𝑠+4)5
Ans:
4
𝑒−4𝑡 𝑡
4!
5.
1
𝑠2−4𝑠+13 3
𝟐𝒕
Ans: 𝒆
𝑠𝑖𝑛3𝑡
Inverse using the formula
𝑳−𝟏[𝑭(𝒔)] = −𝟏
𝑳−𝟏 [ 𝒅
𝑭(𝒔)]
𝒕 𝒅𝒔
Note: This formula is used when 𝐹(𝑠) is cot−1 ∅(𝑠) or tan−1 ∅(𝑠) or 𝑙𝑜𝑔∅(𝑠)
Example: 5.41 Find the inverse Laplace transform for the following
(i) 𝒄𝒐𝒕−𝟏 (𝒔
) (ii) 𝒕𝒂𝒏−𝟏 (𝒂
) (iii) 𝒄𝒐𝒕−𝟏𝒂𝒔
𝒂 𝒔
(iv) 𝒕𝒂𝒏−𝟏(𝒔 + 𝒂) (v) 𝒍𝒐𝒈(𝒔+𝒂
) (vi) 𝒄𝒐𝒕−𝟏 ( 𝟐
)
𝒔+𝒃 𝒔+𝟏 𝒔𝟐
(vii) 𝒕𝒂𝒏−𝟏 ( 𝟐
)
Solution:
(i) 𝐿−1 [𝑐𝑜𝑡−1 (𝑠
)] = −1
𝐿−1 [ 𝑑
(𝑐𝑜𝑡−1 (𝑠
))]
𝑎 𝑡 𝑑𝑠 𝑎
1+𝑠2
𝑎2
𝑡 𝑎 𝑡 𝑎2+𝑠2
𝑎2
𝑎
= −1
𝐿−1 [ −1
(1
)] = 1
𝐿−1 [ −1
(1
)]
𝑡
= 1
𝐿−1 [
𝑎
𝑠2+𝑎2
]
𝐿−1 [𝑐𝑜𝑡−1 (𝑠
)] = 1
𝑠𝑖𝑛𝑎𝑡
𝑎 𝑡
(ii) 𝐿−1 [𝑡𝑎𝑛−1 (𝑎
)] = −1
𝐿−1 [ 𝑑
(𝑡𝑎𝑛−1 (𝑎
))]
𝑠 𝑡 𝑑𝑠 𝑠
𝑡
= −1
𝐿−1 [
1
1+(𝑠
)
𝑎 2 𝑠2 𝑡
(−𝑎
)] = −1
𝐿−1 [
1
𝑠2+𝑎2 𝑠2
𝑠2
(−𝑎
)]
𝑡
= 1
𝐿−1 [
𝑎
𝑠2+𝑎2
]
𝐿−1 [𝑡𝑎𝑛−1 (𝑎
)] = 1
𝑠𝑖𝑛𝑎𝑡
𝑠 𝑡
(iii) 𝐿−1[𝑐𝑜𝑡−1𝑎𝑠] = −1
𝐿−1 [ 𝑑
(𝑐𝑜𝑡−1(𝑎𝑠))]
𝑡 𝑑𝑠
𝑡
= −1
𝐿−1 [
−1
1+𝑎2𝑠2 𝑡
(𝑎)] = 1
𝐿−1 [
𝑎
𝑎2
𝑎2(𝑠2+ 1
)
]
1
𝑎𝑡 𝑠2+ 1
𝑎2
1
𝑎𝑡
1
𝑎
𝑠𝑖𝑛 𝑡
1
𝑎
= 1
𝐿−1 [ ] = [ ]
𝐿−1[𝑐𝑜𝑡−1𝑎𝑠] = 1
𝑠𝑖𝑛 𝑡
𝑡 𝑎
(iv) 𝐿−1[𝑡𝑎𝑛−1(𝑠 + 𝑎)] = 𝑒−𝑎𝑡𝐿−1[𝑡𝑎𝑛−1𝑠]
Engineering Maths - IV
Laplace Transform
Page
37
Engineering Maths - IV
= 𝑒−𝑎𝑡 [−1
𝐿−1 [ 𝑑
(𝑡𝑎𝑛−1𝑠)]]
𝑡 𝑑𝑠
1
𝑡 1+𝑠2
= 𝑒−𝑎𝑡 (−1
) 𝐿−1 [ ]
𝑡
= −1
𝑒−𝑎𝑡𝐿−1 [ 1
1+𝑠2
]
−𝑎𝑡
𝐿−1 [𝑐𝑜𝑡−1 (𝑠
)] = −𝑒
𝑠𝑖𝑛𝑡
𝑎 𝑡
(v) 𝐿−1 [𝑙𝑜𝑔 (𝑠+𝑎
)] = −1
𝐿−1 [ 𝑑
(𝑙𝑜𝑔 (𝑠+𝑎
))]
𝑠+𝑏 𝑡 𝑑𝑠 𝑠+𝑏
= −1
𝐿−1 [ 𝑑
(log(𝑠 + 𝑎) − log(𝑠 + 𝑏))]
𝑡 𝑑𝑠
−
𝑡 𝑠+𝑎 𝑠+𝑏
= −1
𝐿−1 [ 1 1
]
= −1
[𝑒−𝑎𝑡 − 𝑒−𝑏𝑡]
𝑡
𝐿−1 [𝑙𝑜𝑔 (𝑠+𝑎
)] = −1
[𝑒−𝑎𝑡 − 𝑒−𝑏𝑡]
𝑠+𝑏 𝑡
(vi) 𝐿−1 [𝑐𝑜𝑡−1 ( 2
)] = 𝑒−𝑡𝐿−1 [𝑐𝑜𝑡−1 (2
)]
𝑠+1 𝑠
= 𝑒−𝑡 (−1
) 𝐿−1 [ 𝑑
(𝑐𝑜𝑡−1 (2
))]
𝑡 𝑑𝑠 𝑠
𝑡 1+ 4
𝑠2
𝑠2 𝑡
−𝑡
= 𝑒−𝑡 (−1
) 𝐿−1 [ −1
(−2
)] = − 𝑒
𝐿−1 [
1
𝑠2+4 𝑠2
𝑠2
( 2
)]
−𝑡
= − 𝑒
𝐿−1 [
2
𝑡 𝑠2+4
]
−𝑡
𝐿−1 [𝑐𝑜𝑡−1 ( 2
)] = − 𝑒
𝑠𝑖𝑛2𝑡
𝑠+1 𝑡
𝑡 𝑑𝑠
𝑠2 𝑠2
(vii) 𝐿−1 [𝑡𝑎𝑛−1 ( 2
)] = −1
𝐿−1 [ 𝑑
(𝑡𝑎𝑛−1 ( 2
))]
𝑡
= −1
𝐿−1 [
1
1+( )
𝑠2
2 2 𝑠3 𝑡
1
𝑠4+4 𝑠3
𝑠4
(−4
)] = 4
𝐿−1 [ ( 1
)]
𝑠
𝑡 𝑠4+4
= 4
𝐿−1 [ ]
𝑡
= 4
𝐿−1 [
𝑠
(𝑠2)2+22
]
𝑡
= 4
𝐿−1 [
𝑠
(𝑠2+2)2−(2𝑠)2
]
𝑡
= 4
𝐿−1 [
𝑠
(𝑠2+2+2𝑠)(𝑠2+2−2𝑠)
]
𝑡
= 4
𝐿−1 [ 𝑠
( −
1 1
−4𝑠 𝑠2+2+2𝑠 𝑠2+2−2𝑠
)] ∵ {
1
(𝑠2+𝑎𝑥+𝑏)(𝑠2+𝑎𝑥+𝑐)
= [ −
1 1 1
𝑐−𝑏 𝑠2+𝑎𝑥+𝑏 𝑠2+𝑎𝑥+𝑐
}
]
𝑡
= −1
𝐿−1 [( −
1 1
𝑠2+2𝑠+2 𝑠2−2𝑠+2
)]
𝑡
= −1
𝐿−1 [ −
1 1
(𝑠+1)2+1 (𝑠−1)2+1
]
∵ 𝑎2 + 𝑏2 = (𝑎 + 𝑏)2 − 2𝑎𝑏
Laplace Transform
Page
38
1
𝑡 𝑠2+1
1
𝑠2+1
= −1
(𝑒−𝑡𝐿−1 [ ] − 𝑒𝑡𝐿−1 [ ])
𝑡
= −1
(𝑒−𝑡𝑠𝑖𝑛𝑡 − 𝑒𝑡𝑠𝑖𝑛𝑡)
𝑡
= 𝑠𝑖𝑛𝑡
(𝑒−𝑡 − 𝑒𝑡)
𝑡
= 𝑠𝑖𝑛𝑡
2𝑠𝑖𝑛ℎ𝑡
𝑠2
𝐿−1 [𝑡𝑎𝑛−1 ( 2
)] = 2𝑠𝑖𝑛𝑡𝑠𝑖𝑛ℎ𝑡
𝑡
Inverse using the formula
𝒅𝒕
𝑳−𝟏[𝒔𝑭(𝒔)] = 𝒅
𝑳−𝟏[𝑭(𝒔)]
𝒔𝟐+𝒃𝟐
𝟐 𝟐
Example: 5.42 Find 𝑳−𝟏 [𝒔𝒍𝒐𝒈 (𝒔 +𝒂
)]
Solution:
2 2 2 2
𝐿−1 [𝑠𝑙𝑜𝑔 (𝑠 +𝑎
)] = 𝑑
𝐿−1 [𝑠𝑙𝑜𝑔 (𝑠 +𝑎
)] ⋯ (1)
𝑠2+𝑏2 𝑑𝑡 𝑠2+𝑏2
2 2 2 2
𝐿−1 [𝑙𝑜𝑔 (𝑠 +𝑎
)] = 𝐿−1 𝑑
[𝑙𝑜𝑔 (𝑠 +𝑎
)]
𝑠2+𝑏2 𝑑𝑠 𝑠2+𝑏2
= −1
𝐿−1 [ 𝑑
(log(𝑠2 + 𝑎2) − log(𝑠2 + 𝑏2))]
𝑡 𝑑𝑠
𝑡
= −1
𝐿−1 [
𝑠2+𝑎2
1 1
𝑠2+𝑏2
2𝑠 − 2𝑠]
𝑡
= −2
𝐿−1 [ −
𝑠 𝑠
𝑠2+𝑎2 𝑠2+𝑏2
]
𝑡
= −2
[𝑐𝑜𝑠𝑎𝑡 − 𝑐𝑜𝑠𝑏𝑡]
= 2
[𝑐𝑜𝑠𝑏𝑡 − 𝑐𝑜𝑠𝑎𝑡]
𝑡
Substituting in (1), we get
𝑡2
2 2
𝐿−1 [𝑠𝑙𝑜𝑔 (𝑠 +𝑎
)] = 𝑑
[2
[𝑐𝑜𝑠𝑏𝑡 − 𝑐𝑜𝑠𝑎𝑡]]
𝑠2+𝑏2 𝑑𝑡 𝑡
= 2 [𝑡(−𝑏𝑠𝑖𝑛𝑏𝑡+𝑎𝑠𝑖𝑛𝑎𝑡)−(𝑐𝑜𝑠𝑏𝑡−𝑐𝑜𝑠𝑎𝑡
)
]
𝑡2
2 2
𝐿−1 [𝑠𝑙𝑜𝑔 (𝑠 +𝑎
)] = 2 [𝑡(−𝑏𝑠𝑖𝑛𝑏𝑡+𝑎𝑠𝑖𝑛𝑎𝑡)−(𝑐𝑜𝑠𝑏𝑡−𝑐𝑜𝑠𝑎𝑡)
]
𝑠2+𝑏2
Inverse using the formula
𝑳−𝟏 [𝑭(𝒔)
] =
𝒔
𝒕 −𝟏
∫𝟎 𝑳 [𝑭(𝒔)]𝒅𝒕
This formula is used when 𝐹(𝑠) =
𝑜𝑛𝑒 𝑡𝑒𝑟𝑚
𝑠(𝑎𝑛𝑜𝑡ℎ𝑒𝑟 𝑡𝑒𝑟𝑚)
𝒔(𝒔𝟐+𝒂𝟐)
Example: 5.43 Find 𝑳−𝟏 [ 𝟏
]
Solution:
𝐿−1 [ 1
𝑠(𝑠2+𝑎2)
1
(𝑠2+𝑎2)
] 𝑑𝑡
𝑡
𝐿−1 [
] = ∫0
𝑎
𝑡
[𝑠𝑖𝑛𝑎𝑡
] 𝑑𝑡
= ∫0
Engineering Maths - IV
Laplace Transform
Page
39
= [
1 −𝑐𝑜𝑠𝑎𝑡
𝑎 𝑎
𝑡
]
0
= −1
[𝑐𝑜𝑠𝑎𝑡]𝑡
𝑎2 0
= −1
(𝑐𝑜𝑠𝑎𝑡 − 𝑐𝑜𝑠0) = −1
(𝑐𝑜𝑠𝑎𝑡 − 1)
𝑎2 𝑎2
𝑠(𝑠2+𝑎2)
1 1−𝑐𝑜𝑠𝑎𝑡
∴ 𝐿−1 [ ] = 𝑎2
𝒔(𝒔𝟐−𝒂𝟐)
Example: 5.44 Find 𝑳−𝟏 [ 𝟏
]
Solution:
𝐿−1 [ 1
𝑠(𝑠2+𝑎2)
1
(𝑠2−𝑎2)
] 𝑑𝑡
𝑡
𝐿−1 [
] = ∫0
𝑎
𝑡
[𝑠𝑖𝑛ℎ𝑎𝑡
] 𝑑𝑡
= ∫0
= [
1 𝑐𝑜𝑠ℎ𝑎𝑡
𝑎 𝑎
𝑡
]
0
= 1
[𝑐𝑜𝑠ℎ𝑎𝑡]𝑡
𝑎2 0
= 1
(𝑐𝑜𝑠ℎ𝑎𝑡 − 𝑐𝑜𝑠ℎ0) = 1
(𝑐𝑜𝑠ℎ𝑎𝑡 − 1)
𝑎2 𝑎2
𝑠(𝑠2−𝑎2)
1 𝑐𝑜𝑠ℎ𝑎𝑡−1
∴ 𝐿−1 [ ] = 𝑎2
𝒔(𝒔+𝒂)
Example: 5.45 Find 𝑳−𝟏 [ 𝟏
]
Solution:
𝐿−1 [ 1
𝑠(𝑠+𝑎)
1
(𝑠+𝑎)
] 𝑑𝑡
𝑡
𝐿−1 [
] = ∫0
𝑡
𝑒−𝑎𝑡𝑑𝑡
= ∫0
−𝑎
= [ ]
0
𝑒−𝑎𝑡 𝑡
𝑎
= −1
(𝑒−𝑎𝑡 − 1)
∴ 𝐿−1 [
1
] = 1−𝑒−𝑎𝑡
𝑠(𝑠+𝑎) 𝑎
Inverse using Partial Fraction
Example: 5.46 Find
𝒔(𝒔+𝟐)(𝒔−𝟏)
𝒔−𝟐
𝑳−𝟏 [ ]
Solution:
= 𝐴
+ +
𝑠−2 𝐵 𝐶
𝑠(𝑠+2)(𝑠−1) 𝑠 𝑠+2 𝑠−1
= 𝐴(𝑠+2)(𝑠−1)+𝐵𝑠(𝑠−1)+𝐶𝑠(𝑠+2)
𝑠(𝑠+2)(𝑠−1)
𝐴(𝑠 + 2)(𝑠 − 1) + 𝐵𝑠(𝑠 − 1) + 𝐶𝑠(𝑠 + 2) = 𝑠 − 2 ⋯ (1)
Put 𝑠 = 0 in (1) Put 𝑠 = −2 in (1) Put 𝑠 = 1 in (1)
𝐴(2)(−1) = −2 𝐵(−2)(−3) = −4 3𝐶 = −1
Engineering Maths - IV
Laplace Transform
Page
40
⇒ 𝐴 = 1 ⇒ 𝐵 = −4
= −2
6 3
⇒ 𝐶 = −1
3
∴
𝑠−2 2
= 1
− −
𝑠(𝑠+2)(𝑠−1) 𝑠 𝑠+2 3(𝑠−1)
1
𝑠−2 1 2 1 1 1
𝐿−1 [ ] = 𝐿−1 [ ] − 𝐿−1 [ ] − 𝐿−1 [ ]
𝑠(𝑠+2)(𝑠−1) 𝑠 3 𝑠+2 3 𝑠−1
𝐿−1 [ 𝑠−2
] = 1 − 2
𝑒−2𝑡 − 1
𝑒𝑡
𝑠(𝑠+2)(𝑠−1) 3 3
Example: 5.47 Find 𝐿−1 [ 2𝑠−3
(𝑠−1)(𝑠−2)2
]
Solution:
2𝑠−3 𝐴 𝐵
= + +
(𝑠−1)(𝑠−2)2 𝑠−1 𝑠−2 (𝑠−2)2
𝐶
2
= 𝐴(𝑠−2) +𝐵(𝑠−1)(𝑠−2)+𝐶(𝑠−1)
(𝑠−1)(𝑠−2)2
𝐴(𝑠 − 2)2 + 𝐵(𝑠 − 1)(𝑠 − 2) + 𝐶(𝑠 − 1) = 2𝑠 − 3 ⋯ (1)
Put 𝑠 = 1 in (1) Put 𝑠 = 2 in (1) Equating the coefficient of 𝑠2
𝐴 = −1 𝐶 = 1
𝐴 + 𝐵 = 0
𝐵 = −𝐴 ⇒ 𝐵 = 1
∴
2𝑠−3
= −1
+ 1
+
1
(𝑠−1)(𝑠−2)2 𝑠−1 𝑠−2 (𝑠−2)2
2𝑠−3 1 1 1
𝐿−1 [ ] = −𝐿−1 [ ] + 𝐿−1 [ ] + [ ]
(𝑠−1)(𝑠−2)2 𝑠−1 𝑠−2 (𝑠−2)2
𝑠2
= −𝑒𝑡 + 𝑒2𝑡 + 𝑒2𝑡𝐿−1 [ 1
]
(𝑠−1)(𝑠−2)2
2𝑠−3
∴ 𝐿−1 [ ] = −𝑒𝑡 + 𝑒2𝑡 + 𝑒2𝑡𝑡
𝟐
Example: 5.48 Find the inverse Laplace transform of 𝟓𝒔 −𝟏𝟓𝒔−𝟏𝟏
(𝒔+𝟏)(𝒔−𝟐)𝟑
Solution:
5𝑠2−15𝑠−11
= 𝐴 𝐵
+ + +
𝐶 𝐶
(𝑠+1)(𝑠−2)3 𝑠+1 𝑠−2 (𝑠−2)2 (𝑠−2)3
3 2
= 𝐴(𝑠−2) +𝐵(𝑠+1)(𝑠−2) +𝐶(𝑠+1)(𝑠−2)+𝐷(𝑠+1)
(𝑠−1)(𝑠−2)3
𝐴(𝑠 − 2)3 + 𝐵(𝑠 + 1)(𝑠 − 2)2 + 𝐶(𝑠 + 1)(𝑠 − 2) + 𝐷(𝑠 + 1) = 5𝑠2 − 15𝑠 − 11⋯ (1)
Put 𝑠 = −1 in (1)
𝐴(−27) = 9
Put 𝑠 = 2 in (1)
𝐷(3) = −21
Equating the coefficient of 𝑠3
𝐴 + 𝐵 = 0
𝐴 =
9
−27
⇒ 𝐴 = −1
3 3
𝐷 = −21
= −7 𝐵 = −𝐴 ⇒ 𝐵 = 1
3
Put 𝑠 = 0 in (1), we get
−8𝐴 + 4𝐵 − 2𝐶 + 𝐷 = −11
Engineering Maths - IV
Laplace Transform
Page
41
8
+ 4
− 2𝐶 − 7 = −11
3 3
4 − 2𝐶 = 7 − 11
−2𝐶 = −8 ⇒ 𝐶 = 4
2
∴ 5𝑠 −15𝑠−11
= −1
+ + −
1 4 7
(𝑠+1)(𝑠−2)3 3(𝑠+1) 3(𝑠−2) (𝑠−2)2 (𝑠−2)3
2
𝐿−1 [5𝑠 −15𝑠−11
] = −1
𝐿−1 [ 1
] + 1
𝐿−1 [ 1
] + 4𝐿−1 [
(𝑠+1)(𝑠−2)3 3 𝑠+1 3 𝑠−2
1
(𝑠−2)2
] − 7𝐿−1 [
1
(𝑠−2)3
]
3 3 𝑠2 𝑠3
= −1
𝑒−𝑡 + 1
𝑒2𝑡 + 4𝑒2𝑡 𝐿−1 [ 1
] − 7𝑒2𝑡 𝐿−1 [ 1
]
𝑠2
2 2
𝐿−1 [5𝑠 −15𝑠−11
] = −1
𝑒−𝑡 + 1
𝑒2𝑡 + 4𝑒2𝑡 𝐿−1 [ 1
] − 7𝑒2𝑡 𝑡
(𝑠+1)(𝑠−2)3 3 3 2
Example: 5.49 Find the inverse Laplace transform of
𝟒𝒔+𝟓
(𝒔+𝟏)(𝒔𝟐+𝟒)
Solution:
4𝑠+5
=
𝐴
+ 𝐵𝑠+𝑐
(𝑠+1)(𝑠2+4) 𝑠+1 𝑠2+4
2
= 𝐴(𝑠 +4)+(𝐵𝑠+𝑐)(𝑠+1)
(𝑠+1)(𝑠2+4)
𝐴(𝑠2 + 4) + (𝐵𝑠 + 𝑐)(𝑠 + 1) = 4𝑠 + 5 ⋯ ⋯ (1)
Put 𝑠 = −1in (1)
𝐴(1 + 4) + 0 = 4(−1) + 5
Equating coefficients of 𝑠2term in (1)
𝐴 + 𝐵 = 0
Put 𝑠 = 0in (1)
𝐴(4) + 𝐶 = 5
𝐴(5) = 1 ⇒ 𝐴 = 1
5
𝐵 = −𝐴 ⇒ 𝐵 = −1
5
𝐶 = 5 − 4𝐴 = 5 − 4
5
= 25−4
= 21
5 5
∴
4𝑠+5
=
(𝑠+1)(𝑠2+4) 𝑠+1
−1 21
1
𝑠+
5
+ 5 5
𝑠2+4
= −
1 𝑠
+ 21 1
5(𝑠+1) 5(𝑠2+4) 5 (𝑠2+4)
𝐿−1 [ 4𝑠+5 𝑠
(𝑠+1)(𝑠2+4) 5 𝑠+1 5 𝑠2+4 5
1
𝑠2+4
] = 1
𝐿−1 [ 1
] − 1
𝐿−1 [ ] + 21
𝐿−1 [ ]
= 1
𝑒−𝑡 − 1
𝑐𝑜𝑠2𝑡 + 21 𝑠𝑖𝑛2𝑡
5 5 5 2
(𝑠+1)(𝑠2+4) 5 5 10
4𝑠+5 1 1 21
𝐿−1 [ ] = 𝑒−𝑡 − 𝑐𝑜𝑠2𝑡 + 𝑠𝑖𝑛2𝑡
Exercise: 5.8
Find the Inverse Laplace transforms using partial fraction for the following
1.
1
(𝑠+1)(𝑠+3)
Ans:
2
1
(𝑒−𝑡 − 𝑒−3𝑡)
2.
1
𝑠(𝑠+1)(𝑠+2)
Ans:
2
1
(𝑒−2𝑡 − 2𝑒−𝑡 + 1)
3.
54−3𝑠−5
(𝑠+1)(𝑠2−3𝑠+2)
Ans: 2𝑒−𝑡
3𝑡 3𝑡
𝑡 𝑡
+ 2 𝑒 2 𝑐𝑜𝑠ℎ 2 +8𝑒 2 𝑠𝑖𝑛ℎ
2 2
5. 7 INITIALAND FINAL VALUE THEOREMS
Initial value theorem
Engineering Maths - IV
Laplace Transform
Page
42
𝑡→0 𝑠→∞
Proof:
We know that 𝐿[𝑓′(𝑡)] = 𝑠 𝐿[𝑓(𝑡)] − 𝑓(0)
= 𝑠𝐹(𝑠) − 𝑓(0)
∴ 𝑠𝐹(𝑠) = 𝐿[𝑓′(𝑡)] + 𝑓(0)
= ∫
∞
𝑒−𝑠𝑡𝑓′(𝑡)𝑑𝑡 + 𝑓(0)
0
Taking limit as 𝑠 → ∞ on both sides, we have
Engineering Maths - IV
Statement: If𝐿[𝑓(𝑡)] = 𝐹(𝑠), then lim 𝑓(𝑡) = lim𝑠𝐹(𝑠)
𝑠→∞ 𝑠→∞
lim𝑠𝐹(𝑠) = lim[∫0
∞
𝑒−𝑠𝑡𝑓′(𝑡)𝑑𝑡 + 𝑓(0)]
𝑠→∞ 0
∞
= lim[∫ 𝑒−𝑠𝑡 𝑓′ (𝑡)𝑑𝑡] + 𝑓(0)
𝑠→∞
∞
lim[𝑒−𝑠𝑡𝑓′(𝑡)]𝑑𝑡 + 𝑓(0)
= ∫0
∵ 𝑒−∞ = 0
𝑠→∞ 𝑡→0
= 0 + 𝑓(0)
= 𝑓(0)
= lim𝑓(𝑡)
𝑡→0
∴ lim𝑠𝐹(𝑠) = lim𝑓(𝑡)
𝑡→∞ 𝑠→0
Final value theorem
Statement: If the Laplace transforms of 𝑓(𝑡) and 𝑓′(𝑡) exist and 𝐿[𝑓(𝑡)] = 𝐹(𝑠),then lim𝑓(𝑡) = lim𝑠𝐹(𝑠)
Proof:
We know that 𝐿[𝑓′(𝑡)] = 𝑠 𝐿[𝑓(𝑡)] − 𝑓(0)
= 𝑠𝐹(𝑠) − 𝑓(0)
∴ 𝑠𝐹(𝑠) = 𝐿[𝑓′(𝑡)] + 𝑓(0)
= ∫
∞
𝑒−𝑠𝑡𝑓′(𝑡)𝑑𝑡 + 𝑓(0)
0
Taking limit as 𝑠 → 0 on both sides, we have
𝑠→0 𝑠→0 0
lim𝑠𝐹(𝑠) = lim[∫
∞
𝑒−𝑠𝑡𝑓′(𝑡)𝑑𝑡 + 𝑓(0)]
= lim[
𝑠→0 0
∞
∫ 𝑒−𝑠𝑡𝑓′(𝑡)𝑑𝑡] + 𝑓(0)
𝑠→0
∞
lim[𝑒−𝑠𝑡𝑓′(𝑡)]𝑑𝑡 + 𝑓(0)
= ∫0
∞
𝑓′(𝑡)𝑑𝑡 + 𝑓(0)
= ∫0
= [𝑓(𝑡)]∞ + 𝑓(0)
𝑡→∞ 𝑠→0
0
= 𝑓(∞) − 𝑓(0) + 𝑓(0)
= 𝑓(∞)
= lim𝑓(𝑡)
𝑡→∞
∴ lim𝑓(𝑡) = lim𝑠𝐹(𝑠)
Example: 5.50 Verify the initial value theorem for the function 𝒇(𝒕) = 𝒂𝒆−𝒃𝒕
Laplace Transform
Page43
Engineering Maths - IV
Solution:
Given 𝑓(𝑡) = 𝑎𝑒−𝑏𝑡
𝐹(𝑠) = 𝐿[𝑓(𝑡)]
= 𝐿[𝑎𝑒−𝑏𝑡]
= 𝑎
1
𝑠+𝑏
𝑠𝐹(𝑠) = 𝑎𝑠
𝑠+𝑏
Initial value theorem is lim𝑓(𝑡) = lim𝑠𝐹(𝑠)
𝑡→0 𝑠→∞
𝑡→0
lim𝑓(𝑡) = lim 𝑎𝑒−𝑏𝑡
𝑡→0
= 𝑎 ⋯ ⋯ ⋯ (1)
𝑠→∞ 𝑠→∞ 𝑠+𝑏
𝑎𝑠
lim𝑠𝐹(𝑠) = lim [ ]
= lim [
𝑎𝑠
𝑏
𝑠→∞ 𝑠(1+𝑠
)
𝑎
𝑏
𝑠→∞ (1+𝑠
)
] == lim [ ]
𝑡→0 𝑠→∞
= 𝑎 ⋯ ⋯ ⋯ (2)
From (1) and (2), lim𝑓(𝑡) = lim𝑠𝐹(𝑠)
∴ Initial value theorem is verified
Example: 5.51Verify the initial value theorem and Final value theorem for the function
𝒇(𝒕) = 𝟏 + 𝒆−𝒕[𝒔𝒊𝒏𝒕 + 𝒄𝒐𝒔𝒕].
Solution:
Given 𝑓(𝑡) = 1 + 𝑒−𝑡[𝑠𝑖𝑛𝑡 + 𝑐𝑜𝑠𝑡]
𝐹(𝑠) = 𝐿[𝑓(𝑡)]
= 𝐿[1 + 𝑒−𝑡[𝑠𝑖𝑛𝑡 + 𝑐𝑜𝑠𝑡]]
= 𝐿[1] + 𝐿[𝑒−𝑡[𝑠𝑖𝑛𝑡 + 𝑐𝑜𝑠𝑡]]
= 𝐿[1] + 𝐿[𝑠𝑖𝑛𝑡 + 𝑐𝑜𝑠𝑡]𝑠→𝑠+1
𝑠 𝑠2+1
1 𝑠
= 1
+ [ + ]
𝑠2+1 𝑠→𝑠+1
= 1
+ +
1 𝑠+1
𝑠 (𝑠+1)2+1 (𝑠+1)2+1
𝐹(𝑠) = 1
+ +
1 𝑠+1
𝑠 𝑠2+2𝑠+2 𝑠2+2𝑠+2
𝑠𝐹(𝑠) = 1 + +
𝑠 𝑠2+𝑠
𝑠2+2𝑠+2 𝑠2+2𝑠+2
Initial value theorem is lim𝑓(𝑡) = lim𝑠𝐹(𝑠)
𝑡→0 𝑠→∞
lim𝑓(𝑡) = lim[1 + 𝑒−𝑡[𝑠𝑖𝑛𝑡 + 𝑐𝑜𝑠𝑡]]
𝑡→0 𝑡→0
= 1 + 0 + 1 = 2 ⋯ ⋯ ⋯ (1)
lim𝑠𝐹(𝑠) = lim [1 +
𝑠→∞ 𝑠→∞
+
𝑠 𝑠2+𝑠
𝑠2+2𝑠+2 𝑠2+2𝑠+2
]
Laplace Transform
Page44
= 1 + lim [
1
1
(1+𝑠
)
2 2 2 2
𝑠→∞ 𝑠(1+𝑠+𝑠2) (1+𝑠+𝑠2)
+ ]
𝑡→0 𝑠→∞
= 1 + 0 + 1 = 2 ⋯ ⋯ ⋯ (2)
From (1) and (2), lim𝑓(𝑡) = lim𝑠𝐹(𝑠)
𝑡→∞ 𝑠→0
∴ Initial value theorem is verified
Final value theorem is lim𝑓(𝑡) = lim𝑠𝐹(𝑠)
lim𝑓(𝑡) = lim(1 + 𝑒−𝑡[𝑠𝑖𝑛𝑡 + 𝑐𝑜𝑠𝑡])
𝑡→∞ 𝑡→∞
= 1 + 0 = 1 ⋯ ⋯ ⋯ (3)
𝑠→0 𝑠→0
lim𝑠𝐹(𝑠) = lim [1 + +
𝑠 𝑠2+𝑠
𝑠2+2𝑠+2 𝑠2+2𝑠+2
]
𝑡→∞ 𝑠→0
= 1 + 0 + 0 = 1 ⋯ ⋯ ⋯ (4)
From (3) and (4), lim𝑓(𝑡) = lim𝑠𝐹(𝑠)
∴ Final value theorem is verified.
Example: 5.52 Verify the initial value theorem and Final value theorem for the function
𝒔(𝒔+𝟐)𝟐
𝟏
𝒇(𝒕) = 𝑳−𝟏 [ ]
Solution:
𝑠(𝑠+2)2
1
Given 𝑓(𝑡) = 𝐿−1 [ ] ⋯ (1)
(𝑠+2)2
𝑡 −1 1
= ∫
0 𝐿 [ ] 𝑑𝑡 = 𝑠2
𝑡 −2𝑡 −1 1
∫
0 𝑒 𝐿 [ ] 𝑑𝑡
𝑡 −2𝑡
= ∫
0 𝑒 𝑡 𝑑𝑡
= ∫
𝑡
𝑡𝑒−2𝑡 𝑑𝑡
0
𝑒−2𝑡
= [𝑡 ( ) − ]
−2 (−2)2
0
(1)𝑒−2𝑡 𝑡
−2𝑡 −2𝑡
= −𝑡 𝑒
− 𝑒
− 0 + 1
2 4 4
−2𝑡 −2𝑡
∴ 𝑓(𝑡) = 1
− 𝑡𝑒
− 𝑒
4 2 4
From (1), 𝐹(𝑠) =
1
𝑠(𝑠+2)2
1
𝑠𝐹(𝑠) =
(𝑠+2)2
Initial value theorem is lim𝑓(𝑡) = lim𝑠𝐹(𝑠)
𝑡→0 𝑠→∞
𝑡→0 𝑡→0 4 2 4
−2𝑡 −2𝑡
lim𝑓(𝑡) = lim [1
− 𝑡𝑒
− 𝑒
]
= 1
− 0 − 1
= 0
4 4
∴ lim𝑓(𝑡) = 0 ⋯ (2)
𝑡→0
𝑠→∞
lim𝑠𝐹(𝑠) = lim
𝑠→∞ (𝑠+2)2
1
= 0
Engineering Maths - IV
Laplace Transform
Page45
𝑡→0 𝑠→∞
𝑡→∞ 𝑠→0
∴ Initial value theorem is verified
Final value theorem is lim𝑓(𝑡) = lim𝑠𝐹(𝑠)
Engineering Maths - IV
∴ lim𝑠𝐹(𝑠) = 0 ⋯ (3)
𝑠→∞
From (2) and (3), lim𝑓(𝑡) = lim𝑠𝐹(𝑠)
𝑡→∞ 𝑡→∞ 4 2 4
−2𝑡 −2𝑡
lim𝑓(𝑡) = lim [1
− 𝑡𝑒
− 𝑒
]
= 1
− 0 − 0 = 1
⋯ (4)
4 4
𝑠→0 𝑠→0 (𝑠+2)2
lim𝑠𝐹(𝑠) = lim [ 1
]
= 1
⋯ (5)
𝑡→∞ 𝑠→0
4
From (4) and (5), lim𝑓(𝑡) = lim𝑠𝐹(𝑠)
∴ Final value theorem is verified
Example: 5.53 Verify the initial value theorem and Final value theorem for the function
𝒇(𝒕) = 𝒆−𝒕(𝒕 + 𝟐)𝟐
Solution:
Given 𝑓(𝑡) = 𝑒−𝑡(𝑡 + 2)2
= 𝑒−𝑡(𝑡2 + 4𝑡 + 4)
𝐹(𝑠) = 𝐿[𝑓(𝑡)]
= 𝐿[𝑒−𝑡(𝑡2 + 4𝑡 + 4)]
= 𝐿[𝑡2 + 4𝑡 + 4]𝑠→𝑠+1
= [𝐿(𝑡2) + 4𝐿(𝑡) + 4𝐿(1)]𝑠→𝑠+1
𝑠3 𝑠2
= [2!
+ 4 1
+ 4 1
]
𝑠 𝑠→𝑠+1
=
2 1
+ 4 + 4
(𝑠+1)3 (𝑠+1)2 𝑠+1
1
𝑠𝐹(𝑠) = +
2𝑠 4𝑠
+ 4𝑠
(𝑠+1)3 (𝑠+1)2 𝑠+1
Initial value theorem is lim𝑓(𝑡) = lim𝑠𝐹(𝑠)
𝑡→0
𝑡→0 𝑠→∞
lim𝑓(𝑡) = lim[𝑒−𝑡(𝑡2 + 4𝑡 + 4)]
𝑡→0
= 4 ⋯ (1)
𝑠→∞
2𝑠
lim𝑠𝐹(𝑠) = lim [ +
𝑠→∞ (𝑠+1)3 (𝑠+1)2 𝑠+1
4𝑠
+ 4𝑠
]
𝑠→∞ 3
𝑠
𝑠 (1+ )
1 3
2𝑠 4𝑠
= lim [ +
2
𝑠 (1+𝑠
)
1 2 +
4𝑠
1
𝑠(1+𝑠
)
]
𝑠→∞ 2
𝑠
𝑠 (1+ )
1 3
2
= lim [ +
4
𝑠(1+𝑠
)
1 2 +
4
1
(1+𝑠
)
]
= 0 + 0 + 4
Laplace Transform
Page46
𝑡→0 𝑠→∞
𝑡→∞ 𝑠→0
∴ Initial value theorem is verified
Final value theorem is lim𝑓(𝑡) = lim𝑠𝐹(𝑠)
Engineering Maths - IV
= 4 ⋯ (2)
From (1) and (2), lim𝑓(𝑡) = lim𝑠𝐹(𝑠)
lim𝑓(𝑡) = lim[𝑒−𝑡(𝑡2 + 4𝑡 + 4)]
𝑡→∞ 𝑡→∞
𝑠→0 𝑠→0
= 0 ⋯ (3)
2𝑠
lim𝑠𝐹(𝑠) = lim [ +
(𝑠+1)3 (𝑠+1)2 𝑠+1
4𝑠
+ 4𝑠
]
= 0 ⋯ (4)
From (3) and (4), lim𝑓(𝑡) = lim𝑠𝐹(𝑠)
𝑡→∞ 𝑠→0
∴ Final value theorem is verified.
Example: 5.54 If 𝑳[𝒇(𝒕)] =
𝟏
𝒔(𝒔+𝟏)
, find the 𝐥𝐢𝐦𝒇(𝒕) and 𝐥𝐢𝐦𝒇(𝒕) using initial and final value
𝒕→𝟎 𝒕→∞
theorems.
Solution:
Given 𝐿[𝑓(𝑡)] =
1
𝑠(𝑠+1)
⋯ (1)
𝑖𝑒. , 𝐹(𝑠) =
1
=> 𝑠𝐹(𝑠) =
𝑠(𝑠+1) (𝑠+1)
1
Initial value theorem is lim𝑓(𝑡) = lim𝑠𝐹(𝑠)
𝑡→0 𝑠→∞
= lim
1
𝑠→∞ (𝑠+1)
= 0
Final value theorem is lim𝑓(𝑡) = lim𝑠𝐹(𝑠)
𝑡→∞ 𝑠→0
= lim
1
𝑠→0 (𝑠+1)
= 1
Exercise: 5.9
1. Verify the initial value theorem for the function
2. Verify the initial value theorem for the function
3. Verify the initial value theorem for the function
𝑓(𝑡) = 𝑒−𝑡𝑠𝑖𝑛𝑡
𝑓(𝑡) = sin2 𝑡
𝑓(𝑡) = 1 + 𝑒−𝑡 + 𝑡2
4. Verify the Final value theorem for the function 𝑓(𝑡) = 1 − 𝑒−𝑎𝑡
5. Verify the Final value theorem for the function 𝑓(𝑡) = 𝑡2𝑒−3𝑡
5.8 CONVOLUTION THEOREM
Definition: Convolution of two functions
The convolution of two functions 𝑓(𝑡) and 𝑔(𝑡) is denoted by 𝑓(𝑡) ∗ 𝑔(𝑡) and defined by
𝑡
𝑓(𝑢)𝑔(𝑡 − 𝑢)𝑑𝑢.
𝑓(𝑡) ∗ 𝑔(𝑡) = ∫0
State and prove Convolution theorem
Statement: If 𝐿[𝑓(𝑡)] = 𝐹(𝑠) and 𝐿[𝑔(𝑡)] = 𝐺(𝑠), then 𝐿[𝑓(𝑡)] ∗ 𝐿[𝑔(𝑡)] = 𝐹(𝑠)𝐺(𝑠)
Laplace Transform
Page47
Engineering Maths - IV
Proof:
𝑡
𝑓(𝑢)𝑔(𝑡 − 𝑢)𝑑𝑢
We have 𝑓(𝑡) ∗ 𝑔(𝑡) = ∫0
𝐿[𝑓(𝑡) ∗ 𝑔(𝑡)] = ∫
∞
[𝑓(𝑡) ∗ 𝑔(𝑡)] 𝑒−𝑠𝑡𝑑𝑡
0
∞ 𝑡
𝑓(𝑢)𝑔(𝑡 − 𝑢)𝑑𝑢𝑒−𝑠𝑡𝑑𝑡
= ∫0 ∫0
= ∫0 ∫0
∞ 𝑡
𝑓(𝑢)𝑔(𝑡 − 𝑢)𝑒−𝑠𝑡 𝑑𝑢𝑑𝑡 ⋯ (1)
Now we have no change the order of integration.
𝑢 = 0, 𝑢 = 𝑡;𝑡 = 0, 𝑡 = ∞
Change of order is . Draw horizontal strip PQ
At P, 𝑡 = 𝑢, At A 𝑢 = ∞
∞ ∞
𝑓(𝑢)𝑔(𝑡 − 𝑢)𝑒−𝑠𝑡𝑑𝑡𝑑𝑢
𝐿[𝑓(𝑡) ∗ 𝑔(𝑡)] = ∫0 ∫𝑢
∞
= ∫
0 𝑓(𝑢) [∫
𝑢
∞
𝑔(𝑡 − 𝑢)𝑒−𝑠𝑡 𝑑𝑡]𝑑𝑢 ⋯ (2)
Put 𝑡 − 𝑢 = 𝑥 ⋯ (3)
𝑡 = 𝑢 + 𝑥 ⇒ 𝑑𝑡 = 𝑑𝑥
When 𝑡 = 𝑢;(3) ⇒ 𝑥 = 0
When 𝑡 = ∞; (3) ⇒ 𝑥 = ∞
(2) ⇒ 𝐿[𝑓(𝑡) ∗ 𝑔(𝑡)] = ∫0
∞
𝑓(𝑢) [∫0
∞
𝑔(𝑥)𝑒−𝑠(𝑢+𝑥)𝑑𝑥]𝑑𝑢
∞
= ∫
0 𝑓(𝑢) [∫
0
∞
𝑔(𝑥)𝑒−𝑠𝑢𝑒−𝑠𝑥𝑑𝑥]𝑑𝑢
∞
𝑔(𝑥)𝑒−𝑠𝑥𝑑𝑥
∞ −𝑠𝑢
= ∫
0 𝑓(𝑢)𝑒 𝑑𝑢 ∫
0
= 𝐿[𝑓(𝑢)]𝐿[𝑔(𝑥)]
∴ 𝐿[𝑓(𝑡) ∗ 𝑔(𝑡)] = 𝐹(𝑠)𝐺(𝑠)
Note: Convolution theorem is very useful to compute inverse Laplace transform of product of two terms
Convolution theorem is 𝐿[𝑓(𝑡) ∗ 𝑔(𝑡)] = 𝐹(𝑠)𝐺(𝑠)
𝐿−1[𝐹(𝑠)𝐺(𝑠)] = 𝑓(𝑡) ∗ 𝑔(𝑡)
𝐿−1[𝐹(𝑠)𝐺(𝑠)] = 𝐿−1[𝐹(𝑠)] ∗ 𝐿−1[𝐺(𝑠)]
Problems under Convolution theorem
Example: 5.55 Find 𝑳−𝟏 [
𝟏
(𝒔+𝒂)(𝒔+𝒃)
] using convolution theorem.
Solution:
𝐿−1 1 1 1
[ ] = 𝐿−1 [ ] ∗ 𝐿−1 [ ]
(𝑠+𝑎)(𝑠+𝑏) (𝑠+𝑎) (𝑠+𝑏)
= 𝑒−𝑎𝑡 ∗ 𝑒−𝑏𝑡
= ∫0
𝑡
𝑒−𝑎𝑢𝑒−𝑏(𝑡−𝑢) 𝑑𝑢
∫0
= 𝑒−𝑏𝑡 𝑡
𝑒−𝑎𝑢 𝑒𝑏𝑢𝑑𝑢
∫0
= 𝑒−𝑏𝑡 𝑡
𝑒(𝑏−𝑎)𝑢 𝑑𝑢
Laplace Transform
Page48
−𝑏𝑡
= 𝑒 [
𝑏−𝑎
]
0
𝑒(𝑏−𝑎)𝑢 𝑡
−𝑏𝑡
= 𝑒
[𝑒(𝑏−𝑎)𝑡 − 1 ]
𝑏−𝑎
𝑏−𝑎
−𝑏𝑡
= 𝑒
[𝑒𝑏𝑡−𝑎𝑡 − 1 ]
=
1
𝑏−𝑎
[𝑒−𝑏𝑡+𝑏𝑡−𝑎𝑡 − 𝑒−𝑏𝑡 ]
∴ 𝐿−1 [ ] =
1 1
(𝑠+𝑎)(𝑠+𝑏) 𝑏−𝑎
[𝑒−𝑎𝑡 − 𝑒−𝑏𝑡 ]
Example: 5.56 Find the inverse Laplace transform
𝒔𝟐
(𝒔𝟐+𝒂𝟐)(𝒔𝟐+𝒃𝟐)
by using convolution theorem.
Solution:
𝐿−1 [
𝑠2
(𝑠2+𝑎2)(𝑠2+𝑏2)
𝑠 𝑠
] = 𝐿−1 [
(𝑠2+𝑎2) (𝑠2+𝑏2)
]
𝑠 𝑠
= 𝐿−1 [ ] ∗ 𝐿−1 [ ]
(𝑠2+𝑎2) (𝑠2+𝑏2)
= 𝑐𝑜𝑠𝑎𝑡 ∗ 𝑐𝑜𝑠𝑏𝑡
𝑡
= ∫0𝑐𝑜𝑠𝑎𝑢 𝑐𝑜𝑠𝑏(𝑡 − 𝑢)𝑑𝑢
= ∫0
𝑡 cos(𝑎𝑢+𝑏𝑡−𝑏𝑢)+cos(𝑎𝑢−𝑏𝑡+𝑏𝑢)
𝑑𝑢
= 1 ∫
2 0
2
𝑡
(cos(𝑎𝑢 + 𝑏𝑡 − 𝑏𝑢) + cos(𝑎𝑢 − 𝑏𝑡 + 𝑏𝑢)) 𝑑𝑢
= 1 𝑡
[cos(𝑎 − 𝑏) 𝑢 + 𝑏𝑡 + cos(𝑎 + 𝑏) 𝑢 − 𝑏𝑡]𝑑𝑢
∫
2 0
=
2 𝑎 −𝑏 𝑎 +𝑏
+ ]
0
𝑡
1
[𝑠𝑖𝑛[(𝑎−𝑏)𝑢+𝑏𝑡] 𝑠𝑖𝑛[(𝑎+𝑏)𝑢+𝑏𝑡]
= 1
[sin(𝑎𝑡−𝑏𝑡+𝑏𝑡)
+ sin(𝑎𝑡−𝑏𝑡+𝑏𝑡)
− 𝑠𝑖𝑛𝑏𝑡
+ 𝑠𝑖𝑛𝑏𝑡
]
2 𝑎−𝑏 𝑎+𝑏 𝑎−𝑏 𝑎+𝑏
= 1
[sin 𝑎𝑡
+ sin 𝑎𝑡
− 𝑠𝑖𝑛𝑏𝑡
+ 𝑠𝑖𝑛𝑏𝑡
]
2 𝑎−𝑏 𝑎+𝑏 𝑎−𝑏 𝑎+𝑏
= 1
[(𝑎+𝑏)𝑠𝑖𝑛𝑎𝑡+(𝑎−𝑏)𝑠𝑖𝑛𝑎𝑡−(𝑎+𝑏)𝑠𝑖𝑛𝑏𝑡+(𝑎−𝑏)𝑠𝑖𝑛𝑏𝑡
]
2 𝑎2−𝑏2
= 1
[2𝑎𝑠𝑖𝑛𝑎𝑡−2𝑏𝑠𝑖𝑛𝑏𝑡
]
2 𝑎2−𝑏2
= 1
[2(𝑎𝑠𝑖𝑛𝑎𝑡−𝑏𝑠𝑖𝑛𝑏𝑡)
]
2 𝑎2−𝑏2
∴ 𝐿−1 [
𝑠2
] = 𝑎𝑠𝑖𝑛𝑎𝑡−𝑏𝑠𝑖𝑛𝑏𝑡
(𝑠2+𝑎2)(𝑠2+𝑏2) 𝑎2−𝑏2
Example: 5.57 Find the inverse Laplace transform
𝟏
(𝒔𝟐+𝒂𝟐)(𝒔𝟐+𝒃𝟐)
by using convolution theorem.
Solution:
1 1
𝐿−1 [ 1
] = 𝐿−1 [
(𝑠2+𝑎2)(𝑠2+𝑏2) (𝑠2+𝑎2) (𝑠2+𝑏2)
]
1 1
= 𝐿−1 [ ] ∗ 𝐿−1 [ ]
(𝑠2+𝑎2) (𝑠2+𝑏2)
= 1
𝑠𝑖𝑛𝑎𝑡 ∗ 1
𝑠𝑖𝑛𝑏𝑡
𝑎 𝑏
Engineering Maths - IV
Laplace Transform
Page49
= 1 𝑡
∫ 𝑠𝑖𝑛𝑎𝑢 𝑠𝑖𝑛𝑏(𝑡 − 𝑢)𝑑𝑢
𝑎𝑏 0
∫
𝑎𝑏 0
= 1 𝑡 cos(𝑎𝑢−𝑏𝑡+𝑏𝑢)−cos(𝑎𝑢+𝑏𝑡−𝑏𝑢)
𝑑𝑢
=
1
∫
2𝑎𝑏 0
2
𝑡
(cos(𝑎𝑢 − 𝑏𝑡 + 𝑏𝑢) − cos(𝑎𝑢 + 𝑏𝑡 − 𝑏𝑢)) 𝑑𝑢
= 1 𝑡
∫ [cos[(a + b)u − bt] − cos[(a − b)u + bt]]𝑑𝑢
2 0
=
2𝑎𝑏 𝑎+𝑏 𝑎−𝑏
− ]
0
𝑡
1
[𝑠𝑖𝑛[(𝑎+𝑏)𝑢−𝑏𝑡] 𝑠𝑖𝑛[(𝑎−𝑏)𝑢+𝑏𝑡]
=
=
2𝑎𝑏 𝑎+𝑏 𝑎−𝑏 𝑎 +𝑏 𝑎 −𝑏
1
[sin(𝑎𝑡+𝑏𝑡−𝑏𝑡)
− sin(𝑎𝑡−𝑏𝑡+𝑏𝑡)
+ 𝑠𝑖𝑛𝑏𝑡
+ 𝑠𝑖𝑛𝑏𝑡
]
2𝑎𝑏 𝑎+𝑏 𝑎−𝑏 𝑎+𝑏 𝑎−𝑏
1
[sin 𝑎𝑡
− sin 𝑎𝑡
− 𝑠𝑖𝑛𝑏𝑡
+ 𝑠𝑖𝑛𝑏𝑡
]
=
1
[(𝑎−𝑏)𝑠𝑖𝑛𝑎𝑡−(𝑎+𝑏)𝑠𝑖𝑛𝑎𝑡+(𝑎−𝑏)𝑠𝑖𝑛𝑏𝑡+(𝑎+𝑏)𝑠𝑖𝑛𝑏𝑡
]
2𝑎𝑏 𝑎2−𝑏2
=
1
[−2𝑏𝑠𝑖𝑛𝑎𝑡+2𝑎𝑠𝑖𝑛𝑏𝑡
]
2𝑎𝑏 𝑎2−𝑏2
=
1
[2(𝑎𝑠𝑖𝑛𝑏𝑡−𝑏𝑠𝑖𝑛𝑎𝑡)
]
2𝑎𝑏 𝑎2−𝑏2
1 𝑎𝑠𝑖𝑛𝑏𝑡−𝑏𝑠𝑖𝑛𝑎𝑡
∴ 𝐿−1 [ ] =
(𝑠2+𝑎2)(𝑠2+𝑏2) 𝑎𝑏(𝑎2−𝑏2)
Example: 5.58 Find the inverse Laplace transform
𝒔
(𝒔𝟐+𝟒)(𝒔𝟐+𝟗)
by using convolution theorem.
Solution:
𝑠 1 𝑠
𝐿−1 [ ] = 𝐿−1 [
(𝑠2+4)(𝑠2+9) (𝑠2+4) (𝑠2+9)
]
1 𝑠
= 𝐿−1 [ ] ∗ 𝐿−1 [ ]
(𝑠2+4) (𝑠2+9)
2
= 1
𝑠𝑖𝑛2𝑡 ∗ 𝑐𝑜𝑠3𝑡
= 1 𝑡
∫ 𝑠𝑖𝑛2𝑢 𝑐𝑜𝑠3(𝑡 − 𝑢)𝑑𝑢
2 0
2
∫
2 0
= 1 𝑡 sin(2𝑢+3𝑡−3𝑢)+sin(2𝑢−3𝑡+3𝑢)
𝑑𝑢
= 1 𝑡
[sin(3t − 𝑢) + sin(5u − 3t)]𝑑𝑢
∫
4 0
=
4 −1 5
− ]
0
𝑡
1
[−cos(3𝑡−𝑢) cos(5𝑢−3𝑡)
= 1
[cos(3t−t)
− cos(5t−3𝑡)
− 𝑐𝑜𝑠3𝑡
+ 𝑐𝑜𝑠3𝑡
]
4 1 5 1 5
= 1
[𝑐𝑜𝑠2𝑡 − cos2𝑡
− 𝑐𝑜𝑠3𝑡 + 𝑐𝑜𝑠3𝑡
]
4 5 5
= 1
[5𝑐𝑜𝑠2𝑡−𝑐𝑜𝑠2𝑡−5𝑐𝑜𝑠3𝑡+𝑐𝑜𝑠3𝑡
]
4 5
20
= 1
[4𝑐𝑜𝑠2𝑡 − 4𝑐𝑜𝑠3𝑡]
∴ 𝐿−1 [
𝑠
] = 𝑐𝑜𝑠2𝑡−𝑐𝑜𝑠3𝑡
(𝑠2+4)(𝑠2+9) 5
Example: 5.59 Find 𝑳−𝟏 [ 𝒔
] by using convolution theorem.
(𝒔𝟐+𝒂𝟐)
𝟐
Engineering Maths - IV
Laplace Transform
Page50
Engineering Maths - IV
Solution:
𝑠 1
𝐿−1 [ ] = 𝐿−1 [
(𝑠2+𝑎2)2 (𝑠2+𝑎2) (𝑠2+𝑎2)
𝑠
]
1 𝑠
= 𝐿−1 [ ] ∗ 𝐿−1 [ ]
(𝑠2+𝑎2) (𝑠2+𝑎2)
𝑎
= 1
𝑠𝑖𝑛𝑎𝑡 ∗ 𝑐𝑜𝑠𝑎𝑡
= 1 𝑡
∫ 𝑠𝑖𝑛𝑎𝑢 𝑐𝑜𝑠𝑎(𝑡 − 𝑢)𝑑𝑢
𝑎 0
∫
𝑎 0
= 1 𝑡 sin(𝑎𝑢+𝑎𝑡−𝑎𝑢)+sin(𝑎𝑢−𝑎𝑡+𝑎𝑢)
𝑑𝑢
= 1
2
𝑡
∫ [𝑠𝑖𝑛𝑎𝑡 + sin(2𝑎𝑢 − 𝑎𝑡)]𝑑𝑢
2𝑎 0
= 1
[∫
𝑡
𝑠𝑖𝑛𝑎𝑡𝑑𝑢 +
2𝑎 0
𝑡
∫0 sin(2𝑎𝑢 − 𝑎𝑡) 𝑑𝑢]
2𝑎
= 1
[𝑠𝑖𝑛𝑎𝑡 ∫
0
𝑡
𝑑𝑢 + ∫0
𝑡
sin(2𝑎𝑢 − 𝑎𝑡) 𝑑𝑢]
1
0
𝑡
= [𝑠𝑖𝑛𝑎𝑡(𝑢) − (
cos(2𝑎𝑢−𝑎𝑡)
2𝑎 2𝑎
𝑡
) ]
0
= 1
[𝑡𝑠𝑖𝑛𝑎𝑡 − cos(2𝑎𝑡−𝑎𝑡)
+ 𝑐𝑜𝑠𝑎𝑡
] 2
𝑎 2𝑎 2𝑎
= 1
[𝑡𝑠𝑖𝑛𝑎𝑡 − cos 𝑎𝑡
+ 𝑐𝑜𝑠𝑎𝑡
] 2𝑎 2𝑎 2𝑎
= 1
2𝑎
𝑡𝑠𝑖𝑛𝑎𝑡
𝑠 𝑡𝑠𝑖𝑛𝑎𝑡
∴ 𝐿−1 [ ] =
(𝑠2+𝑎2)2 2𝑎
Example: 5.60 Find 𝑳−𝟏 [ 𝟏
] by using convolution theorem.
(𝒔𝟐+𝒂𝟐)
𝟐
Solution:
1 1
𝐿−1 [ ] = 𝐿−1 [
1
(𝑠2+𝑎2)2 (𝑠2+𝑎2) (𝑠2+𝑎2)
]
1 1
= 𝐿−1 [ ] ∗ 𝐿−1 [ ]
(𝑠2+𝑎2) (𝑠2+𝑎2)
𝑎
= 1
𝑠𝑖𝑛𝑎𝑡 ∗ 1
𝑠𝑖𝑛𝑎𝑡
𝑎
= 1 𝑡
𝑠𝑖𝑛𝑎𝑢 𝑠𝑖𝑛𝑎(𝑡 − 𝑢)𝑑𝑢
𝑎2 ∫0
𝑎2 ∫0
= 1 𝑡 cos(𝑎𝑢−𝑎𝑡+𝑎𝑢)−cos(𝑎𝑢+𝑎𝑡−𝑎𝑢)
𝑑𝑢
=
1
2
𝑡
[cos(2𝑎𝑢 − 𝑎𝑡) − 𝑐𝑜𝑠𝑎𝑡]𝑑𝑢
2𝑎2 ∫0
1
2𝑎2 ∫0
𝑡
= [ cos(2𝑎𝑢 − 𝑎𝑡)𝑑𝑢 − ∫0
𝑡
𝑐𝑜𝑠𝑎𝑡 𝑑𝑢]
1
2𝑎2 ∫0
𝑡
= [ cos(2𝑎𝑢 − 𝑎𝑡)𝑑𝑢 − 𝑐𝑜𝑠𝑎𝑡 ∫0
𝑡
𝑑𝑢]
=
2𝑎2 2𝑎 0
𝑡
1
[(sin(2𝑎𝑢−𝑎𝑡)
0
)𝑡
) − 𝑐𝑜𝑠𝑎𝑡(𝑢 ]
=
1
[sin(2𝑎𝑡−𝑎𝑡)
− sin(−𝑎𝑡)
− 𝑡𝑐𝑜𝑠𝑎𝑡]
2𝑎2 2𝑎 2𝑎
Laplace Transform
Page51
=
1
[sin𝑎𝑡
+ 𝑠𝑖𝑛𝑎𝑡
− 𝑡𝑐𝑜𝑠𝑎𝑡]
2𝑎2 2𝑎 2𝑎
1
2𝑎2 2𝑎
= [2sin𝑎𝑡
− 𝑡𝑐𝑜𝑠𝑎𝑡]
(𝑠2+𝑎2)2 2𝑎2 𝑎
1 1 sin𝑎𝑡
∴ 𝐿−1 [ ] = [ − 𝑡𝑐𝑜𝑠𝑎𝑡]
Example: 5.61 Find 𝑳−𝟏 [
𝒔𝟐
(𝒔𝟐+𝒂𝟐)
𝟐] by using convolution theorem.
Solution:
𝐿−1 [
𝑠2
(𝑠2+𝑎2)2
𝑠 𝑠
] = 𝐿−1 [
(𝑠2+𝑎2) (𝑠2+𝑎2)
]
𝑠 𝑠
= 𝐿−1 [ ] ∗ 𝐿−1 [ ]
(𝑠2+𝑎2) (𝑠2+𝑎2)
= 𝑐𝑜𝑠𝑎𝑡 ∗ 𝑐𝑜𝑠𝑎𝑡
𝑡
𝑐𝑜𝑠𝑎𝑢 𝑐𝑜𝑠𝑎(𝑡 − 𝑢)𝑑𝑢
= ∫0
= ∫0
𝑡 cos(𝑎𝑢+𝑎𝑡−𝑎𝑢)+cos(𝑎𝑢−𝑎𝑡+𝑎𝑢)
𝑑𝑢
= 1
2
𝑡
∫ [𝑐𝑜𝑠𝑎𝑡 + cos(2𝑎𝑢 − 𝑎𝑡)]𝑑𝑢
2 0
∫
2 0
= 1
[
𝑡
𝑐𝑜𝑠𝑎𝑡𝑑𝑢 + ∫0
𝑡
cos(2𝑎𝑢 − 𝑎𝑡) 𝑑𝑢]
2
= 1
[𝑐𝑜𝑠𝑎𝑡 ∫
0
𝑡 𝑡
𝑑𝑢 + ∫0 cos(2𝑎𝑢 − 𝑎𝑡) 𝑑𝑢]
1
0
𝑡
= [𝑐𝑜𝑠𝑎𝑡(𝑢) + (
sin(2𝑎𝑢−𝑎𝑡)
2 2𝑎
𝑡
) ]
0
= 1
[𝑡𝑐𝑜𝑠𝑎𝑡 + sin(2𝑎𝑡−𝑎𝑡)
+ 𝑠𝑖𝑛𝑎𝑡
]
2 2𝑎 2𝑎
= 1
[𝑡𝑐𝑜𝑠𝑎𝑡 + sin𝑎𝑡
+ 𝑠𝑖𝑛𝑎𝑡
]
2 2𝑎 2𝑎
= 1
[𝑡𝑐𝑜𝑠𝑎𝑡 + 2sin𝑎𝑡
]
2 2𝑎
∴ 𝐿−1 [
𝑠2
(𝑠2+𝑎2)2 2 𝑎
] = 1
[𝑡𝑐𝑜𝑠𝑎𝑡 + sin𝑎𝑡
]
Example: 5.62 Find 𝑳−𝟏 [
𝒔𝟐
(𝒔𝟐+𝟒)
𝟐] by using convolution theorem.
Solution:
𝐿−1 [
𝑠2
] = 𝐿−1 [
𝑠 𝑠
(𝑠2+22)2 (𝑠2+22) (𝑠2+22)
]
𝑠 𝑠
= 𝐿−1 [ ] ∗ 𝐿−1 [ ]
(𝑠2+22) (𝑠2+22)
= 𝑐𝑜𝑠2𝑡 ∗ 𝑐𝑜𝑠2𝑡
𝑡
= ∫0𝑐𝑜𝑠2𝑢 𝑐𝑜𝑠2(𝑡 − 𝑢)𝑑𝑢
= ∫0
𝑡 cos(2𝑢+2𝑡−2𝑢)+cos(2𝑢−2𝑡+2𝑢)
𝑑𝑢
= 1
2
𝑡
[𝑐𝑜𝑠2𝑡 + cos(4𝑢 − 2𝑡)]𝑑𝑢
∫
2 0
∫
2 0
= 1
[
𝑡
𝑐𝑜𝑠2𝑡𝑑𝑢 + ∫0
𝑡
cos(4𝑢 − 2𝑡) 𝑑𝑢]
Engineering Maths - IV
Laplace Transform
Page52
2
= 1
[𝑐𝑜𝑠2𝑡 ∫
0
𝑡 𝑡
𝑑𝑢 + ∫0 cos(4𝑢 − 2𝑡) 𝑑𝑢]
1
0
𝑡
= [𝑐𝑜𝑠2𝑡(𝑢) + (
sin(4𝑢−2𝑡)
2 4
𝑡
) ]
0
= 1
[𝑡𝑐𝑜𝑠2𝑡 + sin(4𝑡−2𝑡)
− sin(−2𝑡)
]
2 4 4
= 1
[𝑡𝑐𝑜𝑠2𝑡 + sin2𝑡
+ 𝑠𝑖𝑛2𝑡
]
2 4 4
= 1
[𝑡𝑐𝑜𝑠2𝑡 + 2sin2𝑡
]
2 4
∴ 𝐿−1 [
𝑠2
(𝑠2+𝑎2)2 2 2
] = 1
[𝑡𝑐𝑜𝑠2𝑡 + sin2𝑡
]
Example: 5.63 Find 𝑳−𝟏 [
𝟏
𝒔(𝒔𝟐+𝟒)
] by using convolution theorem.
Solution:
1 1 1
𝐿−1 [ ] = 𝐿−1 [
𝑠(𝑠2+4) 𝑠 𝑠2+4
]
= 𝐿−1 [1
] ∗ 𝐿−1 [ 1
]
𝑠 𝑠2+4
= 1 ∗ sin2𝑡
2
= sin2𝑡
∗ 1
2
2
= ∫0
𝑡 sin2𝑢 (1)
𝑑𝑢
= [
−𝑐𝑜𝑠2
𝑢 4 0
𝑡 1
4
] = (−𝑐𝑜𝑠2𝑡 + 1)
4
= 1
(1 − 𝑐𝑜𝑠2𝑡)
Example: 5.64 Find the inverse Laplace transform
𝒔+𝟐
(𝒔𝟐+𝟒𝒔+𝟏𝟑)
𝟐 by using convolution theorem.
Solution:
𝑠+2 1
𝐿−1 [ 𝑠+2
] = 𝐿−1 [
(𝑠2+4𝑠+13)2 𝑠2+4𝑠+13 𝑠2+4𝑠+13
]
𝑠+2 1
= 𝐿−1 [ ] ∗ 𝐿−1 [ ]
𝑠2+4𝑠+13 𝑠2+4𝑠+13
= 𝐿−1 𝑠+2 1
[ ] ∗ 𝐿−1 [ ]
(𝑠+2)2+9 (𝑠+2)2+9
= 𝑒−2𝑡𝐿−1 𝑠 −2𝑡 −1 1
[ ] ∗ 𝑒 𝐿 [ ]
𝑠2+9 𝑠2+9
−2𝑡
= 𝑒−2𝑡𝑐𝑜𝑠3𝑡 ∗ 𝑒 𝑠𝑖𝑛3𝑡
3
= ∫0 3
𝑡 𝑠𝑖𝑛3(𝑡−𝑢)
𝑒−2𝑢𝑐𝑜𝑠3𝑢 𝑒−2(𝑡−𝑢) 𝑑𝑢
= ∫0
𝑡 𝑠𝑖𝑛(3𝑡−3𝑢)
𝑒−2𝑢𝑐𝑜𝑠3𝑢 𝑒−2𝑡+2𝑢 𝑑𝑢
= 1
3
𝑡
∫ 𝑒−2𝑢 −2𝑡+2𝑢 𝑐𝑜𝑠3𝑢 sin(3𝑡 − 3𝑢) 𝑑𝑢
3 0
3 2
∫0
−2𝑡
= 𝑒 𝑡 sin(3𝑢+3𝑡−3𝑢)−sin(3𝑢−3𝑡+3𝑢)
𝑑𝑢
Engineering Maths - IV
Laplace Transform
Page53
−2𝑡
= 𝑒
6
𝑡
∫
0
[𝑠𝑖𝑛3𝑡 − sin(6𝑢 − 3𝑡)]𝑑𝑢
6
−2𝑡
= 𝑒
[∫
𝑡
𝑠𝑖𝑛3𝑡𝑑𝑢 − ∫
0 0
𝑡
sin(6𝑢 − 3𝑡) 𝑑𝑢]
6
−2𝑡
= 𝑒
[𝑠𝑖𝑛3𝑡 ∫
0
𝑡 𝑡
𝑑𝑢 − ∫
0 sin(6𝑢 − 3𝑡) 𝑑𝑢]
𝑒−2𝑡
0
𝑡
= [𝑠𝑖𝑛3𝑡(𝑢) + (
cos(6𝑢 −3𝑡)
6 6
𝑡
) ]
0
6
−2𝑡
= 𝑒
[𝑡𝑠𝑖𝑛3𝑡 + cos(6𝑡−3𝑡)
− cos(−3𝑡)
]
6 6
−2𝑡
= 𝑒
[𝑡𝑠𝑖𝑛3𝑡 + cos3𝑡
− 𝑐𝑜𝑠3𝑡
]
6 6 6
6
−2𝑡
= 𝑒
𝑡𝑠𝑖𝑛3𝑡
𝑠+2 𝑒−2𝑡
∴ 𝐿−1 [ ] = 𝑡𝑠𝑖𝑛3𝑡
(𝑠2+4𝑠+13)2 6
Example: 5.65 Find the inverse Laplace transform
𝟏
(𝒔+𝟏)(𝒔𝟐+𝟒)
by using convolution theorem.
Solution:
𝐿−1 [ 1
(𝑠2+4)(𝑠+1)
] = 𝐿−1 [ 1
𝑠+1 𝑠2+4
1
]
1 1
= 𝐿−1 [ ] ∗ 𝐿−1 [ ]
𝑠+1 𝑠2+4
= 𝑒−𝑡 ∗ 𝑐𝑜𝑠2𝑡
𝑡
𝑒−(𝑡−𝑢)𝑐𝑜𝑠2𝑢 𝑑𝑢
= ∫0
−𝑡 𝑢
𝑡
= 𝑒 ∫0𝑒 𝑐𝑜𝑠2𝑢 𝑑𝑢
−𝑡 [
= 𝑒 (𝑐𝑜𝑠2𝑢 + 2𝑠𝑖𝑛2𝑢)]
12+22
0
𝑒𝑢 𝑡
5
−𝑡
= 𝑒
[𝑒𝑡(𝑐𝑜𝑠2𝑡 + 2𝑠𝑖𝑛2𝑡) − 𝑒0(𝑐𝑜𝑠0 − 0)]
5
−𝑡
= 𝑒
[𝑒𝑡(𝑐𝑜𝑠2𝑡 + 2𝑠𝑖𝑛2𝑡) − 1]
∴ 𝐿−1 [
1
(𝑠2+4)(𝑠+1) 5
−𝑡
] = 𝑒
[𝑒𝑡(𝑐𝑜𝑠2𝑡 + 2𝑠𝑖𝑛2𝑡) − 1]
Exercise: 5.10
Find the inverse Laplace transforms using convolution theorem for the following
1.
1
Ans: 1 − 𝑐𝑜𝑠𝑡
2.
𝑠(𝑠2+1)
𝑠
8 2
Ans: 1
[sin2𝑡
− 𝑡𝑐𝑜𝑠2𝑡]
3.
(𝑠2+4)2
𝑠2
Ans: 1
[𝑡𝑐𝑜𝑠2𝑡 + sin2𝑡
]
2 2
4.
(𝑠2+4)2
1
2
Ans: 1
[𝑒−𝑡 + 𝑠𝑖𝑛𝑡 − 𝑐𝑜𝑠𝑡]
5.
(𝑠+1)(𝑠2+1)
1
(𝑠+1)(𝑠2+4)
Ans: − 1
𝑒−𝑡 + 1
𝑐𝑜𝑠2𝑡 − 1
𝑠𝑖𝑛2𝑡
5 5 10
∵ ∫ 𝑒𝑎𝑡 𝑐𝑜𝑠𝑏𝑡𝑑𝑡 =
𝑒𝑎𝑡
𝑎2 + 𝑏2
(𝑎𝑐𝑜𝑠𝑏𝑡 + 𝑏𝑠𝑖𝑛𝑏𝑡)
Engineering Maths - IV
Laplace Transform
Page54
Laplace & Inverse Transform convoltuion them.pptx
Laplace & Inverse Transform convoltuion them.pptx
Laplace & Inverse Transform convoltuion them.pptx
Laplace & Inverse Transform convoltuion them.pptx
Laplace & Inverse Transform convoltuion them.pptx
Laplace & Inverse Transform convoltuion them.pptx
Laplace & Inverse Transform convoltuion them.pptx
Laplace & Inverse Transform convoltuion them.pptx
Laplace & Inverse Transform convoltuion them.pptx
Laplace & Inverse Transform convoltuion them.pptx
Laplace & Inverse Transform convoltuion them.pptx
Laplace & Inverse Transform convoltuion them.pptx

More Related Content

Similar to Laplace & Inverse Transform convoltuion them.pptx

BSC_Computer Science_Discrete Mathematics_Unit-I
BSC_Computer Science_Discrete Mathematics_Unit-IBSC_Computer Science_Discrete Mathematics_Unit-I
BSC_Computer Science_Discrete Mathematics_Unit-I
Rai University
 
BSC_COMPUTER _SCIENCE_UNIT-1_DISCRETE MATHEMATICS
BSC_COMPUTER _SCIENCE_UNIT-1_DISCRETE MATHEMATICSBSC_COMPUTER _SCIENCE_UNIT-1_DISCRETE MATHEMATICS
BSC_COMPUTER _SCIENCE_UNIT-1_DISCRETE MATHEMATICS
Rai University
 
Analysis of a self-sustained vibration of mass-spring oscillator on moving belt
Analysis of a self-sustained vibration of mass-spring oscillator on moving beltAnalysis of a self-sustained vibration of mass-spring oscillator on moving belt
Analysis of a self-sustained vibration of mass-spring oscillator on moving belt
Varun Jadhav
 
Hawkinrad a source_notes iii -sqrd
Hawkinrad a source_notes iii -sqrdHawkinrad a source_notes iii -sqrd
Hawkinrad a source_notes iii -sqrd
foxtrot jp R
 
BSC_COMPUTER _SCIENCE_UNIT-2_DISCRETE MATHEMATICS
BSC_COMPUTER _SCIENCE_UNIT-2_DISCRETE MATHEMATICSBSC_COMPUTER _SCIENCE_UNIT-2_DISCRETE MATHEMATICS
BSC_COMPUTER _SCIENCE_UNIT-2_DISCRETE MATHEMATICS
Rai University
 
A Non Local Boundary Value Problem with Integral Boundary Condition
A Non Local Boundary Value Problem with Integral Boundary ConditionA Non Local Boundary Value Problem with Integral Boundary Condition
A Non Local Boundary Value Problem with Integral Boundary Condition
IJMERJOURNAL
 
Lecture5_Laplace_ODE.pdf
Lecture5_Laplace_ODE.pdfLecture5_Laplace_ODE.pdf
Lecture5_Laplace_ODE.pdf
MohammedKhodary4
 
PRODUCT RULES
PRODUCT RULESPRODUCT RULES
PRODUCT RULES
NumanUsama
 
Laurents & Taylors series of complex numbers.pptx
Laurents & Taylors series of complex numbers.pptxLaurents & Taylors series of complex numbers.pptx
Laurents & Taylors series of complex numbers.pptx
jyotidighole2
 
Laplace transformation
Laplace transformationLaplace transformation
Laplace transformation
Santhanam Krishnan
 
Sbe final exam jan17 - solved-converted
Sbe final exam jan17 - solved-convertedSbe final exam jan17 - solved-converted
Sbe final exam jan17 - solved-converted
cairo university
 
publisher in research
publisher in researchpublisher in research
publisher in research
rikaseorika
 
Matrix Transformations on Some Difference Sequence Spaces
Matrix Transformations on Some Difference Sequence SpacesMatrix Transformations on Some Difference Sequence Spaces
Matrix Transformations on Some Difference Sequence Spaces
IOSR Journals
 
Differential Calculus- differentiation
Differential Calculus- differentiationDifferential Calculus- differentiation
Differential Calculus- differentiation
Santhanam Krishnan
 
Hawkinrad a source_notes ii _secured
Hawkinrad a source_notes ii _securedHawkinrad a source_notes ii _secured
Hawkinrad a source_notes ii _secured
foxtrot jp R
 
Laplace Transform and its applications
Laplace Transform and its applicationsLaplace Transform and its applications
Laplace Transform and its applications
DeepRaval7
 
Btech_II_ engineering mathematics_unit4
Btech_II_ engineering mathematics_unit4Btech_II_ engineering mathematics_unit4
Btech_II_ engineering mathematics_unit4
Rai University
 
On Bernstein Polynomials
On Bernstein PolynomialsOn Bernstein Polynomials
On Bernstein Polynomials
IOSR Journals
 
B.tech ii unit-5 material vector integration
B.tech ii unit-5 material vector integrationB.tech ii unit-5 material vector integration
B.tech ii unit-5 material vector integration
Rai University
 

Similar to Laplace & Inverse Transform convoltuion them.pptx (20)

BSC_Computer Science_Discrete Mathematics_Unit-I
BSC_Computer Science_Discrete Mathematics_Unit-IBSC_Computer Science_Discrete Mathematics_Unit-I
BSC_Computer Science_Discrete Mathematics_Unit-I
 
BSC_COMPUTER _SCIENCE_UNIT-1_DISCRETE MATHEMATICS
BSC_COMPUTER _SCIENCE_UNIT-1_DISCRETE MATHEMATICSBSC_COMPUTER _SCIENCE_UNIT-1_DISCRETE MATHEMATICS
BSC_COMPUTER _SCIENCE_UNIT-1_DISCRETE MATHEMATICS
 
Analysis of a self-sustained vibration of mass-spring oscillator on moving belt
Analysis of a self-sustained vibration of mass-spring oscillator on moving beltAnalysis of a self-sustained vibration of mass-spring oscillator on moving belt
Analysis of a self-sustained vibration of mass-spring oscillator on moving belt
 
Hawkinrad a source_notes iii -sqrd
Hawkinrad a source_notes iii -sqrdHawkinrad a source_notes iii -sqrd
Hawkinrad a source_notes iii -sqrd
 
BSC_COMPUTER _SCIENCE_UNIT-2_DISCRETE MATHEMATICS
BSC_COMPUTER _SCIENCE_UNIT-2_DISCRETE MATHEMATICSBSC_COMPUTER _SCIENCE_UNIT-2_DISCRETE MATHEMATICS
BSC_COMPUTER _SCIENCE_UNIT-2_DISCRETE MATHEMATICS
 
A Non Local Boundary Value Problem with Integral Boundary Condition
A Non Local Boundary Value Problem with Integral Boundary ConditionA Non Local Boundary Value Problem with Integral Boundary Condition
A Non Local Boundary Value Problem with Integral Boundary Condition
 
Lecture5_Laplace_ODE.pdf
Lecture5_Laplace_ODE.pdfLecture5_Laplace_ODE.pdf
Lecture5_Laplace_ODE.pdf
 
PRODUCT RULES
PRODUCT RULESPRODUCT RULES
PRODUCT RULES
 
Laurents & Taylors series of complex numbers.pptx
Laurents & Taylors series of complex numbers.pptxLaurents & Taylors series of complex numbers.pptx
Laurents & Taylors series of complex numbers.pptx
 
Laplace transformation
Laplace transformationLaplace transformation
Laplace transformation
 
Sbe final exam jan17 - solved-converted
Sbe final exam jan17 - solved-convertedSbe final exam jan17 - solved-converted
Sbe final exam jan17 - solved-converted
 
publisher in research
publisher in researchpublisher in research
publisher in research
 
Matrix Transformations on Some Difference Sequence Spaces
Matrix Transformations on Some Difference Sequence SpacesMatrix Transformations on Some Difference Sequence Spaces
Matrix Transformations on Some Difference Sequence Spaces
 
Differential Calculus- differentiation
Differential Calculus- differentiationDifferential Calculus- differentiation
Differential Calculus- differentiation
 
Hawkinrad a source_notes ii _secured
Hawkinrad a source_notes ii _securedHawkinrad a source_notes ii _secured
Hawkinrad a source_notes ii _secured
 
Laplace Transform and its applications
Laplace Transform and its applicationsLaplace Transform and its applications
Laplace Transform and its applications
 
Btech_II_ engineering mathematics_unit4
Btech_II_ engineering mathematics_unit4Btech_II_ engineering mathematics_unit4
Btech_II_ engineering mathematics_unit4
 
new math seminar paper
new math seminar papernew math seminar paper
new math seminar paper
 
On Bernstein Polynomials
On Bernstein PolynomialsOn Bernstein Polynomials
On Bernstein Polynomials
 
B.tech ii unit-5 material vector integration
B.tech ii unit-5 material vector integrationB.tech ii unit-5 material vector integration
B.tech ii unit-5 material vector integration
 

Recently uploaded

The Benefits and Techniques of Trenchless Pipe Repair.pdf
The Benefits and Techniques of Trenchless Pipe Repair.pdfThe Benefits and Techniques of Trenchless Pipe Repair.pdf
The Benefits and Techniques of Trenchless Pipe Repair.pdf
Pipe Restoration Solutions
 
Event Management System Vb Net Project Report.pdf
Event Management System Vb Net  Project Report.pdfEvent Management System Vb Net  Project Report.pdf
Event Management System Vb Net Project Report.pdf
Kamal Acharya
 
Gen AI Study Jams _ For the GDSC Leads in India.pdf
Gen AI Study Jams _ For the GDSC Leads in India.pdfGen AI Study Jams _ For the GDSC Leads in India.pdf
Gen AI Study Jams _ For the GDSC Leads in India.pdf
gdsczhcet
 
Quality defects in TMT Bars, Possible causes and Potential Solutions.
Quality defects in TMT Bars, Possible causes and Potential Solutions.Quality defects in TMT Bars, Possible causes and Potential Solutions.
Quality defects in TMT Bars, Possible causes and Potential Solutions.
PrashantGoswami42
 
Student information management system project report ii.pdf
Student information management system project report ii.pdfStudent information management system project report ii.pdf
Student information management system project report ii.pdf
Kamal Acharya
 
WATER CRISIS and its solutions-pptx 1234
WATER CRISIS and its solutions-pptx 1234WATER CRISIS and its solutions-pptx 1234
WATER CRISIS and its solutions-pptx 1234
AafreenAbuthahir2
 
Industrial Training at Shahjalal Fertilizer Company Limited (SFCL)
Industrial Training at Shahjalal Fertilizer Company Limited (SFCL)Industrial Training at Shahjalal Fertilizer Company Limited (SFCL)
Industrial Training at Shahjalal Fertilizer Company Limited (SFCL)
MdTanvirMahtab2
 
Vaccine management system project report documentation..pdf
Vaccine management system project report documentation..pdfVaccine management system project report documentation..pdf
Vaccine management system project report documentation..pdf
Kamal Acharya
 
Planning Of Procurement o different goods and services
Planning Of Procurement o different goods and servicesPlanning Of Procurement o different goods and services
Planning Of Procurement o different goods and services
JoytuBarua2
 
Nuclear Power Economics and Structuring 2024
Nuclear Power Economics and Structuring 2024Nuclear Power Economics and Structuring 2024
Nuclear Power Economics and Structuring 2024
Massimo Talia
 
J.Yang, ICLR 2024, MLILAB, KAIST AI.pdf
J.Yang,  ICLR 2024, MLILAB, KAIST AI.pdfJ.Yang,  ICLR 2024, MLILAB, KAIST AI.pdf
J.Yang, ICLR 2024, MLILAB, KAIST AI.pdf
MLILAB
 
power quality voltage fluctuation UNIT - I.pptx
power quality voltage fluctuation UNIT - I.pptxpower quality voltage fluctuation UNIT - I.pptx
power quality voltage fluctuation UNIT - I.pptx
ViniHema
 
LIGA(E)11111111111111111111111111111111111111111.ppt
LIGA(E)11111111111111111111111111111111111111111.pptLIGA(E)11111111111111111111111111111111111111111.ppt
LIGA(E)11111111111111111111111111111111111111111.ppt
ssuser9bd3ba
 
Standard Reomte Control Interface - Neometrix
Standard Reomte Control Interface - NeometrixStandard Reomte Control Interface - Neometrix
Standard Reomte Control Interface - Neometrix
Neometrix_Engineering_Pvt_Ltd
 
Halogenation process of chemical process industries
Halogenation process of chemical process industriesHalogenation process of chemical process industries
Halogenation process of chemical process industries
MuhammadTufail242431
 
AKS UNIVERSITY Satna Final Year Project By OM Hardaha.pdf
AKS UNIVERSITY Satna Final Year Project By OM Hardaha.pdfAKS UNIVERSITY Satna Final Year Project By OM Hardaha.pdf
AKS UNIVERSITY Satna Final Year Project By OM Hardaha.pdf
SamSarthak3
 
COLLEGE BUS MANAGEMENT SYSTEM PROJECT REPORT.pdf
COLLEGE BUS MANAGEMENT SYSTEM PROJECT REPORT.pdfCOLLEGE BUS MANAGEMENT SYSTEM PROJECT REPORT.pdf
COLLEGE BUS MANAGEMENT SYSTEM PROJECT REPORT.pdf
Kamal Acharya
 
CFD Simulation of By-pass Flow in a HRSG module by R&R Consult.pptx
CFD Simulation of By-pass Flow in a HRSG module by R&R Consult.pptxCFD Simulation of By-pass Flow in a HRSG module by R&R Consult.pptx
CFD Simulation of By-pass Flow in a HRSG module by R&R Consult.pptx
R&R Consult
 
MCQ Soil mechanics questions (Soil shear strength).pdf
MCQ Soil mechanics questions (Soil shear strength).pdfMCQ Soil mechanics questions (Soil shear strength).pdf
MCQ Soil mechanics questions (Soil shear strength).pdf
Osamah Alsalih
 
Design and Analysis of Algorithms-DP,Backtracking,Graphs,B&B
Design and Analysis of Algorithms-DP,Backtracking,Graphs,B&BDesign and Analysis of Algorithms-DP,Backtracking,Graphs,B&B
Design and Analysis of Algorithms-DP,Backtracking,Graphs,B&B
Sreedhar Chowdam
 

Recently uploaded (20)

The Benefits and Techniques of Trenchless Pipe Repair.pdf
The Benefits and Techniques of Trenchless Pipe Repair.pdfThe Benefits and Techniques of Trenchless Pipe Repair.pdf
The Benefits and Techniques of Trenchless Pipe Repair.pdf
 
Event Management System Vb Net Project Report.pdf
Event Management System Vb Net  Project Report.pdfEvent Management System Vb Net  Project Report.pdf
Event Management System Vb Net Project Report.pdf
 
Gen AI Study Jams _ For the GDSC Leads in India.pdf
Gen AI Study Jams _ For the GDSC Leads in India.pdfGen AI Study Jams _ For the GDSC Leads in India.pdf
Gen AI Study Jams _ For the GDSC Leads in India.pdf
 
Quality defects in TMT Bars, Possible causes and Potential Solutions.
Quality defects in TMT Bars, Possible causes and Potential Solutions.Quality defects in TMT Bars, Possible causes and Potential Solutions.
Quality defects in TMT Bars, Possible causes and Potential Solutions.
 
Student information management system project report ii.pdf
Student information management system project report ii.pdfStudent information management system project report ii.pdf
Student information management system project report ii.pdf
 
WATER CRISIS and its solutions-pptx 1234
WATER CRISIS and its solutions-pptx 1234WATER CRISIS and its solutions-pptx 1234
WATER CRISIS and its solutions-pptx 1234
 
Industrial Training at Shahjalal Fertilizer Company Limited (SFCL)
Industrial Training at Shahjalal Fertilizer Company Limited (SFCL)Industrial Training at Shahjalal Fertilizer Company Limited (SFCL)
Industrial Training at Shahjalal Fertilizer Company Limited (SFCL)
 
Vaccine management system project report documentation..pdf
Vaccine management system project report documentation..pdfVaccine management system project report documentation..pdf
Vaccine management system project report documentation..pdf
 
Planning Of Procurement o different goods and services
Planning Of Procurement o different goods and servicesPlanning Of Procurement o different goods and services
Planning Of Procurement o different goods and services
 
Nuclear Power Economics and Structuring 2024
Nuclear Power Economics and Structuring 2024Nuclear Power Economics and Structuring 2024
Nuclear Power Economics and Structuring 2024
 
J.Yang, ICLR 2024, MLILAB, KAIST AI.pdf
J.Yang,  ICLR 2024, MLILAB, KAIST AI.pdfJ.Yang,  ICLR 2024, MLILAB, KAIST AI.pdf
J.Yang, ICLR 2024, MLILAB, KAIST AI.pdf
 
power quality voltage fluctuation UNIT - I.pptx
power quality voltage fluctuation UNIT - I.pptxpower quality voltage fluctuation UNIT - I.pptx
power quality voltage fluctuation UNIT - I.pptx
 
LIGA(E)11111111111111111111111111111111111111111.ppt
LIGA(E)11111111111111111111111111111111111111111.pptLIGA(E)11111111111111111111111111111111111111111.ppt
LIGA(E)11111111111111111111111111111111111111111.ppt
 
Standard Reomte Control Interface - Neometrix
Standard Reomte Control Interface - NeometrixStandard Reomte Control Interface - Neometrix
Standard Reomte Control Interface - Neometrix
 
Halogenation process of chemical process industries
Halogenation process of chemical process industriesHalogenation process of chemical process industries
Halogenation process of chemical process industries
 
AKS UNIVERSITY Satna Final Year Project By OM Hardaha.pdf
AKS UNIVERSITY Satna Final Year Project By OM Hardaha.pdfAKS UNIVERSITY Satna Final Year Project By OM Hardaha.pdf
AKS UNIVERSITY Satna Final Year Project By OM Hardaha.pdf
 
COLLEGE BUS MANAGEMENT SYSTEM PROJECT REPORT.pdf
COLLEGE BUS MANAGEMENT SYSTEM PROJECT REPORT.pdfCOLLEGE BUS MANAGEMENT SYSTEM PROJECT REPORT.pdf
COLLEGE BUS MANAGEMENT SYSTEM PROJECT REPORT.pdf
 
CFD Simulation of By-pass Flow in a HRSG module by R&R Consult.pptx
CFD Simulation of By-pass Flow in a HRSG module by R&R Consult.pptxCFD Simulation of By-pass Flow in a HRSG module by R&R Consult.pptx
CFD Simulation of By-pass Flow in a HRSG module by R&R Consult.pptx
 
MCQ Soil mechanics questions (Soil shear strength).pdf
MCQ Soil mechanics questions (Soil shear strength).pdfMCQ Soil mechanics questions (Soil shear strength).pdf
MCQ Soil mechanics questions (Soil shear strength).pdf
 
Design and Analysis of Algorithms-DP,Backtracking,Graphs,B&B
Design and Analysis of Algorithms-DP,Backtracking,Graphs,B&BDesign and Analysis of Algorithms-DP,Backtracking,Graphs,B&B
Design and Analysis of Algorithms-DP,Backtracking,Graphs,B&B
 

Laplace & Inverse Transform convoltuion them.pptx

  • 1. ENGINEERING MATHEMATICS -IV SUBJECT CODE: BSH251 (Regulation 2016-17) Common to all branches of B.E UNIT-I LAPLACE TRANSFORM Engineering Maths - IV Laplace Transform Page 1
  • 2. Engineering Maths - IV UNIT-V LAPLACE TRANSFORM 1. INTRODUCTION A transformation is an operation which converts a mathematical expression to a different but equivalent form. The well known transformation logarithms reduce multiplication and division to a simpler process of addition subtraction. The Laplace transform is a powerful mathematical technique which solves linear equations with given initial conditions by using algebra methods. The Laplace transform can also be used to solve systems of differential equations, Partial differential equations and integral equations. In this chapter, we will discuss about the definition, properties of Laplace transform and derive the transforms of some functions which usually occur in the solution of linear differential equations. 2. LAPLACE TRANSFORM Let 𝑓(𝑡) be a function of t defined for all 𝑡 ≥ 0 .then the Laplace transform of𝑓(𝑡), denoted by 𝐿[ 𝑓(𝑡)] is defined by 𝐿[ 𝑓(𝑡)] = ∫0 ∞ 𝑒−𝑠𝑡 𝑓(𝑡)𝑑𝑡 𝑡→∞ Provided that the integral exists, “s” is a parameter which may be real or complex. Clearly 𝐿[ 𝑓(𝑡)]is a function of s and is briefly written as 𝐹(𝑠) (𝑖. 𝑒. ) 𝐿[ 𝑓(𝑡)] = 𝐹(𝑠) Piecewise continuous function A function 𝑓(𝑡) is said to be piecewise continuous is an interval 𝑎 ≤ 𝑡 ≤ 𝑏, if the interval can be sub divided into a finite number of intervals in each of which the function is continuous and has finite right and left hand limits. Exponential order A function 𝑓(𝑡) is said to be exponential order if lim 𝑒−𝑠𝑡𝑓(𝑡) is a finite quantity, where 𝑠 > 0(exists). Example: 5. 1 Show that the function 𝒇(𝒕) = 𝒆𝒕𝟑 is not of exponential order. Solution: lim 𝑒−𝑠𝑡 𝑒𝑡3 =lim 𝑒−𝑠𝑡+𝑡3 = lim 𝑒𝑡3−𝑠𝑡 𝑡→∞ 𝑡→∞ 𝑡→∞ = 𝑒∞ = ∞, not a finite quantity. Hence 𝑓(𝑡) = 𝑒𝑡3 is not of exponential order. Sufficient conditions for the existence of the Laplace transform The Laplace transform of 𝑓(𝑡) exists if i) ii) 𝑓(𝑡) is piecewise continuous in the interval 𝑎 ≤ 𝑡 ≤ 𝑏 𝑓(𝑡) is of exponential order. Laplace Transform Page 2
  • 3. Engineering Maths - IV Note: The above conditions are only sufficient conditions and not a necessary condition. Example: 5.2 Prove that Laplace transform of 𝒆𝒕𝟐 does not exist. Solution: lim 𝑒−𝑠𝑡 𝑒𝑡2 =lim 𝑒−𝑠𝑡+𝑡2 = lim 𝑒𝑡2−𝑠𝑡 𝑡→∞ 𝑡→∞ 𝑡→∞ = 𝑒∞ = ∞ ,not a finite quantity. ∴ 𝑒𝑡2 is not of exponential order. Hence Laplace transform of 𝑒𝑡2 does not exist. 5.3 PROPERTIES OF LAPLACE TRANSFORM Property: 1 Linear property 𝑳[𝒂𝒇(𝒕) ± 𝒃𝒈(𝒕)] = 𝒂𝑳[𝒇(𝒕)] ± 𝒃𝑳[𝒈(𝒕)] , where a and b are constants. Proof: 𝐿[𝑎𝑓(𝑡) ± 𝑏𝑔(𝑡)] = ∫ ∞ [𝑎𝑓(𝑡) ± 𝑏𝑔(𝑡)] 𝑒−𝑠𝑡 𝑑𝑡 0 =𝑎 ∫0 ∞ 𝑔(𝑡) 𝑒−𝑠𝑡𝑑𝑡 ∞ 𝑓(𝑡)𝑒−𝑠𝑡𝑑𝑡 ± 𝑏 ∫ 0 𝐿[𝑎𝑓(𝑡) ± 𝑏𝑔(𝑡)] = 𝑎 𝐿[𝑓(𝑡)] ± 𝑏 𝐿[𝑔(𝑡)] Property: 2 Change of scale property. 𝒂 𝒂 If 𝑳[𝒇(𝒕)] = 𝑭(𝒔), then 𝑳[𝒇(𝒂𝒕)] = 𝟏 𝑭 (𝒔 ) ;𝒂 > 0 Proof: Given 𝐿[𝑓(𝑡)] = 𝐹(𝑠) ∞ 𝑒−𝑠𝑡 𝑓(𝑡) 𝑑𝑡 = 𝐹(𝑠) ⋯ ⋯ (1) ∴ ∫0 By the definition of Laplace transform, we have 𝐿[𝑓(𝑎𝑡)] = ∫0 ∞ 𝑒−𝑠𝑡 𝑓(𝑎𝑡) 𝑑𝑡 ⋯ ⋯ (2) Put at= 𝑥 𝑖𝑒. , 𝑡 = 𝒙 ⇒ 𝑑𝑡 = 𝑑𝑥 𝒂 𝑎 −𝒔𝒙 ∞ 𝑑𝑥 (2) ⇒ 𝐿[𝑓(𝑎𝑡)] = ∫0 𝑒 𝒂 𝑓(𝑥) 𝑎 −𝒔𝒙 𝟏 ∞ = ∫ 𝑒 𝒂 𝑓(𝑥)𝑑𝑥 𝒂 0 Replace 𝑥 by t, −𝑠 𝑡 1 ∞ 𝐿[𝑓(𝑎𝑡)] = ∫ 𝑒 𝑎 𝑓(𝑡)𝑑𝑡 𝑎 0 𝟏 𝒔 𝒂 𝒂 𝑳[𝒇(𝒂𝒕)] = 𝑭 ( ) ;𝒂 > 0 Property: 3 First shifting property. 𝑳[𝒆−𝒂𝒕𝒇(𝒕)] = 𝑭(𝒔 + 𝒂) 𝑳[𝒆𝒂𝒕𝒇(𝒕)] = 𝑭(𝒔 − 𝒂) If 𝑳[𝒇(𝒕)] = 𝑭(𝒔), then i) ii) Proof: (i) 𝐿[𝑒−𝑎𝑡𝑓(𝑡)] = 𝐹(𝑠 + 𝑎) Laplace Transform Page 3
  • 4. Given 𝐿[𝑓(𝑡)] = 𝐹(𝑠) ∞ 𝑒−𝑠𝑡 𝑓(𝑡) 𝑑𝑡 = 𝐹(𝑠) ⋯ (1) ∴ ∫0 By the definition of Laplace transform, we have ∞ 𝑒−𝑠𝑡 𝑒−𝑎𝑡𝑓(𝑡) 𝑑𝑡 𝐿[𝑒−𝑎𝑡𝑓(𝑎𝑡)] = ∫ 0 ∞ 𝑒−(𝑠+𝑎)𝑡 𝑓(𝑡) 𝑑𝑡 = ∫0 = 𝐹(𝑠 + 𝑎) by (1) ∞ 𝑒−𝑠𝑡 𝑒𝑎𝑡𝑓(𝑡) 𝑑𝑡 (ii) 𝐿[𝑒𝑎𝑡𝑓(𝑎𝑡)] = ∫ 0 ∞ 𝑒−(𝑠−𝑎)𝑡 𝑓(𝑡) 𝑑𝑡 = ∫0 = 𝐹(𝑠 − 𝑎) by (1) Property: 4 Laplace transforms of derivatives 𝑳[𝒇′(𝒕)] = 𝒔𝑳[𝒇(𝒕)] − 𝒇(𝟎) Proof: ∞ ∞ 𝐿[𝑓′(𝑡)] = ∫ 𝑒−𝑠𝑡 𝑓′(𝑡) 𝑑𝑡 = ∫ 𝑢𝑑𝑣 0 0 0 = [𝑢𝑣]∞ − ∫ 𝑢𝑑𝑣 0 ∞ 𝑓(𝑡) (−𝑠)𝑒−𝑠𝑡 𝑑𝑡 = [𝑒−𝑠𝑡 𝑓(𝑡)]∞ − ∫ 0 = 0 − 𝑓(0) + 𝑠𝐿[𝑓(𝑡)] = 𝑠𝐿[𝑓(𝑡)] − 𝑓(0) 𝑳[𝒇′(𝒕)] = 𝒔𝑳[𝒇(𝒕)] − 𝒇(𝟎) Property: 5 Laplace transform of derivative of order n 𝑳[𝒇𝒏(𝒕)] = 𝒔𝒏𝑳[𝒇(𝒕)] − 𝒔𝒏−𝟏 𝒇(𝟎) − 𝒔𝒏−𝟐𝒇′(𝟎)⋯ − 𝒔𝒏−𝟑 𝒇′′(𝟎) − ⋯ 𝒇𝒏−𝟏 (𝟎) Proof: We know that 𝐿[𝑓′(𝑡)] = 𝑠𝐿[𝑓(𝑡)] − 𝑓(0)⋯ ⋯ (1) 𝐿[𝑓𝑛(𝑡)] = 𝐿[[𝑓′(𝑡)]′] = 𝑠𝐿[𝑓′(𝑡)] − 𝑓′(0) = 𝑠[𝑠𝐿[𝑓(𝑡)] − 𝑓(0)] − 𝑓′(0) = 𝑠2𝐿[𝑓(𝑡)] − 𝑠𝑓(0) − 𝑓′(0) Similarly, 𝐿[𝑓′′′(𝑡)] = 𝑠3𝐿[𝑓(𝑡)] − 𝑠2𝑓(0) − 𝑠𝑓′(0) − 𝑓′′(0) In general, 𝑳[𝒇𝒏(𝒕)] = 𝒔𝒏𝑳[𝒇(𝒕)] − 𝒔𝒏−𝟏 𝒇(𝟎) − 𝒔𝒏−𝟐𝒇′(𝟎) ⋯ − 𝒔𝒏−𝟑 𝒇′′(𝟎) − ⋯ 𝒇𝒏−𝟏 (𝟎) Laplace transform of integrals 𝒕 𝑭(𝒔) ∫ 𝟎 𝒇(𝒕)𝒅𝒕] = 𝒔 Theorem: 1 If𝑳[𝒇(𝒕)] = 𝑭(𝒔), then 𝑳 [ Proof: 𝑡 𝑓(𝑡)𝑑𝑡 Let 𝑔(𝑡) = ∫0 ∴ 𝑔′(𝑡) = 𝑓(𝑡) 0 𝑓(𝑡)𝑑𝑡 = 0 And 𝑔(0) = ∫0 𝑢 = 𝑒−𝑠𝑡 ∴ 𝑑𝑢 = −𝑠𝑒−𝑠𝑡𝑑𝑡 𝑑𝑣 = 𝑓′(𝑡)𝑑𝑡 ∴ 𝑣 = ∫ 𝑓′(𝑡)𝑑𝑡 = 𝑓(𝑡) Engineering Maths - IV Laplace Transform Page 4
  • 5. Engineering Maths - IV Now𝐿[𝑔′(𝑡)] = 𝐿[𝑓(𝑡)] 𝑠𝐿[𝑔(𝑡)] − 𝑔(0) = 𝐿[𝑓(𝑡)] 𝑠𝐿[𝑔(𝑡)] = 𝐿[𝑓(𝑡)] ∴ 𝑔(0) = 0 𝐿[𝑔(𝑡)] = 𝐿[𝑓(𝑡)] 𝑠 𝒕 𝑭(𝒔) ∴ 𝑳 [∫ 𝟎 𝒇(𝒕)𝒅𝒕] = 𝒔 𝒅𝒔 Theorem: 2 If𝑳[𝒇(𝒕)] = 𝑭(𝒔), then 𝑳[𝒕𝒇(𝒕)] = − 𝒅 𝑭(𝒔) Proof: Given 𝐿[𝑓(𝑡)] = 𝐹(𝑠) ∴ ∫ ∞ 𝑒−𝑠𝑡 𝑓(𝑡) 𝑑𝑡 = 𝐹(𝑠) ⋯ ⋯ (1) 0 Differentiating (1) with respect to s, we get 𝑑 ∞ 𝑒−𝑠𝑡 𝑓(𝑡) 𝑑𝑡 = 𝑑 𝐹(𝑠) ∫ 𝑑𝑠 0 𝑑𝑠 𝑑𝑠 ∞ 𝜕 (𝑒−𝑠𝑡 )𝑓(𝑡) 𝑑𝑡 = 𝑑 𝐹(𝑠) ∫0 𝜕𝑠 𝑑𝑠 ∞ 𝑑 (−𝑡)𝑒−𝑠𝑡 𝑓(𝑡) 𝑑𝑡 = 𝐹(𝑠) ∫0 𝒅𝒔 ∞ 𝒅 𝑒−𝑠𝑡 𝑓(𝑡) 𝑑𝑡 = 𝐹(𝑠) − ∫ 0 𝑑𝑠 −𝐿[𝑡𝑓(𝑡)] = 𝑑 𝐹(𝑠) 𝒅𝒔 ∴ 𝑳[𝒕𝒇(𝒕)] = − 𝒅 𝑭(𝒔) 𝒅𝒔𝒏 𝒏 Note: In general 𝑳[𝒕𝒏𝒇(𝒕)] = (−𝟏)𝒏 𝒅 𝑭(𝒔) Example: 5.3 If 𝑳[𝒇(𝒕)] = 𝒔𝟐−𝒔+𝟏 (𝟐𝒔+𝟏)𝟐(𝒔−𝟏) then find 𝑳[𝒇(𝟐𝒕)]. Solution: Given 𝐿[𝑓(𝑡)] = 𝑠2−𝑠+1 (2𝑠+1)2(𝑠−1) = 𝐹(𝑠) 2 2 𝐿[𝑓(2𝑡)] = 1 𝐹 (𝑠 ) = 1 2 𝑠 2 𝑠 ( ) − +1 2 2 2 2 𝑠 𝑠 2 (2 +1) ( −1) = 1 𝑠2 𝑠+4] [ 4 − 2 𝑠−2 = 2 (𝑠+1)2 ( ) 2 𝑠2−2𝑠+1 4(𝑠+1)2(𝑠−2) Laplace transform of some Standard functions Result: 1 Prove that 𝑳[𝒕𝒏] = 𝚪(𝒏+𝟏) 𝒔𝒏+𝟏 Proof: Laplace Transform Page 5
  • 6. ∞ We know that 𝐿[𝑓(𝑡)] = ∫ 𝑒−𝑠𝑡 𝑓(𝑡) 𝑑𝑡 0 ∞ 𝑒−𝑠𝑡 𝑡𝑛 𝑑𝑡 𝐿[𝑡𝑛] = ∫ 0 [ 𝑛] −𝑢 ∞ 𝑢 𝑛 𝑑𝑢 𝐿 𝑡 = ∫0 𝑒 (𝑠 ) 𝑠 𝑠𝑛+1 ∞ 𝑢𝑛 𝑒−𝑢 = ∫0 𝑑𝑢 = 1 ∞ 𝑒−𝑢𝑢𝑛 𝑑𝑢 𝑠𝑛+1 ∫0 ∴ 𝑳[𝒕𝒏] = 𝚪(𝒏+𝟏) 𝒔𝒏+𝟏 ∞ −𝑢 𝑛 ∵ ∫0 𝑒 𝑢 𝑑𝑢 Note: If n is an integer, then Γ(𝑛 + 1) = 𝑛! n! 𝑠𝑛+1 ∴ 𝐿[𝑡𝑛] = if n is an integer If 𝑛 = 0 , then 𝐿[1] = 1 𝑠 If 𝑛 = 1 , then 𝐿[𝑡] = 1 𝑠2 Similarly 𝐿[𝑡2] = 2! 𝑠3 𝐿[𝑡3] = 3! 𝑠4 𝟏 𝒔−𝒂 Result: 2 Prove that 𝑳(𝒆𝒂𝒕) = , 𝒔 > 𝒂 Proof: We know that 𝐿[𝑓(𝑡)] = ∫ ∞ 𝑒−𝑠𝑡 𝑓(𝑡) 𝑑𝑡 0 ∞ 𝑒−𝑠𝑡𝑒𝑎𝑡 𝑑𝑡 ∴ 𝐿(𝑒𝑎𝑡) = ∫ 0 ∞ 𝑒−𝑡(𝑠−𝑎) 𝑓(𝑡) 𝑑𝑡 = ∫0 = [ ] −(𝑠−𝑎) 0 𝑒−𝑡(𝑠−𝑎) ∞ 𝑠−𝑎 = − [0 − ( 1 )] ∴ 𝐿(𝑒𝑎𝑡) = 1 𝑠−𝑎 𝟏 𝒔+𝒂 Result: 3 Prove that 𝑳(𝒆−𝒂𝒕) = , 𝒔 > 𝒂 Proof: We know that 𝐿[𝑓(𝑡)] = ∫ ∞ 𝑒−𝑠𝑡 𝑓(𝑡) 𝑑𝑡 0 ∞ 𝑒−𝑠𝑡𝑒−𝑎𝑡 𝑑𝑡 ∴ 𝐿(𝑒−𝑎𝑡) = ∫ 0 ∞ 𝑒−𝑡(𝑠+𝑎) 𝑓(𝑡) 𝑑𝑡 = ∫0 = [ ] −(𝑠+𝑎) 0 𝑒−𝑡(𝑠+𝑎) ∞ 𝑠+𝑎 = − [0 − ( 1 )] ∴ 𝐿(𝑒𝑎𝑡) = 1 𝑠+𝑎 Let 𝑠𝑡 = 𝑢 ⋯ ⋯ (1) 𝑡 = 𝑢 𝑠 𝑑𝑡 = 𝑑𝑢 𝑠 When 𝑡 → 0(1) => 𝑢 → 0 , 𝑡 → ∞,(1) => 𝑢 → ∞ Engineering Maths - IV Laplace Transform Page 6
  • 7. 𝒂 𝒔𝟐+𝒂𝟐 Result: 4 Prove that 𝑳[𝒔𝒊𝒏𝒂𝒕] = Proof: ∞ 𝑒−𝑠𝑡 𝑓(𝑡) 𝑑𝑡 We know that 𝐿[𝑓(𝑡)] = ∫0 ∞ 𝑒−𝑠𝑡 𝑠𝑖𝑛𝑎𝑡 𝑑𝑡 𝐿[𝑠𝑖𝑛𝑎𝑡] = ∫0 ∴ 𝐿[𝑠𝑖𝑛𝑎𝑡] = 𝒂 𝒔𝟐+𝒂𝟐 , 𝑠 > |𝑎| [∵ ∫0 ∞ 𝑒−𝑎𝑡 𝑠𝑖𝑛𝑏𝑡 𝑑𝑡 = 𝒃 𝒂𝟐+𝒃𝟐 ] 𝒔 𝒔𝟐+𝒂𝟐 Result: 5 Prove that 𝑳[𝒄𝒐𝒔𝒂𝒕] = Proof: ∞ 𝑒−𝑠𝑡 𝑓(𝑡) 𝑑𝑡 We know that 𝐿[𝑓(𝑡)] = ∫0 ∞ 𝑒−𝑠𝑡 𝑐𝑜𝑠𝑎𝑡 𝑑𝑡 𝐿[𝑐𝑜𝑠𝑎𝑡] = ∫0 ∴ 𝐿[𝑐𝑜𝑠𝑎𝑡] = 𝑠 𝑠2+𝑎2 , 𝑠 > |𝑎| ∵ ∫0 ∞ 𝑒−𝑎𝑡 𝑐𝑜𝑠𝑏𝑡 𝑑𝑡 = 𝒂 𝒂𝟐+𝒃𝟐 𝒂 𝒔𝟐−𝒂𝟐 Result: 6 Prove that 𝑳[𝒔𝒊𝒏𝒉𝒂𝒕] = Proof: , 𝒔 > |𝒂| 2 𝑎𝑡 −𝑎𝑡 We have 𝐿[𝑠𝑖𝑛ℎ𝑎𝑡] = 𝐿 [𝑒 −𝑒 ] 2 = 1 [𝐿(𝑒𝑎𝑡) − 𝐿(𝑒−𝑎𝑡)] − 2 𝒔−𝒂 𝒔+𝒂 = 1 [ 𝟏 𝟏 ] = 1 [𝒔+𝒂−𝒔+𝒂 ] 2 𝒔𝟐−𝒂𝟐 𝟐𝒂 2 𝒔𝟐−𝒂𝟐 = 1 [ ] ∴ 𝐿[𝑠𝑖𝑛ℎ𝑎𝑡] = 𝒂 𝒔𝟐−𝒂𝟐 , 𝑠 > |𝑎| 𝒔 𝒔𝟐−𝒂𝟐 Result: 7 Prove that 𝑳[𝒄𝒐𝒔𝒉𝒂𝒕] = Proof: , 𝒔 > |𝒂| 2 𝑎𝑡 −𝑎𝑡 We have 𝐿[𝑐𝑜𝑠ℎ𝑎𝑡] = 𝐿 [𝑒 +𝑒 ] 2 = 1 [𝐿(𝑒𝑎𝑡) + 𝐿(𝑒−𝑎𝑡)] + 2 𝒔−𝒂 𝒔+𝒂 = 1 [ 𝟏 𝟏 ] = 1 [𝒔+𝒂+𝒔−𝒂 ] 2 𝒔𝟐−𝒂𝟐 𝟐𝒔 2 𝒔𝟐−𝒂𝟐 = 1 [ ] ∴ 𝐿[𝑐𝑜𝑠ℎ𝑎𝑡] = 𝒔 𝒔𝟐−𝒂𝟐 , 𝑠 > |𝑎| 𝟏 Example: 5.4 Find 𝑳 [𝒕𝟐] Solution: We have 𝐿[𝑡𝑛] = Γ(𝑛+1) 𝑠𝑛+1 Engineering Maths - IV Laplace Transform Page 7
  • 8. Put 𝑛 = 𝟏 𝟐 𝟏 Γ 𝟏 ∴ 𝐿 [𝑡𝟐] = 𝟐 ( +1) 𝟏 𝑠𝟐 +1 ∵ Γ(𝑛 + 1) = 𝑛Γ𝑛 = 𝟏 Γ 𝟏 𝟐 𝟐 ( ) 𝟏 𝑠𝟐 +1 𝟐 ∵ Γ (𝟏 ) = √𝜋 = √𝜋 𝟑 2𝑠𝟐 𝟏 ∴ 𝐿 [𝑡𝟐] = √𝜋 2𝑠√𝑠 − 𝟏 Example: 5.5 Find the Laplace transform of 𝒕 𝟐 or 𝟏 √𝑡 Solution: We have 𝐿[𝑡𝑛] = Γ(𝑛+1) 𝑠𝑛+1 Put 𝑛 = − 𝟏 𝟐 ∴ 𝐿 [𝑡− 𝟏 𝟏 𝟐] = 𝟐 Γ(− +1) 𝟏 𝑠 𝟐 − +1 ∵ Γ(𝑛 + 1) = 𝑛Γ𝑛 Γ 𝟏 = 𝟐 ( ) 𝟏 𝑠𝟐 𝟏 𝟐 ∵ Γ ( ) = √𝜋 = √𝜋 √𝑠 ∴ 𝐿 𝟏 𝜋 [ ] = √ √𝑡 𝑠 FORMULA 𝑳[𝒇(𝒕)] = 𝑭(𝒔) 𝑳[𝒇(𝒕)] = 𝑭(𝒔) 𝑳[𝟏] = 𝟏 𝒔 𝑳[𝒕] = 𝟏 𝒔𝟐 𝑳[𝒕𝒏] = 𝚪(𝒏+𝟏) if n is not an integer 𝒔𝒏+𝟏 𝑳[𝒕𝒏] = 𝐧! if n is an integer 𝒔𝒏+𝟏 𝑳(𝒆𝒂𝒕) = 𝟏 𝒔−𝒂 𝑳(𝒆𝒂𝒕) = 𝟏 𝒔+𝒂 𝒂 𝑳[𝒔𝒊𝒏𝒂𝒕] = 𝒔𝟐 + 𝒂𝟐 𝒔 𝑳[𝒄𝒐𝒔𝒂𝒕] = 𝒔𝟐 + 𝒂𝟐 𝒔 𝑳[𝒄𝒐𝒔𝒉𝒂𝒕] = 𝒔𝟐 − 𝒂𝟐 𝒂 𝑳[𝒔𝒊𝒏𝒉𝒂𝒕] = 𝒔𝟐 − 𝒂𝟐 Problems using Linear property Example: 5.6 Find the Laplace transform for the following i. 𝟑𝒕𝟐 + 𝟐𝒕 + 𝟏 ii. (𝒕 + 𝟐)𝟑 iii. 𝒂𝒕 v. 𝒔𝒊𝒏√𝟐 𝒕 vi. 𝒔𝒊𝒏(𝒂𝒕 + 𝒃) vii. 𝒄𝒐𝒔𝟑𝟐𝒕 ix. 𝒔𝒊𝒏𝟐𝒕 x. 𝒄𝒐𝒔𝟐𝟐𝒕 xi. 𝒄𝒐𝒔𝟓𝒕𝒄𝒐𝒔𝟒𝒕 Engineering Maths - IV Laplace Transform Page 8
  • 9. iv. 𝒆𝟐𝒕+𝟑 viii. 𝒔𝒊𝒏𝟑𝒕 Engineering Maths - IV Solution: (i) Given 𝑓(𝑡) = 3𝑡2 + 2𝑡 + 1 𝐿[𝑓(𝑡)] = 𝐿[3𝑡2 + 2𝑡 + 1] = 𝐿[3𝑡2] + 𝐿[2𝑡] + 𝐿[1] = 𝐿[3𝑡2] + 𝐿[2𝑡] + 𝐿[1] = 3𝐿[𝑡2] + 2𝐿[𝑡] + 𝐿[1] 𝑠3 𝑠2 = 3 2 + 2 1 + 1 𝑠 𝑠3 𝑠2 ∴ 𝐿[3𝑡2 + 2𝑡 + 1] = 6 + 2 + 1 𝑠 𝑠4 𝑠3 𝑠2 (ii) Given 𝑓(𝑡) = (𝑡 + 2)3 = 𝑡3 + 3𝑡2(2) + 3𝑡22 + 23 𝐿[𝑓(𝑡)] = 𝐿[𝑡3 + 3𝑡2(2) + 3𝑡22 + 23] = 𝐿[𝑡3] + 𝐿[6𝑡2] + 𝐿[12𝑡] + 𝐿[8] = 𝐿[𝑡3] + 6𝐿[𝑡2] + 12𝐿[𝑡] + 8𝐿[1] = 6 + 12 + 12 + 12 𝑠 (𝑖𝑖𝑖) Given 𝑓(𝑡) = 𝑎𝑡 𝐿[𝑓(𝑡)] = 𝐿[𝑎𝑡 ] = 𝐿[𝑒𝑡𝑙𝑜𝑔𝑎 ] 𝐿[𝑎𝑡] = 1 𝑠−𝑙𝑜𝑔 𝑎 (iv)Given 𝑓(𝑡) = 𝑒2𝑡+3 𝐿[𝑓(𝑡)] = 𝐿[𝑒2𝑡+3] = 𝐿[𝑒2𝑡 . 𝑒3] = 𝑒3𝐿[𝑒2𝑡] 𝑠−2 1 = 𝑒3 [ ] 𝑠−2 1 ∴ 𝐿[𝑒2𝑡+3] = 𝑒3 [ ] (v) 𝐿[𝑠𝑖𝑛√2𝑡] = √2 𝑠2+2 (vi)Given 𝑓(𝑡) = sin(𝑎𝑡 + 𝑏) = 𝑠𝑖𝑛𝑎𝑡𝑐𝑜𝑠𝑏 + 𝑐𝑜𝑠𝑎𝑡𝑠𝑖𝑛𝑏 𝐿[𝑓(𝑡)] = 𝐿[sin(𝑎𝑡 + 𝑏)] = 𝐿[𝑠𝑖𝑛𝑎𝑡𝑐𝑜𝑠𝑏 + 𝑐𝑜𝑠𝑎𝑡𝑠𝑖𝑛𝑏] = 𝑐𝑜𝑠𝑏 𝐿[sin𝑎𝑡] + 𝑠𝑖𝑛𝑏 𝐿[cos𝑎𝑡] 𝐿[sin(𝑎𝑡 + 𝑏)] = 𝑐𝑜𝑠𝑏 + 𝑠𝑖𝑛𝑏 𝑠 𝑠 𝑠2+𝑎2 𝑠2+𝑎2 4 (vii) Given 𝑓(𝑡) = cos3 2𝑡 = 1 [3𝑐𝑜𝑠2𝑡 + 𝑐𝑜𝑠6𝑡] 4 𝐿[𝑓(𝑡)] = 1 𝐿[3𝑐𝑜𝑠2𝑡 + 𝑐𝑜𝑠6𝑡] 4 = 1 [3𝐿(𝑐𝑜𝑠2𝑡) + 𝐿(𝑐𝑜𝑠6𝑡)] = 1 [3 + 𝑠 𝑠 4 𝑠2+4 𝑠2+36 ] 3𝑐𝑜𝑠𝜃 + 𝑐𝑜𝑠3𝜃 ∵ cos3 𝜃 = 4 Laplace Transform Page 9
  • 10. 𝐿[cos3 2𝑡] = 1 [3 + 𝑠 𝑠 4 𝑠2+4 𝑠2+36 ] 4 (viii) Given 𝑓(𝑡) = sin3 𝑡 = 1 [3𝑠𝑖𝑛𝑡 − 𝑠𝑖𝑛3𝑡] 4 𝐿[𝑓(𝑡)] = 1 𝐿[3𝑠𝑖𝑛𝑡 − 𝑠𝑖𝑛3𝑡] 4 = 1 [3𝐿(𝑠𝑖𝑛𝑡) − 𝐿(𝑠𝑖𝑛3𝑡)] = 1 [3 − 1 3 4 𝑠2+1 𝑠2+9 ] 𝐿[sin3 𝑡] = 3 [ − 1 1 4 𝑠2+1 𝑠2+9 ] (ix) Given 𝑓(𝑡) = sin2 𝑡 = 1−𝑐𝑜𝑠2𝑡 2 2 𝐿[𝑓(𝑡)] = 𝐿 [1−𝑐𝑜𝑠2𝑡 ] 2 = 1 [𝐿(1) − 𝐿(𝑐𝑜𝑠2𝑡)] = 1 [1 − 2 𝑠 𝑠2+4 𝑠 ] 𝐿[cos2 2𝑡] = 1 [1 − 2 𝑠 𝑠2+4 𝑠 ] (x) Given 𝑓(𝑡) = cos2 2𝑡 = 1+𝑐𝑜𝑠4𝑡 2 2 𝐿[𝑓(𝑡)] = 𝐿 [1+𝑐𝑜𝑠4𝑡 ] 2 = 1 [𝐿(1) + 𝐿(𝑐𝑜𝑠4𝑡)] = 1 [1 + 2 𝑠 𝑠2+16 𝑠 ] 𝐿[cos2 2𝑡] = 1 [1 + 2 𝑠 𝑠2+16 𝑠 ] (xi) Given 𝑓(𝑡) = 𝑐𝑜𝑠5𝑡𝑐𝑜𝑠4𝑡 𝐿[𝑓(𝑡)] = 𝐿[𝑐𝑜𝑠5𝑡𝑐𝑜𝑠4𝑡] 2 = 1 [𝐿(𝑐𝑜𝑠9𝑡) + 𝐿(𝑐𝑜𝑠𝑡)] = 1 [ 𝑠 𝑠 2 𝑠2+81 𝑠2+1 + ] Problems using First Shifting theorem 𝑳[𝒆−𝒂𝒕𝒇(𝒕)] = 𝑳[𝒇(𝒕)]𝒔→𝒔+𝒂 𝑳[𝒆𝒂𝒕𝒇(𝒕)] = 𝑳[𝒇(𝒕)]𝒔→𝒔−𝒂 Example: 5.7 Find the Laplace transform for the following: i. ii. 𝒕𝒆−𝟑𝒕 𝒕𝟑𝒆𝟐𝒕 iii. 𝒆𝟒𝒕𝒔𝒊𝒏𝟐𝒕 iv. 𝒆−𝟓𝒕𝒄𝒐𝒔𝟑𝒕 v. 𝒔𝒊𝒏𝒉𝟐𝒕𝒄𝒐𝒔𝟑𝒕 vii. 𝒕𝟐𝟐𝒕 viii. 𝒕𝟑𝟐−𝒕 ix. 𝒆−𝟐𝒕𝒔𝒊𝒏𝟑𝒕𝒄𝒐𝒔𝟐𝒕 x. 𝒆−𝟑𝒕𝒄𝒐𝒔𝟒𝒕𝒄𝒐𝒔𝟐𝒕 xi. 𝒆𝟒𝒕𝒄𝒐𝒔𝟑𝒕𝒔𝒊𝒏𝟐𝒕 Engineering Maths - IV Laplace Transform Page 10
  • 11. vi. cosh3tsin2t (i) 𝒕𝒆−𝟑𝒕 𝐿[𝑡𝑒−3𝑡] = 𝐿[𝑡]𝑠→𝑠+3 = ( 1 ) 𝑠2 𝑠→𝑠+3 ∵ 𝐿(𝑡) = 1 𝑠2 ∴ 𝐿[𝑡𝑒−3𝑡] = 1 (𝑠+3)2 (ii) 𝒕𝟑𝒆𝟐𝒕 𝐿[𝑡3𝑒2𝑡] = 𝐿[𝑡3]𝑠→𝑠−2 = (3! ) 𝑠4 𝑠→𝑠−2 ∵ 𝐿(𝑡) = 3! 𝑠3+1 ∴ 𝐿[𝑡3𝑒2𝑡] = 6 (𝑠−2)4 (iii) 𝒆𝟒𝒕𝒔𝒊𝒏𝟐𝒕 𝐿[𝑒4𝑡𝑠𝑖𝑛2𝑡] = 𝐿[𝑠𝑖𝑛2𝑡]𝑠→𝑠−4 2 = ( ) = 𝑠2+22 𝑠→𝑠−4 2 = (𝑠−4)2+4 2 𝑠2−8𝑠+16+4 ∴ 𝐿[𝑒4𝑡𝑠𝑖𝑛2𝑡] = 2 𝑠2−8𝑠+20 (iv) 𝑳[𝒆−𝟓𝒕𝒄𝒐𝒔𝟑𝒕] 𝐿[𝑒−5𝑡𝑐𝑜𝑠3𝑡] = 𝐿[𝑐𝑜𝑠3𝑡]𝑠→𝑠+5 𝑠 = ( ) = 𝑠2+32 𝑠→𝑠+5 𝑠+5 = (𝑠+5)2+9 𝑠+5 𝑠2+10𝑠+25+9 𝑠+5 𝑠2+10𝑠+34 ∴ 𝐿[𝑒−5𝑡𝑐𝑜𝑠3𝑡] = (v) 𝑳[𝒔𝒊𝒏𝒉𝟐𝒕𝒄𝒐𝒔𝟑𝒕] 2 2𝑡 −2𝑡 𝐿[𝑠𝑖𝑛ℎ2𝑡𝑐𝑜𝑠3𝑡] = 𝐿 [(𝑒 −𝑒 ) 𝑐𝑜𝑠3𝑡] 2 = 1 [𝐿(𝑒2𝑡𝑐𝑜𝑠3𝑡) − 𝐿(𝑒−2𝑡𝑐𝑜𝑠3𝑡)] 2 𝑠→𝑠−2 = 1 [𝐿(𝑐𝑜𝑠3𝑡) − 𝐿(𝑐𝑜𝑠3𝑡)𝑠→𝑠+2] 2 = 1 [( ) 𝑠 𝑠 − ( ) 𝑠2+32 𝑠→𝑠−2 𝑠2+32 𝑠→𝑠+2 ] ∴ 𝐿[𝑠𝑖𝑛ℎ2𝑡𝑐𝑜𝑠3𝑡] = 1 [ 𝑠−2 𝑠+2 2 (𝑠−2)2+9 (𝑠+2)2+9 − ] (vi) 𝑳[𝒄𝒐𝒔𝒉𝟑𝒕𝒔𝒊𝒏𝟐𝒕] 2 3𝑡 −3𝑡 𝐿[𝑐𝑜𝑠ℎ3𝑡𝑠𝑖𝑛2𝑡] = 𝐿 [(𝑒 +𝑒 ) 𝑠𝑖𝑛2𝑡] Engineering Maths - IV Laplace Transform Page 11
  • 12. 2 = 1 [𝐿(𝑒3𝑡𝑠𝑖𝑛2𝑡) + 𝐿(𝑒−3𝑡𝑠𝑖𝑛2𝑡)] 2 𝑠→𝑠−3 = 1 [𝐿(𝑠𝑖𝑛2𝑡) + 𝐿(𝑠𝑖𝑛2𝑡)𝑠→𝑠+3] 2 = 1 [( ) 2 2 + ( ) 𝑠2+22 𝑠→𝑠−3 𝑠2+22 𝑠→𝑠+3 ] ∴ 𝐿[𝑐𝑜𝑠ℎ3𝑡𝑠𝑖𝑛2𝑡] = 1 [ 2 2 2 (𝑠−3)2+4 (𝑠+3)2+4 + ] (vii) 𝒕𝟐𝟐𝒕 𝐿[𝑡22𝑡] = 𝐿[𝑡2𝑒𝑙𝑜𝑔2𝑡 ] = 𝐿[𝑡2𝑒𝑡𝑙𝑜𝑔2] = 𝐿[𝑡2]𝑠→𝑠−𝑙𝑜𝑔2 = (2! ) 𝑠3 𝑠→𝑠−𝑙𝑜𝑔2 2 = (𝑠−𝑙𝑜𝑔2)3 ∴ 𝐿[𝑡22𝑡] = 2 (𝑠−𝑙𝑜𝑔2)3 (viii) 𝒕𝟑𝟐−𝒕 𝐿[𝑡32−𝑡] = 𝐿[𝑡3𝑒𝑙𝑜𝑔2−𝑡 ] = 𝐿[𝑡3𝑒−𝑡𝑙𝑜𝑔2] = 𝐿[𝑡3]𝑠→𝑠+𝑙𝑜𝑔2 = (3! ) = 𝑠4 𝑠→𝑠+𝑙𝑜𝑔2 6 ∴ 𝐿[𝑡32−𝑡] = (𝑠+𝑙𝑜𝑔2)4 6 (𝑠+𝑙𝑜𝑔2)4 (ix) 𝑳[𝒆−𝟐𝒕𝒔𝒊𝒏𝟑𝒕𝒄𝒐𝒔𝟐𝒕] 𝐿[𝑒−2𝑡𝑠𝑖𝑛3𝑡𝑐𝑜𝑠2𝑡] = 𝐿[𝑠𝑖𝑛3𝑡𝑐𝑜𝑠2𝑡]𝑠→𝑠+2 2 = 1 𝐿[sin(3𝑡 + 2𝑡) + sin(3𝑡 − 2𝑡)]𝑠→𝑠+2 2 = 1 𝐿[sin 5𝑡 + sin 𝑡]𝑠→𝑠+2 2 = 1 [L(sin 5𝑡) + 𝐿(sin 𝑡)]𝑠→𝑠+2 = 1 [ 2 𝑠2+52 5 1 + ] 𝑠2+12 𝑠→𝑠+2 = 1 [ + 5 1 2 (𝑠+2)2+25 (𝑠+2)2+1 ] ∴ 𝐿[𝑒−2𝑡𝑠𝑖𝑛3𝑡𝑐𝑜𝑠2𝑡] == 1 [ + 5 1 2 (𝑠+2)2+25 (𝑠+2)2+1 ] (x) 𝑳[𝒆−𝟑𝒕𝒄𝒐𝒔𝟒𝒕𝒄𝒐𝒔𝟐𝒕] 𝐿[𝑒−3𝑡𝑐𝑜𝑠4𝑡𝑐𝑜𝑠2𝑡] = 𝐿[𝑐𝑜𝑠4𝑡𝑐𝑜𝑠2𝑡]𝑠→𝑠+3 2 = 1 𝐿[cos(4𝑡 + 2𝑡) + cos(4𝑡 − 2𝑡)]𝑠→𝑠+3 2 = 1 𝐿[cos6t + cos2𝑡]𝑠→𝑠+3 Engineering Maths - IV Laplace Transform Page 12
  • 13. Engineering Mathematics - II 2 = 1 [L(cos6𝑡) + 𝐿(cos2 𝑡)]𝑠→𝑠+3 = 1 [ 2 𝑠2+62 𝑠 𝑠 + ] 𝑠2+22 𝑠→𝑠+3 = 1 [ + 𝑠+3 𝑠+3 2 (𝑠+3)2+36 (𝑠+3)2+4 ] ∴ 𝐿[𝑒−3𝑡𝑐𝑜𝑠4𝑡𝑐𝑜𝑠2𝑡] = 1 [ 𝑠+3 𝑠+3 2 (𝑠+3)2+36 (𝑠+3)2+4 + ] (xi) 𝑳[𝒆𝟒𝒕𝒄𝒐𝒔𝟑𝒕𝒔𝒊𝒏𝟐𝒕] 𝐿[𝑒4𝑡𝑐𝑜𝑠3𝑡𝑠𝑖𝑛2𝑡] = 𝐿[𝑐𝑜𝑠3𝑡𝑠𝑖𝑛2𝑡]𝑠→𝑠−4 2 = 1 𝐿[sin(3𝑡 + 2𝑡) − sin(3𝑡 − 2𝑡)]𝑠→𝑠−4 2 = 1 𝐿[sin 5𝑡 − sin 𝑡]𝑠→𝑠−4 2 = 1 [L(sin 5𝑡) − 𝐿(sin 𝑡)]𝑠→𝑠−4 = 1 [ 2 𝑠2+52 5 1 − ] 𝑠2+12 𝑠→𝑠−4 = 1 [ + 5 1 2 (𝑠−4)2+25 (𝑠−4)2+1 ] ∴ 𝐿[𝑒4𝑡𝑐𝑜𝑠3𝑡𝑠𝑖𝑛2𝑡] = 1 [ 5 1 2 (𝑠−4)2+25 (𝑠−4)2+1 + ] Exercise: 5.1 Find the Laplace transform for the following 1. cos2 3𝑡 Ans: 𝟏 [ 𝟑𝒔 + 𝒔 𝟒 𝒔𝟐+𝟗 𝒔𝟐+𝟖𝟏 ] 2. 𝑠𝑖𝑛3𝑡𝑐𝑜𝑠4𝑡 Ans: 𝟏 [ 𝟕 𝟏 − ] 3. 𝑡𝑒2𝑡 Ans: 𝟒 𝒔𝟐+𝟒𝟗 𝒔𝟐+𝟏 𝟏 4. 𝑡4𝑒−3𝑡 Ans: (𝒔−𝟐)𝟐 𝟒! 5. 𝑒4𝑡𝑠𝑖𝑛2𝑡 Ans: (𝒔−𝟑)𝟓 𝟐 6. 𝑒−5𝑡𝑐𝑜𝑠3𝑡 Ans: (𝒔−𝟒)𝟐+𝟒 𝒔+𝟓 7. 𝑡33𝑡 Ans: (𝒔+𝟓)𝟐+𝟗 𝟑! 8. 𝑡54−𝑡 Ans: (𝒔−𝒍𝒐𝒈𝟑)𝟒 𝟓! (𝒔+𝒍𝒐𝒈𝟒)𝟔 9. 𝑒−2𝑡𝑠𝑖𝑛3𝑡𝑐𝑜𝑠2𝑡 Ans: + 𝟓 𝟏 (𝒔+𝟐)𝟐+𝟐𝟓 (𝒔+𝟐)𝟐+𝟏 10. 𝑒−3𝑡𝑐𝑜𝑠4𝑡𝑐𝑜𝑠2𝑡 Ans: + 𝒔+𝟑 𝒔+𝟑 (𝒔+𝟑)𝟐+𝟑𝟔 (𝒔+𝟑)𝟐+𝟒 11. 𝑠𝑖𝑛ℎ𝑡𝑠𝑖𝑛4𝑡 Ans: − 𝟒 𝟒 (𝒔−𝟏)𝟐+𝟏𝟔 (𝒔+𝟏)𝟐+𝟏𝟔 12. 𝑐𝑜𝑠ℎ2𝑡𝑐𝑜𝑠2𝑡 Ans: 1 [ − 𝑠−2 𝑠+2 2 (𝑠−2)2+4 (𝑠+2)2+4 ] Laplace Transform Page 13
  • 14. Engineering Maths - IV 5.4 LAPLACE TRANSFORM OF DERIVATIVES AND INTEGRALS Problems using the formula 𝑳[𝒕𝒇(𝒕)] = −𝒅 𝑳[𝒇(𝒕)] 𝒅𝒔 Example: 5.8 Find the Laplace transform for 𝒕𝒔𝒊𝒏𝟒𝒕 Solution: 𝑑𝑠 𝐿[𝑡𝑠𝑖𝑛4𝑡] = −𝑑 𝐿[𝑡𝑠𝑖𝑛4𝑡] = −𝑑 [ 4 𝑑𝑠 𝑠2+4 ] 2 = −[(𝑠 +16)0−4(2𝑠)] (𝑠2+16)2 ∴ 𝐿[𝑡𝑠𝑖𝑛4𝑡] = 8𝑠 (𝑠2+16)2 Example: 5.9 Find 𝑳[𝒕𝒔𝒊𝒏𝟐𝒕] Solution: 𝐿[𝑡𝑠𝑖𝑛2𝑡] = −𝑑 𝐿[sin2 𝑡] = −𝑑 𝐿 [(1−𝑐𝑜𝑠2𝑡) ] 𝑑𝑠 𝑑𝑠 2 2 𝑑𝑠 = − 1 𝑑 [𝐿(1) − 𝐿(𝑐𝑜𝑠2𝑡)] 𝒔 2 𝑑𝑠 𝑠 𝒔𝟐+𝟒 = − 1 𝑑 [1 − ] 𝟐 𝟐 = − 1 𝑑 [𝒔 +𝟒−𝒔 ] 2 𝑑𝑠 𝒔(𝒔𝟐+𝟒) = − 1 𝑑 [ 2 𝑑𝑠 𝒔(𝒔𝟐+𝟒) 𝟒 ] = − 4 𝑑 [ 2 𝑑𝑠 𝒔(𝒔𝟐+𝟒) 𝟏 ] 𝟐 = −2 [𝟎−(𝟑𝒔 +𝟒) ] (𝒔𝟑+𝟒𝒔) 𝟐 𝟐 ∴ 𝐿[𝑡𝑠𝑖𝑛2𝑡] = 𝟐(𝟑𝒔 +𝟒) (𝒔𝟑+𝟒𝒔) 𝟐 Example: 5.10 Find 𝑳[𝒕𝒄𝒐𝒔𝟐𝟐𝒕] Solution: 𝐿[𝑐𝑜𝑠22𝑡] = −𝑑 𝐿[cos2 2𝑡] = −𝑑 𝐿 [(1+𝑐𝑜𝑠4𝑡) ] 𝑑𝑠 𝑑𝑠 2 2 𝑑𝑠 = − 1 𝑑 [𝐿(1) + 𝐿(𝑐𝑜𝑠4𝑡)] = − 1 𝑑 [1 + 2 𝑑𝑠 𝑠 𝒔𝟐+𝟏𝟔 𝒔 ] 𝑠2 2 (𝒔𝟐+𝟏𝟔)𝟐 𝟐 = − 1 [− 1 + (𝒔 +𝟏𝟔)𝟏−𝒔.𝟐𝒔 ] 𝑠2 2 (𝒔𝟐+𝟏𝟔)𝟐 𝟐 𝟐 = − 1 [− 1 + 𝒔 +𝟏𝟔−𝟐𝒔 ] ∴ 𝐿[𝑐𝑜𝑠22𝑡] = 1 [ 1 − 2 𝑠2 (𝒔𝟐+𝟏𝟔)𝟐 𝟏𝟔−𝒔𝟐 ] Laplace Transform Page 14
  • 15. Engineering Maths - IV Example: 5.11 Find the Laplace transform for 𝒕𝒔𝒊𝒏𝒉𝟐𝒕 Solution: 𝑑𝑠 𝐿[𝑠𝑖𝑛ℎ2𝑡] = −𝑑 𝐿[𝑠𝑖𝑛ℎ2𝑡] = −𝑑 [ 2 𝑑𝑠 𝑠2−4 ] 2 = −[(𝑠 −4)0−2(2𝑠)] (𝑠2−4)2 ∴ 𝐿[𝑡𝑠𝑖𝑛ℎ2𝑡] = 4𝑠 (𝑠2−4)2 Example: 5.12 Find the Laplace transform for 𝒇(𝒕) = 𝒔𝒊𝒏𝒂𝒕 − 𝒂𝒕𝒄𝒐𝒔𝒂𝒕 Solution: 𝐿[𝑠𝑖𝑛𝑎𝑡 − 𝑎𝑡𝑐𝑜𝑠𝑎𝑡] = L(sin 𝑎𝑡) − 𝑎 L(tcosat) = 𝑎 𝑑𝑠 − 𝑎 (−𝑑 𝐿[𝑐𝑜𝑠𝑎𝑡]) = 𝑠2+𝑎2 𝑎 𝑠 𝑠2+𝑎2 𝑑𝑠 𝑠2+𝑎2 + 𝑎 𝑑 [ ] = = 2 2 𝑎 + 𝑎 [(𝑠 +𝑎 )1−𝑠(2𝑠) ] 𝑠2+𝑎2 (𝑠2+𝑎2)2 𝑎 + 𝑎 [𝑠2+𝑎2−𝑠2 ] 𝑠2+𝑎2 (𝑠2+𝑎2)2 = 𝑎 𝑠2+𝑎2 (𝑠2+𝑎2)2 2 2 + 𝑎 [ 𝑎 −𝑠 ] 2 2 2 2 = 𝑎(𝑠 +𝑎 )+𝑎(𝑎 −𝑠 ) (𝑠2+𝑎2)2 2 3 3 2 = 𝑎𝑠 +𝑎 +𝑎 −𝑎𝑠 ) (𝑠2+𝑎2)2 ∴ 𝐿[𝑠𝑖𝑛𝑎𝑡 − 𝑎𝑡𝑐𝑜𝑠𝑎𝑡] = 2𝑎3 (𝑠2+𝑎2)2 Example: 5.13 Find the Laplace transform for the following (i) 𝒕𝒆−𝟑𝒕𝒔𝒊𝒏𝟐𝒕 (ii) 𝒕𝒆−𝒕𝒄𝒐𝒔𝒂𝒕 9iii) 𝒕𝒔𝒊𝒏𝒉𝒕𝒄𝒐𝒔𝟐𝒕 Solution: 𝑑𝑠 (i) 𝐿[𝑡𝑒−3𝑡𝑠𝑖𝑛2𝑡] = 𝐿[𝑡𝑠𝑖𝑛2𝑡] = −𝑑 𝐿[𝑠𝑖𝑛2𝑡] 𝑠→𝑠+3 𝑠→𝑠+3 𝑑𝑠 = −𝑑 ( 2 ) 𝑠2+22 𝑠→𝑠+3 (𝑠2+4)2 2 = [(𝑠 +4)0−2(2𝑠) ] 𝑠→𝑠+3 = [ 4𝑠 ] (𝑠2+4)2 𝑠→𝑠+3 ∴ 𝐿[𝑡𝑒−3𝑡 𝑠𝑖𝑛2𝑡] = 4(𝑠+3) ((𝑠+3)2+4 )2 𝑠→𝑠+1 𝑑𝑠 (ii) 𝐿[𝑡𝑒−𝑡𝑐𝑜𝑠𝑎𝑡] = 𝐿[𝑡𝑐𝑜𝑠𝑎𝑡] = −𝑑 𝐿[𝑐𝑜𝑠𝑎𝑡]𝑠→𝑠+1 𝑑𝑠 = −𝑑 ( 𝑠 ) 𝑠2+𝑎2 𝑠→𝑠+1 Laplace Transform Page 15
  • 16. (𝑠2+𝑎2)2 Engineering Maths - IV 2 2 = − [(𝑠 +𝑎 )1−𝑠(2𝑠) ] 𝑠→𝑠+1 2 2 = − [ 𝑎 −𝑠 ] (𝑠2+𝑎2)2 𝑠→𝑠+1 2 2 = [ 𝑠 −𝑎 ] (𝑠2+𝑎2)2 𝑠→𝑠+1 ∴ 𝐿[𝑡𝑒−𝑡𝑐𝑜𝑠𝑎𝑡] = (𝑠+1)2−𝑎2 ((𝑠+1)2+𝑎2 )2 (iii) 𝐿[𝑡𝑠𝑖𝑛ℎ𝑡𝑐𝑜𝑠2𝑡] 2 𝑡 −𝑡 𝐿[𝑡𝑠𝑖𝑛ℎ𝑡𝑐𝑜𝑠2𝑡] = 𝐿 [𝑡 (𝑒 −𝑒 ) 𝑐𝑜𝑠2𝑡] 2 = 1 [𝐿(𝑡𝑒𝑡𝑐𝑜𝑠2𝑡) − 𝐿(𝑡𝑒−𝑡𝑐𝑜𝑠2𝑡)] 2 𝑑𝑠 𝑑𝑠 = 1 [−𝑑 𝐿[𝑐𝑜𝑠2𝑡] + 𝑑 𝐿[𝑐𝑜𝑠2𝑡] 𝑠→𝑠−1 𝑠→𝑠+1] 2 𝑑𝑠 = 1 [−𝑑 ( 𝑠 𝑠2+4 𝑠→𝑠−1 𝑑𝑠 𝑠 𝑠2+4 𝑠→𝑠+1 ) + 𝑑 ( ) ] 2 (𝑠2+4)2 2 = 1 [− [(𝑠 +4)1−𝑠(2𝑠) ] 𝑠→𝑠−1 (𝑠2+4)2 2 + [(𝑠 +4)1−𝑠(2𝑠) ] 𝑠→𝑠+1 ] 2 = 1 [− [ 4−𝑠 ] 2 + [ 4−𝑠 ] 2 (𝑠2+4)2 𝑠→𝑠−1 (𝑠2+4)2 𝑠→𝑠+1 ] 2 ∴ 𝐿[𝑡𝑠𝑖𝑛ℎ𝑡𝑐𝑜𝑠2𝑡] = 1 [ (𝑠−1) −4 + 4−(𝑠+1)2 2 ((𝑠−1)2+4 )2 ((𝑠+1)2+4 )2 ] Problems using the formula 𝟐 𝑳[𝒕𝟐𝒇(𝒕)] = 𝒅 𝑳[𝒇(𝒕)] 𝒅𝒔𝟐 Example: 5.14 Find the Laplace transform for (i)𝒕𝟐𝒔𝒊𝒏𝒕 (ii) 𝒕𝟐𝒄𝒐𝒔𝟐𝒕 Solution: 𝑑𝑠2 2 (i) 𝐿[𝑡2𝑠𝑖𝑛𝑡] = 𝑑 𝐿[𝑠𝑖𝑛𝑡] 1 𝑑𝑠2 𝑠2+1 2 = 𝑑 [ ] 2 = 𝑑 ([(𝑠 +1)0−1(2𝑠)] ) 𝑑𝑠 (𝑠2+1)2 = 𝑑 ( 𝑑𝑠 (𝑠2+1)2 −2𝑠 ) = −2 𝑑 ( 𝑑𝑠 (𝑠2+1)2 𝑠 ) = 2 2 2 −2[(𝑠 +1) (1)−𝑠(2)(𝑠 +1)(2𝑠)] (𝑠2+1)4 2 2 2 = −2(𝑠 +1)[(𝑠 +1)−4𝑠 ] (𝑠2+1)4 2 = −2[1−3𝑠 ] (𝑠2+1)3 2 ∴ 𝐿[𝑡2𝑠𝑖𝑛𝑡] = 6𝑠 −2 (𝑠2+1)3 Laplace Transform Page 16
  • 17. 𝑑𝑠2 2 (ii) 𝐿[𝑡2𝑐𝑜𝑠2𝑡] = 𝑑 𝐿[𝑐𝑜𝑠2𝑡] 𝑠 2 = 𝑑 [ ] = ( 𝑑𝑠2 𝑠2+4 2 𝑑 [(𝑠 +4)1−𝑠(2𝑠)] 𝑑𝑠 (𝑠2+4)2 ) 2 = 𝑑 ( 4−𝑠 ) 𝑑𝑠 (𝑠2+4)2 = 2 2 ( 2 2 [(𝑠 +4) −2𝑠)−(4−𝑠 )2(𝑠 +4)(2𝑠)] (𝑠2+4)4 2 2 2 = 2𝑠(𝑠 +4)[(𝑠 +4)(−1)−(4−𝑠 )2] (𝑠2+4)4 2 = 2𝑠[𝑠 −12] (𝑠2+4)3 2 ∴ 𝐿[𝑡2 cos2𝑡] = 2𝑠[𝑠 −12] (𝑠2+4)3 Example: 5.15 Find the Laplace transform for (i)𝒕𝟐𝒆−𝟐𝒕𝒄𝒐𝒔𝒕 (ii) 𝒕𝟐𝒆𝟒𝒕𝒔𝒊𝒏𝟑𝒕 Solution: (i) 𝐿[𝑡2𝑒−2𝑡𝑐𝑜𝑠𝑡] = 𝐿[𝑡2𝑐𝑜𝑠𝑡]𝑠→𝑠+2 𝑑𝑠2 2 = 𝑑 𝐿[𝑐𝑜𝑠𝑡]𝑠→𝑠+2 𝑑𝑠2 𝑠 2 = 𝑑 ( ) 𝑠2+1 𝑠→𝑠+2 𝑑𝑠 (𝑠2+1)2 2 = 𝑑 [(𝑠 +1)1−𝑠(2𝑠) ] 𝑠→𝑠+2 2 = 𝑑 [ 1−𝑠 ] 𝑑𝑠 (𝑠2+1)2 𝑠→𝑠+2 = [ 2 2 ( 2 2 [(𝑠 +1) −2𝑠)−(1−𝑠 )2(𝑠 +1)(2𝑠)] (𝑠2+1)4 ] 𝑠→𝑠+2 (𝑠2+1)4 2 2 = (𝑠2 + 1)[[(𝑠 +1)(−2𝑠)−4𝑠(1−𝑠 )] ] 𝑠→𝑠+2 (𝑠2+1)3 3 3 = [−2𝑠 −2𝑠−4𝑠+4𝑠 ] 𝑠→𝑠+2 (𝑠2+1)3 3 = [2𝑠 −6𝑠 ] 𝑠→𝑠+2 ∴ 3 𝐿[𝑡2𝑒−2𝑡𝑐𝑜𝑠𝑡] = 2(𝑠+2) −6(𝑠+2) ((𝑠+2)2+1 )3 (ii)𝐿[𝑡2𝑒4𝑡𝑠𝑖𝑛3𝑡] = 𝐿[𝑡2𝑠𝑖𝑛3𝑡] 𝑠→𝑠−4 𝑑𝑠2 2 = 𝑑 𝐿[𝑠𝑖𝑛3𝑡]𝑠→𝑠−4 𝑑𝑠2 3 2 = 𝑑 ( ) 𝑠2+9 𝑠→𝑠−4 𝑑𝑠 (𝑠2+9)2 2 = 𝑑 [(𝑠 +9)0−3(2𝑠) ] = 𝑑 [ −6𝑠 ] 𝑑𝑠 (𝑠2+9)2 𝑠→𝑠−4 𝑠→𝑠−4 = −6 𝑑 [ 𝑠 ] 𝑑𝑠 (𝑠2+9)2 𝑠→𝑠−4 = −6 [ 2 2 [(𝑠 +9) ( ) 2 1 −(𝑠)2(𝑠 +9)(2𝑠)] (𝑠2+9)4 ] 𝑠→𝑠−4 Engineering Maths - IV Laplace Transform Page 17
  • 18. (𝑠2+9)4 Engineering Mathematics - II 2 2 = −6(𝑠2 + 9)[[(𝑠 +9)−4𝑠 ] ] 𝑠→𝑠−4 (𝑠2+9)3 2 = −6 [ 9−3𝑠 ] 𝑠→𝑠−4 (𝑠2+9)3 2 = [18𝑠 −54 ] 𝑠→𝑠−4 2 ∴ 𝐿[𝑡2𝑒4𝑡𝑠𝑖𝑛3𝑡] = 18(𝑠−4) −54 ((𝑠−4)2+9 )3 Exercise: 5.2 Find the Laplace transform for the following 2𝑎𝑠 (𝑠2+𝑎2)2 𝑠2−𝑎2 (𝑠2+𝑎2)2 6(𝑠+4) (𝑠+4)2+9 8𝑠 (𝑠2+64)2 − 4𝑠 (𝑠2+16)2 (𝑠−2)2−4 ((𝑠+4)2+4)2 1. 𝑡𝑠𝑖𝑛𝑎𝑡 Ans: 2. 𝑡𝑐𝑜𝑠𝑎𝑡 Ans: 3. 𝑡𝑒−4𝑡𝑠𝑖𝑛3𝑡 Ans: 4. 𝑡𝑐𝑜𝑠2𝑡𝑠𝑖𝑛6𝑡 Ans: 5. 𝑡𝑒−2𝑡𝑐𝑜𝑠2𝑡 Ans: Problems using the formula 𝑳 [𝒇(𝒕) ] = 𝒕 ∞ ∫𝒔 𝑳[𝒇(𝒕)]𝒅𝒔 𝑡→0 𝑡 This formula is valid if lim 𝑓(𝑡) is finite. The following formula is very useful in this section 𝑠 𝑑𝑠 ∫ = 𝑙𝑜𝑔𝑠 𝑠+𝑎 𝑑𝑠 ∫ = log(𝑠 + 𝑎) 𝑠 𝑑𝑠 1 ∫ = log(𝑠2 + 𝑎2) 𝑠2+𝑎2 2 𝑎 𝑑𝑠 𝑠 ∫ = tan−1 𝑠2+𝑎2 𝑎 𝒕 Example: 5.16 Find 𝑳 [𝒄𝒐𝒔𝒂𝒕 ] 𝑡→0 𝑡 0 0 Solution: lim 𝑐𝑜𝑠𝑎𝑡 = 𝑐𝑜𝑠𝑎(0) = 1 = ∞ ∴ Laplace transform does not exists. 𝒕 Example: 5.17 Find 𝑳 [𝒔𝒊𝒏𝒂𝒕 ] 𝑡→0 Solution: lim 𝑠𝑖𝑛𝑎𝑡 = 𝑠𝑖𝑛𝑎(0) = 0 𝑡 0 0 = lim𝑎𝑐𝑜𝑠𝑎𝑡 𝑡→0 (by applying L−Hospital rule) Laplace Transform Page 18
  • 19. Engineering Maths - IV lim𝑎𝑐𝑜𝑠𝑎𝑡 = 𝑎𝑐𝑜𝑠0 = 𝑎, finite quantity. 𝑡→0 Hence Laplace transform exists 𝑡 𝐿 [𝑠𝑖𝑛𝑎𝑡 ] = ∞ 𝐿[(𝑠𝑖𝑛𝑎𝑡)]𝑑𝑠 ∫𝑠 ∞ 𝑎 = ∫𝑠 𝑠2+𝑎2 𝑑𝑠 = [tan ] 𝑎 𝑠 −1 𝑠 ∞ 𝑎 = [tan−1∞ − tan−1 𝑠 ] = [𝜋 − tan−1 𝑠 ] 2 𝑎 ∴ 𝐿 [𝑠𝑖𝑛𝑎𝑡 ] = 𝑐𝑜t−1 𝑠 𝑡 𝑎 𝒕 𝟑 Example: 5.18 Find 𝑳 [𝒔𝒊𝒏 𝒕 ] Solution: 𝑡→0 𝑡→0 𝒔𝒊𝒏𝟑𝒕 = 𝟑𝒔𝒊𝒏𝒕−𝒔𝒊𝒏𝟑𝒕 𝒕 𝟒𝒕 𝒔𝒊𝒏𝟑𝒕 𝟑𝒔𝒊𝒏𝒕−𝒔𝒊𝒏𝟑𝒕 lim = lim 𝒕 𝟒𝒕 = 0−0 = 0 0 0 (by applying L−Hospital rule) 𝑡→0 4𝑡 = lim 3𝑠𝑖𝑛𝑡−𝑠𝑖𝑛3𝑡 = 0 𝑡 Hence Laplace transform exists 3 𝐿 [𝑠𝑖𝑛 𝑡 ] = 𝐿 [3𝑠𝑖𝑛𝑡 −𝑠𝑖𝑛3𝑡 ] = 1 4𝑡 ∞ 𝐿[(3𝑠𝑖𝑛𝑡 − 𝑠𝑖𝑛3𝑡)]𝑑𝑠 ∫ 4 𝑠 𝑠2+1 𝑠2+9 = 1 ∫ ∞ (3 1 − 3 ) 𝑑𝑠 4 𝑠 1 [ −1 −1 𝑠 = 3tan 𝑠 − tan ] 4 3 𝑠 ∞ = 1 [3(tan−1 ∞ − tan−1 𝑠) − (tan−1 ∞ − tan−1 𝑠 )] 4 3 = 1 [(𝜋 − tan−1 𝑠) − (𝜋 − tan−1 𝑠 ) ] 4 2 2 3 = 1 [cot−1 𝑠 − 𝑐𝑜t−1 𝑠 ] 4 3 𝒕 Example: 5.19 Find 𝑳 [𝒆−𝟐𝒕 𝒔𝒊𝒏𝟐𝒕𝒄𝒐𝒔𝟑𝒕 ] 𝒕 𝒕 Solution: 𝐿 [𝑒−2𝑡 𝒔𝒊𝒏𝟐𝒕𝒄𝒐𝒔𝟑𝒕 ] = 𝐿 [𝒔𝒊𝒏𝟐𝒕𝒄𝒐𝒔𝟑𝒕 ] 𝑠→𝑠+2 = 1 [∫ ∞ 𝐿(sin(3𝑡 + 2𝑡) − sin(3𝑡 − 2𝑡))𝑑𝑠] 2 𝑠 𝑠→𝑠+2 = 1 [∫ ∞ 𝐿((sin 5𝑡) − 𝐿(sin 𝑡))𝑑𝑠] 2 𝑠 𝑠→𝑠+2 Laplace Transform Page 19
  • 20. − 2 𝑠2+52 𝑠2+12 1 ∞ [ 5 = 1 [ ∫ 𝑠 ]] 𝑠→𝑠+2 2 5 −1 𝑠 −1 𝑠 ∞ = 1 [[tan −tan 𝑠] ] 𝑠→𝑠+2 = 1 [[(tan−1 ∞ − tan−1 𝑠 ) − (tan−1 ∞ − tan−1 𝑠)] ] 2 5 𝑠→𝑠+2 2 2 5 2 = 1 [(𝜋 − tan−1 𝑠 ) − (𝜋 − tan−1 𝑠) ] 𝑠→𝑠+2 = 1 [cot−1 𝑠 − 𝑐𝑜t−1 𝑠 ] 2 5 𝑠→𝑠+2 = 1 [cot−1 (𝑠+2) − 𝑐𝑜t−1(𝑠 + 2)] 2 5 −𝒂𝒕 −𝒃𝒕 Example: 5.20 Find the Laplace transform for 𝒆 −𝒆 𝒕 Solution: 𝑡→0 𝑡 𝑡→0 0 0 0 −𝑎𝑡 −𝑏𝑡 0 0 lim𝑒 −𝑒 = lim 𝑒 −𝑒 = 1−1 = 0 (use L− Hospital rule) 𝑡→0 −𝑎𝑡 −𝑏𝑡 = lim −𝑎𝑒 +𝑏𝑒 1 = −𝑎 + 𝑏 = 𝑏 − 𝑎 = a finite quantity Hence Laplace transform exists. 𝑡 −𝑎𝑡 −𝑏𝑡 𝐿 [𝑒 −𝑒 ] = ∞ −𝑎𝑡 −𝑏𝑡 ∫𝑠 𝐿[𝑒 − 𝑒 ]𝑑𝑠 ∞ −𝑎𝑡 −𝑏𝑡 = ∫𝑠 [𝐿(𝑒 ) − 𝐿(𝑒 )]𝑑𝑠 𝑠+𝑎 𝑠+𝑏 ∞ 1 1 = ∫ 𝑠 ( − ) 𝑑𝑠 𝑠 = [log(𝑠 + 𝑎) − log(𝑠 + 𝑏)]∞ = [𝑙𝑜𝑔 ] 𝑠+𝑏 𝑠 𝑠+𝑎 ∞ = [𝑙𝑜𝑔 𝑎 𝑠 𝑠(1+ ) 𝑏 ] 𝑠(1+𝑠 ) 𝑠 ∞ = 𝑙𝑜𝑔1 − 𝑙𝑜𝑔 𝑠+𝑎 = 0 − 𝑙𝑜𝑔 𝑠+𝑎 𝑠+𝑏 𝑠+𝑏 ∵ 𝑙𝑜𝑔1 = 0 = 𝑙𝑜𝑔 𝑠+𝑎 𝑠+𝑏 Example: 5.21 Find the Laplace transform of Solution: 𝟏−𝒄𝒐𝒔𝒕 𝒕 𝑡→0 lim 𝟏−𝒄𝒐𝒔𝒕 = 0 𝒕 0 𝑡→0 𝟏 1 lim 𝒔𝒊𝒏𝒕 = 0 = 0 (use L− Hospital rule) 𝑡 𝐿 [1−𝑐𝑜𝑠𝑡 ] exists. 𝐿 [1−𝑐𝑜𝑠𝑡 ] = 𝑡 ∞ ∫𝑠 𝐿[(1 − 𝑐𝑜𝑠𝑡)]𝑑𝑠 𝑠 𝑠 𝑠2+1 ∞ 1 ( − ) 𝑑𝑠 = ∫𝑠 Engineering Maths - IV Laplace Transform Page 20
  • 21. 1 2 2 = [log𝑠 − log(𝑠 + 1)] 𝑠 ∞ 𝑠 ∞ = [log𝑠 − log√𝑠2 + 1] 𝑠 = [𝑙𝑜𝑔 ] √𝑠2+1 𝑠 ∞ = 0 − 𝑙𝑜𝑔 𝑠 √𝑠2+1 = 𝑙𝑜𝑔 √𝑠2+1 𝑠 Example: 5.22 Find the Laplace transform for 𝒄𝒐𝒔𝒂𝒕−𝒄𝒐𝒔𝒃𝒕 𝒕 Solution: 𝑡→0 lim 𝑐𝑜𝑠𝑎𝑡−𝑐𝑜𝑠𝑏𝑡 = 1−1 = 0 𝑡 0 0 (use L− Hospital rule) 𝑡→0 1 = lim −𝑎𝑠𝑖𝑛𝑎𝑡+𝑏𝑠𝑖𝑛𝑏𝑡 = 0 = a finite quantity Hence Laplace transform exists. 𝑡 𝐿 [𝑐𝑜𝑠𝑎𝑡−𝑐𝑜𝑠𝑏𝑡 ] = ∞ ∫𝑠 𝐿[𝑐𝑜𝑠𝑎𝑡 − 𝑐𝑜𝑠𝑏𝑡]𝑑𝑠 ∞ [𝐿(𝑐𝑜𝑠𝑎𝑡) − 𝐿(𝑐𝑜𝑠𝑏𝑡)]𝑑𝑠 = ∫𝑠 ( − 𝑠2+𝑎2 𝑠2+𝑏2 𝑠 ) 𝑑𝑠 ∞ 𝑠 = ∫𝑠 [1 2 ( 2 2) 1 2 2 2 = log 𝑠 + 𝑎 − log(𝑠 + 𝑏 )] 𝑠 ∞ = [𝑙𝑜𝑔 ] 2 𝑠2+𝑏2 𝑠 1 𝑠2+𝑎2 ∞ 2 2 𝑎2 𝑠2 𝑠2(1+𝑏2 ) 𝑠 (1+ ) = 1 [𝑙𝑜𝑔 𝑠2 ] 𝑠 ∞ 2 𝑎2 𝑏2 (1+ ) = 1 [𝑙𝑜𝑔 𝑠2 ] (1+ ) 𝑠2 𝑠 ∞ 2 2 2 2 = 1 [𝑙𝑜𝑔1 − 𝑙𝑜𝑔 𝑠 +𝑎 ] = − 1 [𝑙𝑜𝑔 𝑠 +𝑎 ] 2 𝑠2+𝑏2 2 𝑠2+𝑏2 [∵ 𝑙𝑜𝑔1 = 0] 2 2 = 1 [𝑙𝑜𝑔 𝑠 +𝑏 ] 2 𝑠2+𝑎2 Example: 5.23 Find the Laplace transform of Solution: 𝒔𝒊𝒏𝟐𝒕 𝒕 𝑡→0 𝑠𝑖𝑛2𝑡 = 1−𝑐𝑜𝑠2𝑡 𝑡 2𝑡 lim 𝟏−𝒄𝒐𝒔𝟐𝒕 = 0 𝟐𝒕 0 𝑡→0 𝟐 1 lim 𝟐𝒔𝒊𝒏𝟐𝒕 = 0 = 0 (use L− Hospital rule) Laplace transform exists. Engineering Maths - IV Laplace Transform Page 21
  • 22. 𝑡 2𝑡 2 𝐿 [𝑠𝑖𝑛 𝑡 ] = 𝐿 [1−𝑐𝑜𝑠2𝑡 ] = 1 ∞ ∫ 𝐿[(1 − 𝑐𝑜𝑠2𝑡)]𝑑𝑠 2 𝑠 = 1 ∞ [𝐿(1) − 𝐿(𝑐𝑜𝑠2𝑡)]𝑑𝑠 ∫ 2 𝑠 𝑠 𝑠2+4 = 1 ∞ (1 − 𝑠 ) 𝑑𝑠 ∫ 2 𝑠 1 [ 1 2 2 2 = log𝑠 − log(𝑠 + 4)] 𝑠 ∞ 2 𝑠 = 1 [log𝑠 − log√𝑠2 + 4] ∞ 2 1 [ = 𝑙𝑜𝑔 𝑠 2 √𝑠 +4 ∞ ] 𝑠 2 = 1 [0 − 𝑙𝑜𝑔 𝑠 √𝑠2+4 ] = 1 𝑙𝑜𝑔 √𝑠2+4 2 𝑠 Example: 5.24 Find the Laplace transform for 𝒔𝒊𝒏𝟐𝒕𝒔𝒊𝒏𝟓𝒕 𝒕 Solution: 𝑡 𝐿 [𝑠𝑖𝑛2𝑡𝑠𝑖𝑛5𝑡 ] = ∞ 𝐿[𝑠𝑖𝑛2𝑡𝑠𝑖𝑛5𝑡]𝑑𝑠 ∫𝑠 2 ∞ 1 [𝐿(cos(−3𝑡) − 𝐿(𝑐𝑜𝑠7𝑡)]𝑑𝑠 = ∫𝑠 = 1 ∞ ∫ [𝐿(cos(3𝑡) − 𝐿(𝑐𝑜𝑠7𝑡)]𝑑𝑠 2 𝑠 [∵ cos(−𝜃) = 𝑐𝑜𝑠𝜃] 𝑠 𝑠2+9 𝑠2+49 ( − ) 𝑑𝑠 = 1 ∞ 𝑠 ∫ 2 𝑠 1 [1 2 2 ( 2 ) 1 2 2 = log 𝑠 + 9 − log(𝑠 + 49)] 𝑠 ∞ = [𝑙𝑜𝑔 ] 4 𝑠2+49 𝑠 1 𝑠2+9 ∞ 1 4 = [𝑙𝑜𝑔 2 9 𝑠2 𝑠 (1+ ) 2 49 𝑠2 𝑠 (1+ ) ] 𝑠 ∞ 9 𝑠2 4 (1+49 ) = 1 [𝑙𝑜𝑔 (1+𝑠2) ] 𝑠 ∞ 2 2 = 1 [𝑙𝑜𝑔1 − 𝑙𝑜𝑔 𝑠 +9 ] = − 1 [𝑙𝑜𝑔 𝑠 +9 ] 4 𝑠2+49 4 𝑠2+49 [∵ 𝑙𝑜𝑔1 = 0] 2 = 1 [𝑙𝑜𝑔 𝑠 +49 ] 4 𝑠2+9 Problems using 𝑳 [ 𝒕 1 𝑠 ∫𝟎 𝒇(𝒕)𝒅𝒕] = 𝑳[𝒇(𝒕)] Example: 5.25 Find the Laplace transform for (i) 𝒕 −𝟐𝒕 ∫𝟎 𝒆 𝒅𝒕 (ii) 𝒕 ∫𝟎𝒄𝒐𝒔𝟐𝒕𝒅𝒕 (iii) ∫𝟎 𝒕 𝒕𝒔𝒊𝒏𝟑𝒕𝒅𝒕 (iv) 𝒕 𝒕 ∫ 𝟎 𝒄𝒐𝒔𝒕𝒅𝒕 Solution: (i) 𝐿 [∫ 𝑡 𝑒−2𝑡𝑑𝑡] = 1 𝐿[𝑒−2𝑡] = 1 ( 1 ) 0 𝑠 𝑠 𝑠+2 Engineering Maths - IV Laplace Transform Page 22
  • 23. ∴ 𝐿 [∫ 0 𝑡 𝑒−2𝑡𝑑𝑡] = 1 𝑠(𝑠+2) (ii) 𝐿 [∫ 𝑡 𝑐𝑜𝑠2𝑡𝑑𝑡] = 1 𝐿[𝑐𝑜𝑠2𝑡] = 1 ( 0 𝑠 𝑠2+4 𝑠 ) ∴ 𝐿 [∫ 0 𝑠 𝑡 𝑐𝑜𝑠2𝑡𝑑𝑡] = 1 𝑠2+4 (iii) 𝐿 [ 𝑠 𝑡 1 ∫0 𝑡𝑠𝑖𝑛3𝑡𝑑𝑡] = 𝐿[𝑡𝑠𝑖𝑛3𝑡] 𝑠 𝑑𝑠 = 1 [−𝑑 [𝐿[𝑠𝑖𝑛3𝑡]]] = −1 [ 𝑑 [ 3 𝑠 𝑑𝑠 𝑠2+9 ]] = −1 [ −6𝑠 𝑠 (𝑠2+9)2 ] 𝑡 ∴ 𝐿 [∫ 0 𝑡𝑠𝑖𝑛3𝑡𝑑𝑡] = 6 (𝑠2+9)2 (iv) 𝐿 [𝑡 𝑑𝑠 𝑡 −𝑑 𝑡 ∫0 𝑐𝑜𝑠𝑡𝑑𝑡] = 𝐿 [∫0𝑐𝑜𝑠𝑡𝑑𝑡] = −𝑑 [1 ( 𝑠 𝑑𝑠 𝑠 𝑠2+1 )] 1 𝑑𝑠 𝑠2+1 = − 𝑑 [ ] −2𝑠 = − [ ] ∴ 𝐿 [∫ 0 𝑡 𝑡𝑠𝑖𝑛3𝑡𝑑𝑡] = (𝑠2+1)2 2𝑠 (𝑠2+1)2 𝒕 𝒕𝒄𝒐𝒔𝟒𝒕𝒅𝒕 Example: 5.26 Find the Laplace transform for 𝒆−𝒕 ∫ 𝟎 Solution: 𝐿 [𝑒−𝑡 ∫ 0 𝑡 𝑡𝑐𝑜𝑠4𝑡𝑑𝑡] = 𝐿 [ 𝑠 𝑑𝑠 𝑠→𝑠+1 𝑠→𝑠+1 = − (1 𝑑 𝑡 −1 𝑑 ∫0 𝑡𝑐𝑜𝑠4𝑡𝑑𝑡] = [ 𝐿(𝑐𝑜𝑠4𝑡)] 𝑠 ) 𝑠 𝑑𝑠 𝑠2+16 𝑠→𝑠+1 𝑠 (𝑠2+16)2 2 = [−1 (𝑠 +16)1−𝑠(2𝑠) ] 𝑠→𝑠+1 𝑠 (𝑠2+16)2 2 2 = [−1 (𝑠 +16−2𝑠 ) ] 𝑠→𝑠+1 2 = [−1 (−𝑠 +16) ] 𝑠 (𝑠2+16)2 𝑠→𝑠+1 2 = [1 (𝑠 −16) ] 𝑠 (𝑠2+16)2 𝑠→𝑠+1 ∴ 𝐿 [𝑒−𝑡 ∫ 0 𝑡 𝑡𝑐𝑜𝑠4𝑡𝑑𝑡] = 𝑠+1 ((𝑠+1)2+16 )2 2 1 [ (𝑠+1) −16 ] 𝒕 𝒕 𝒔𝒊𝒏𝒕 𝒅𝒕 Example: 5.27 Find the Laplace transform of 𝒆−𝒕 ∫ 𝟎 Solution: 𝐿 [𝑒−𝑡 ∫ 0 𝑡 𝑡 ∫0 𝑡 𝑠𝑖𝑛𝑡 𝑑𝑡] = 𝐿 [ 𝑡 𝑠𝑖𝑛𝑡 𝑑𝑡] 𝑠→𝑠+1 Engineering Maths - IV Laplace Transform Page 23
  • 24. Engineering Mathematics - II 𝑠 𝑡 = [1 𝐿 (𝑠𝑖𝑛𝑡 )] 𝑠→𝑠+1 = [1 ∫ 𝑠 𝑠 ∞ 𝐿(𝑠𝑖𝑛𝑡)𝑑𝑠] 𝑠→𝑠+1 ∞ 1 = [1 ∫ 𝑠 𝑠 ] 𝑠2+1 𝑠→𝑠+1 𝑠 𝑠 = [1 [tan−1 𝑠 ]∞ ] 𝑠→𝑠+1 𝑠 = [1 (tan−1 ∞ − tan−1 𝑠) ] 𝑠→𝑠+1 𝑠 2 = [1 (𝜋 − tan−1 𝑠) ] 𝑠→𝑠+1 𝑠 = [1 cot−1 𝑠 ] 𝑠→𝑠+1 𝑡 ∴ 𝐿 [𝑒−𝑡 ∫ 0 𝑡 𝑠𝑖𝑛𝑡 𝑑𝑡] = 1 𝑠+1 𝑐𝑜t−1(𝑠 + 1) Exercise: 5.3 Find the Laplace transform of 1. 𝑠𝑖𝑛 𝑡 𝑡 Ans: cot−1 𝑠 2 2. 𝑒−2𝑡 𝑠𝑖𝑛𝑡 𝑡 Ans: 𝑐𝑜t−1(𝑠 + 2) 3. 𝑠𝑖𝑛𝑎𝑡−𝑠𝑖𝑛𝑏𝑡 𝑡 Ans: −𝑎𝑡 4. 𝑒 −𝑐𝑜𝑠𝑏𝑡 𝑡 Ans: cot−1 𝑠 − cot−1 𝑠 𝑎 𝑏 𝑙𝑜𝑔 √𝑠2+𝑏2 𝑠+𝑎 −𝑡 5. 1−𝑒 𝑡 Ans: 𝑙𝑜𝑔 s+1 𝑠 𝑡 𝑡 𝑠𝑖𝑛𝑡 𝑑𝑡 6. 𝑒−𝑡 ∫ 0 Ans: 1 𝑠+1 𝑐𝑜t−1(𝑠 + 1) 7. 𝑒−𝑡 ∫ 𝑡 𝑡𝑐𝑜𝑠𝑡𝑑𝑡 0 Ans: [ 1 𝑠2+2𝑠 𝑠+1 (𝑠2+2𝑠+2 )2 ] 𝑡 𝑡𝑒−𝑡𝑠𝑖𝑛𝑡𝑑𝑡 8. 𝑒−𝑡 ∫ 0 𝑠 𝑠2+2𝑠+2 Ans: 1 [ 2(𝑠+1) ] ∞ 𝑓(𝑡)𝑒−𝑠𝑡𝑑𝑡 = 𝐿[𝑓(𝑡)] Evaluation of integrals using Laplace transform Note: (i) ∫ 0 (ii) ∫0 ∞ 𝑓(𝑡)𝑒−𝑎𝑡 𝑑𝑡 = [𝐿[𝑓(𝑡)]] 𝑠=𝑎 (iii) ∫0 ∞ 𝑓(𝑡)𝑑𝑡 = [𝐿[𝑓(𝑡)]] 𝑠=0 𝒔𝟐+𝟒 Example: 5.28 If 𝑳[𝒇(𝒕)] = 𝒔+𝟐 , then find the value of ∞ ∫𝟎 𝒇(𝒕)𝒅𝒕 Solution: Given 𝐿[𝑓(𝑡)] = 𝑠+2 𝑠2+4 We know that ∫0 ∞ 𝑓(𝑡)𝑑𝑡 = [𝐿[𝑓(𝑡)]] 𝑠=0 𝑠+2 = [ ] = 2 𝑠2+4 𝑠=0 4 Laplace Transform Page 24
  • 25. 2 ∞ 1 ∫0 𝑓(𝑡)𝑑𝑡 = 𝒔𝟐−𝟗 ∞ 𝒆−𝟐𝒕𝒇(𝒕)𝒅𝒕 Example: 5.29 If 𝑳[𝒇(𝒕)] = 𝟓𝒔+𝟒 , then find the value of ∫ 𝟎 Solution: Given 𝐿[𝑓(𝑡)] = 5𝑠+4 𝑠2−9 We know that ∫0 ∞ 𝑒−2𝑡 𝑓(𝑡)𝑑𝑡 = [𝐿[𝑓(𝑡)]] 𝑠=2 = [5𝑠+4 ] = 14 𝑠2−9 𝑠=2 −5 5 ∞ −14 𝑒−2𝑡𝑓(𝑡)𝑑𝑡 = ∴ ∫0 Example: 5.30 Find the values of the following integrals using Laplace transforms: (i) ∫𝟎 ∞ 𝒕𝒆−𝟐𝒕𝒄𝒐𝒔𝟐𝒕𝒅𝒕 (ii) ∫𝟎 ∞ 𝒕𝟐𝒆−𝒕𝒔𝒊𝒏𝒕𝒅𝒕 (iii) 𝒕 ∞ 𝒆−𝒕−𝒆−𝟐𝒕 ( ) 𝒅𝒕 ∫𝟎 𝒕 (iv) ∫𝟎 ∞ 𝟏−𝒄𝒐𝒔𝒕 ( ) 𝒆−𝒕𝒅𝒕 (v) 𝒕 ∞ 𝒆−𝒂𝒕−𝒄𝒐𝒔𝒃𝒕 ∫ 𝟎 ( ) 𝒅𝒕 Solution: (i) ∫ ∞ 𝑡𝑒−2𝑡𝑐𝑜𝑠2𝑡𝑑𝑡 = 𝐿[𝑡𝑐𝑜𝑠2𝑡] 0 𝑠=2 𝑑𝑠 = [−𝑑 𝐿(𝑐𝑜𝑠2𝑡)] 𝑠=2 = −𝑑 ( 𝑠 ) 𝑑𝑠 𝑠2+4 𝑠=2 (𝑠2+4)2 2 = − [ (𝑠 +4)1−𝑠(2𝑠) ] 𝑠=2 2 = − [ (4−𝑠 ) ] (𝑠2+4)2 𝑠=2 = − (4−4) = 0 (4+4)2 (ii) 𝑠=1 𝑑𝑠2 ∞ 2 −𝑡 2 𝑑2 ∫0 𝑡 𝑒 𝑠𝑖𝑛𝑡𝑑𝑡 = 𝐿[𝑡 𝑠𝑖𝑛𝑡] = 𝐿[𝑠𝑖𝑛𝑡]𝑠=1 1 2 = 𝑑 ( ) 𝑑𝑠2 𝑠2+1 𝑠=1 = 𝑑 [ −1(2𝑠) ] 𝑑𝑠 (𝑠2+1)2 𝑠=1 = −2 𝑑 [ 𝑠 ] 𝑑𝑠 (𝑠2+1)2 𝑠=1 2 [(𝑠 +1) = −2 [ 2 2 (1)−𝑠.2(𝑠 +1)(2𝑠))] (𝑠2+1)4 ] 𝑠=1 = −2 [ [(𝑠2+1)[(𝑠2+1)−4𝑠2)]] (𝑠2+1)4 ] 𝑠=1 (𝑠2+1)3 2 = −2 [(1−3𝑠 ) ] 𝑠=1 (𝑠2+1)3 3 = [ 6𝑠 −2 ] 𝑠=1 = 4 = 1 (iii) ∫0 𝑡 𝑡 ∞ 𝑒−𝑡−𝑒−2𝑡 −𝑡 −2𝑡 ( ) 𝑑𝑡 = 𝐿 [𝑒 −𝑒 ] 𝑠=0 𝑠=0 8 2 = ∫ ∞ [𝐿[𝑒−𝑡 − 𝑒−2𝑡]𝑑𝑠] 𝑠 Engineering Maths - IV Laplace Transform Page 25
  • 26. 𝑠=0 ∞ = ∫ [[𝐿(𝑒−𝑡) − 𝐿(𝑒−2𝑡)]𝑑𝑠] 𝑠 𝑠+1 𝑠+2 − 1 ) 𝑑𝑠] 𝑠=0 ∞ [( 1 = ∫𝑠 = {[log(𝑠 + 1) − log(𝑠 + 2)]∞} 𝑠 𝑠 =0 𝑠+2 𝑠 𝑠+1 ∞ = {[𝑙𝑜𝑔 ] } 𝑠=0 = {𝑙𝑜𝑔 𝑠 𝑠(1+ ) 2 𝑠(1+𝑠 ) 𝑠 1 ∞ } 𝑠=0 = [0 − 𝑙𝑜𝑔 𝑠+1 ] 𝑠+2 𝑠=0 ∵ 𝑙𝑜𝑔1 = 0 = [𝑙𝑜𝑔 𝑠+2 ] = 𝑙𝑜𝑔2 𝑠+1 𝑠=0 (iv) 𝑡 ∞ 1−𝑐𝑜𝑠𝑡 −𝑡 ∫0 ( ) 𝑒 𝑑𝑡 ∫0 𝑡 𝑡 ∞ 1−𝑐𝑜𝑠𝑡 1−𝑐𝑜𝑠𝑡 ( ) 𝑒−𝑡𝑑𝑡 = 𝐿 [ ] 𝑆=1 𝑆=1 ∞ [𝐿[(1 − 𝑐𝑜𝑠𝑡)]𝑑𝑠] = ∫𝑠 𝑆=1 ∞ [[𝐿(1) − 𝐿(𝑐𝑜𝑠𝑡)]𝑑𝑠] = ∫𝑠 𝑠 𝑠2+1 𝑆=1 ∞ 1 𝑠 = ∫ 𝑠 [( − ) 𝑑𝑠] 2 𝑠 ∞ = {[log𝑠 − 1 log(𝑠2 + 1)] } 𝑆=1 𝑠 ∞ = {[log𝑠 − log√𝑠2 + 1] } 𝑆=1 = {[𝑙𝑜𝑔 𝑠 √𝑠2+1 𝑠 ∞ ] } 𝑆=1 = [0 − 𝑙𝑜𝑔 𝑠 ] √𝑠2+1 𝑠=1 𝑠 = [𝑙𝑜𝑔 √𝑠2+1 ] 𝑠=1 = 𝑙𝑜𝑔√2 𝑡 −𝑎𝑡 (v) ∞ (𝑒 −𝑐𝑜𝑠𝑏𝑡 ) 𝑑𝑡 ∫0 𝑡 𝑡 ∞ 𝑒−𝑎𝑡−𝑐𝑜𝑠𝑏𝑡 𝑒−𝑎𝑡−𝑐𝑜𝑠𝑏𝑡 ∫0 ( ) 𝑑𝑡 = 𝐿 [ ] 𝑆=0 𝑆=0 ∞ [𝐿[(𝑒−𝑎𝑡 − 𝑐𝑜𝑠𝑏𝑡)]𝑑𝑠] = ∫𝑠 𝑆=0 ∞ [[𝐿(𝑒−𝑎𝑡) − 𝐿(𝑐𝑜𝑠𝑏𝑡)]𝑑𝑠] = ∫𝑠 − 𝑠+𝑎 𝑠2+𝑏2 𝑠 ) 𝑑𝑠] 𝑆=0 ∞ [( 1 = ∫𝑠 1 2 2 2 = {[log(𝑠 + 𝑎) − log(𝑠 + 𝑏 )] 𝑠 ∞ } 𝑆=0 2 𝑠 2 ∞ = {[log(𝑠 + 𝑎) − log√𝑠 + 𝑏 ] } 𝑆=0 𝑠+𝑎 𝑠 ∞ = {[𝑙𝑜𝑔 ] } √𝑠2+𝑏2 𝑆=0 Engineering Maths - IV Laplace Transform Page 26
  • 27. = [0 − 𝑙𝑜𝑔 𝑠+𝑎 ] √𝑠2+𝑏2 𝑠=0 𝑠+𝑎 = [𝑙𝑜𝑔 √𝑠2+𝑏2 ] 𝑠=0 √𝑏2 = 𝑙𝑜𝑔 𝑎 = 𝑙𝑜𝑔 𝑏 𝑎 Exercise: 5.4 Find the values of the following integrals using Laplace transforms 1. ∫0 ∞ 𝑡𝑒−2𝑡𝑐𝑜𝑠𝑡𝑑𝑡 Ans: 3 25 13 250 2. ∫0 ∞ 𝑡𝑒−3𝑡𝑠𝑖𝑛𝑡𝑑𝑡 Ans: 𝑡 3. ∫0 ∞ 𝑒−𝑎𝑡−𝑒−𝑏𝑡 ( ) 𝑑𝑡 Ans: 𝑙𝑜𝑔 𝑏 𝑎 𝑡 4. ∫0 ∞ sin2 𝑡 𝑒−2𝑡 𝑑𝑡 Ans: 4 1 𝑙𝑜𝑔2 5. ∫0 ∞ 𝑐𝑜𝑠𝑎𝑡−𝑐𝑜𝑠𝑏𝑡 ( ) 𝑑𝑡 Ans: 𝑙𝑜𝑔 𝑎 𝑡 𝑏 ∞ 𝒇(𝒕)𝒆−𝒔𝒕𝒅𝒕 = 𝑳[𝒇(𝒕)] Laplace transform of Piecewise continuous functions ∫𝟎 Example: 5.31 Find the Laplace transform of 𝒇(𝒕) = { 𝒆−𝒕; 𝟎 < 𝑡 < 𝜋 𝟎 ; 𝒕 > 𝜋 Solution: ∞ 𝑓(𝑡)𝑒−𝑠𝑡𝑑𝑡 𝐿[𝑓(𝑡)] = ∫0 = ∫0 𝜋 𝑒−𝑠𝑡𝑒−𝑡𝑑𝑡 + ∞ 𝑒−𝑠𝑡 0𝑑𝑡 ∫𝜋 𝜋 𝑒−(𝑠+1)𝑡𝑑𝑡 = ∫0 = [ ] = 𝑒−(𝑠+1)𝑡 𝜋 𝑒−(𝑠+1)𝜋−𝑒0 −(𝑠+1) 0 −(𝑠+1) −(𝑠+1)𝜋 ∴ 𝐿[𝑓(𝑡)] = 1−𝑒 −(𝑠+1) Example: 5.32 Find the Laplace transform of 𝒇(𝒕) = { 𝒔𝒊𝒏𝒕; 𝟎 < 𝑡 < 𝜋 𝟎 ; 𝒕 > 𝜋 Solution: 𝐿[𝑓(𝑡)] = ∞ −𝑠𝑡 ∫0 𝑓(𝑡)𝑒 𝑑𝑡 = ∫𝟎 ∞ 𝑒−𝑠𝑡0𝑑𝑡 𝝅 𝑒−𝑠𝑡𝑠𝑖𝑛𝑡𝑑𝑡 + ∫ 𝜋 𝜋 𝑒−𝑠𝑡𝑠𝑖𝑛𝑡𝑑𝑡 = ∫0 𝒆−𝒔𝒕 𝟎 𝝅 𝑒−𝑠𝜋 [ = [ (−𝑠𝑠𝑖𝑛𝑡 − 𝑐𝑜𝑠𝑡)] = −𝑠𝑠𝑖𝑛𝜋 − 𝑐𝑜𝑠𝜋] − (−𝒔)𝟐+𝟏 𝑠2+1 𝑒0 2 𝑠 +1 [−𝑠𝑠𝑖𝑛0 − 𝑐𝑜𝑠0] 1 −𝑠𝜋 −𝑠𝜋 = 𝑒 (0 + 1) − (−1) = 𝑒 +1 𝑠2+1 𝑠2+1 𝑠2+1 Engineering Maths - IV Laplace Transform Page 27
  • 28. −𝒔𝝅 ∴ 𝐿[𝑓(𝑡)] = 𝒆 +𝟏 𝒔𝟐+𝟏 ( ) Example: 5.33 Find the Laplace transform of 𝒇 𝒕 = { 𝒕;𝟎 < 𝑡 < 1 𝟎 ; 𝒕 > 1 Solution: ∞ 𝑓(𝑡)𝑒−𝑠𝑡𝑑𝑡 𝐿[𝑓(𝑡)] = ∫0 1 −𝑠𝑡 = ∫0 𝑒 𝑡𝑑𝑡 + ∞ −𝑠𝑡 ∫1 𝑒 0𝑑𝑡 1 𝑡𝑒−𝑠𝑡𝑑𝑡 = ∫0 0 𝑒−𝑠𝑡 𝑒−𝑠𝑡 1 𝒆−𝒔 𝒆−𝒔 𝟏 = [𝑡 − (1) ] = − − 0 + −𝑠 (−𝑠)2 −𝒔 𝒔𝟐 𝒔𝟐 𝒔 −𝒔 −𝒔 ∴ 𝐿[𝑓(𝑡)] = − 𝒆 − 𝒆 + 𝟏 𝒔𝟐 𝒔𝟐 Exercise: 5.5 ( ) 1. Find the Laplace transform of 𝑓 𝑡 = { 0; 0 < 𝑡 < 2 3 ; 𝑡 > 2 Ans: 𝟑𝒆−𝟐𝒔 𝒔 ( ) 2. Find the Laplace transform of 𝑓 𝑡 = { 𝑒𝑡 ; 0 < 𝑡 < 1 0 ; 𝑡 > 1 Ans: 𝟏−𝒆−(𝒔−𝟏) 𝒔−𝟏 ( ) 3. Find the Laplace transform of 𝑓 𝑡 = { 1; 0 < 𝑡 < 1 0 ; 𝑡 > 1 Ans: 𝟏−𝒆−𝒔 𝒔 Unit step function The unit step function 𝑈(𝑡 − 𝑎) is defined as 𝑈(𝑡 − 𝑎) = {0; 𝑡 < 𝑎 0 ; 𝑡 > 𝑎 Example: 5.34 Find the Laplace transform of unit step functions. Solution: ∞ 𝑈(𝑡 − 𝑎)𝑒−𝑠𝑡𝑑𝑡 𝐿[𝑈(𝑡 − 𝑎)] = ∫0 ∞ 𝑒−𝑠𝑡𝑑𝑡 ∞ (1)𝑒−𝑠𝑡𝑑𝑡 = ∫ 𝑎 𝑎 = ∫00𝑑𝑡 + ∫𝑎 = [ −𝑠 𝑒−𝑠𝑡 ∞ ] = 0 − 𝑎 𝑒−𝑠𝑎 −𝑠 = 𝑒−𝑠𝑎 𝑠 −𝑠𝑎 𝐿[𝑈(𝑡 − 𝑎)] = 𝑒 𝑠 Second Shifting theorem Statement: If 𝐿[𝑓(𝑡)] = 𝐹(𝑠), then 𝐿[𝑓(𝑡 − 𝑎)𝑈(𝑡 − 𝑎)] = 𝑒−𝑎𝑠𝐹(𝑠) Proof: 𝑈(𝑡 − 𝑎)𝑓(𝑡 − 𝑎) = { 0; 𝑡 < 𝑎 𝑓(𝑡 − 𝑎) ; 𝑡 > 𝑎 By the definition of Laplace transform, ∞ 𝑈(𝑡 − 𝑎)𝑓(𝑡 − 𝑎)𝑒−𝑠𝑡𝑑𝑡 𝐿[𝑈(𝑡 − 𝑎)𝑓(𝑡 − 𝑎)] = ∫0 ∞ 𝑓(𝑡 − 𝑎)𝑒−𝑠𝑡𝑑𝑡 𝑎 = ∫00𝑑𝑡 + ∫𝑎 ∞ 𝑒−𝑠(𝑎+𝑥)𝑓(𝑥)𝑑𝑥 𝐿[𝑈(𝑡 − 𝑎)𝑓(𝑡 − 𝑎)] = ∫0 Engineering Maths - IV Laplace Transform Page 28
  • 29. ∞ = ∫ 𝑒−𝑠𝑎𝑒−𝑠𝑥𝑓(𝑥)𝑑𝑥 0 = 𝑒−𝑠𝑎 ∞ 𝑒−𝑠𝑥𝑓(𝑥)𝑑𝑥 ∫0 ∞ 𝑒−𝑠𝑡𝑓(𝑡)𝑑𝑡 Replace x by t 𝐿[𝑈(𝑡 − 𝑎)𝑓(𝑡 − 𝑎)] = 𝑒−𝑠𝑎 ∫ 0 = 𝑒−𝑠𝑎𝐿[𝑓(𝑡)] = 𝑒−𝑠𝑎𝐹(𝑠) 𝐿[𝑈(𝑡 − 𝑎)𝑓(𝑡 − 𝑎)] = 𝑒−𝑠𝑎𝐹(𝑠) 5.5 PERIODIC FUNCTIONS Definition: A function 𝑓(𝑡) is said to be periodic if 𝑓(𝑡 + 𝑇) = 𝑓(𝑡) for all values of t and for certain values of T. The smallest value of T for which 𝑓(𝑡 + 𝑇) = 𝑓(𝑡) for all t is called periodic function. Example: 𝑠𝑖𝑛𝑡 = sin(𝑡 + 2𝜋) = sin(𝑡 + 4𝜋) ⋯ ∴ 𝑠𝑖𝑛𝑡 is periodic function with period 2𝜋. Let 𝒇(𝒕) be a periodic function with period T. Then 𝟏 𝑳[𝒇(𝒕)] = 𝟏 − 𝒆−𝒔𝑻 𝑻 ∫ 𝒆−𝒔𝒕𝒇(𝒕)𝒅𝒕 𝟎 Problems on Laplace transform of Periodic function Example: 5.35 Find the Laplace transform of 𝒇(𝒕) = { 𝒔𝒊𝒏𝑚𝒕; 𝟎 < 𝑡 < 𝝅 𝑚 𝟎 ; 𝝅 < 𝑡 < 𝟐𝝅 𝑚 𝑚 𝑚 𝒇(𝒕 + 𝟐𝝅 ) = 𝒇(𝒕) Solution: The given function is a periodic function with period 𝑇 = 2𝜋 𝜔 𝐿[𝑓(𝑡)] = 1 𝑇 𝑒−𝑠𝑡𝑓(𝑡)𝑑𝑡 1−𝑒−𝑠𝑇 ∫0 = 1 −𝟐𝝅𝒔 1−𝑒 𝑚 𝝅 𝑚 0 𝟐𝝅 −𝑠𝑡 𝑚 −𝑠𝑡 𝝅 𝑚 [∫ 𝑠𝑖𝑛𝜔𝑡𝑒 𝑑𝑡 + ∫ 𝑒 (0)𝑑𝑡] = 1 1−𝑒 𝜔 𝜋 ∫𝜔 𝑠𝑖𝑛𝜔𝑡𝑒−𝑠𝑡𝑑𝑡 −2𝜋𝑠 0 = 1 −2𝜋 𝑠 1−𝑒 𝜔 𝑒−𝑠𝑡 [ (−𝑠𝑠𝑖𝑛𝜔𝑡 − 𝜔𝑐𝑜𝑠𝜔𝑡)] (−𝑠)2+𝜔2 0 𝜋 𝜔 = 1−𝑒 𝜔 −𝑠𝜋 1 𝑒 𝜔 −2𝜋𝑠 { 𝑠2+𝜔2 [−𝑠𝑠𝑖𝑛𝜋 − 𝜔𝑐𝑜𝑠𝜋] + 𝜔 𝑠2+𝜔2 } = 1−𝑒 𝜔 −𝑠𝜋 𝑠2+𝜔2 1 𝑒 𝜔 𝜔+𝜔 −2𝜋𝑠 [ ] = 1 12−(𝑒 𝜔 ) −𝜋𝑠 2 [ −𝑠𝜋 𝜔(𝑒 𝜔 +1) 𝑠2+𝜔2 ] Let 𝑡 − 𝑎 = 𝑥 ⋯ (1) 𝑡 = 𝑎 + 𝑥 𝑑𝑡 = 𝑑𝑥 When 𝑡 = 𝑎, (1) => 𝑥 = 0 When 𝑡 = ∞, (1) => 𝑥 = ∞ Engineering Maths - IV Laplace Transform Page 29
  • 30. = 1 −𝜋 𝑠 (1−𝑒 𝜔 )(1+𝑒 𝜔 ) −𝜋𝑠 [ −𝑠𝜋 𝜔(𝑒 𝜔 +1) 𝑠2+𝜔2 ] ∴ 𝐿[𝑓(𝑡)] = 𝜔 −𝜋𝑠 (1−𝑒 𝜔 )(𝑠2+𝜔2) Example: 5.36 Find the Laplace transform of 𝒇(𝒕) = { 𝑬;𝟎 ≤ 𝒕 ≤ 𝒂 −𝑬 ;𝒂 ≤ 𝒕 ≤ 𝟐𝒂 given that 𝒇(𝒕 + 𝟐𝒂) = 𝒇(𝒕). Solution: The given function is a periodic function with period 𝑇 = 2𝑎 𝐿[𝑓(𝑡)] = 1 𝑇 𝑒−𝑠𝑡𝑓(𝑡)𝑑𝑡 1−𝑒−𝑠𝑇 ∫0 = 1 2𝑎 𝑒−𝑠𝑡𝑓(𝑡)𝑑𝑡 1−𝑒−2𝑎𝑠 ∫0 = 1 1−𝑒−2𝑎𝑠 ∫0 [ 𝑎 𝐸𝑒−𝑠𝑡 𝑑𝑡 + ∫ 𝑎 2𝑎 −𝐸𝑒−𝑠𝑡𝑑𝑡] = 1 1−𝑒−2𝑎𝑠 [𝐸 ∫0 𝑎 𝑒−𝑠𝑡𝑑𝑡 − 𝐸 ∫ 𝑎 2𝑎 𝑒−𝑠𝑡𝑑𝑡] 𝐸 = [[ ] 1−𝑒−2𝑎𝑠 −𝑠 0 𝑎 𝑒−𝑠𝑡 𝑎 𝑒−𝑠𝑡 2𝑎 − [ ] ] −𝑠 = 𝐸 = 1−𝑒−2𝑎𝑠 𝐸 1−𝑒−2𝑎𝑠 𝑠 −𝑎𝑠 −2𝑎𝑠 −𝑎𝑠 [𝑒 + 1 − 𝑒 − 𝑒 ] −𝑠 𝑠 𝑠 𝑠 −𝑎𝑠 −2𝑎𝑠 [1−2𝑒 +𝑒 ] = 12−(𝑒−𝑎𝑠)2 𝑠 −𝑎𝑠 2 𝐸 [(1−𝑒 ) ] = 𝐸 (1−𝑒−𝑎𝑠)(1+𝑒−𝑎𝑠) 𝑠 −𝑎𝑠 2 [(1−𝑒 ) ] −𝑎𝑠 = 𝐸 (1−𝑒 ) 𝑠 (1+𝑒−𝑎𝑠) ∴ 𝐿[𝑓(𝑡)] = 𝐸 tanh (𝑎𝑠 ) 𝑠 2 Example: 5.37 Find the Laplace transform of 𝒇(𝒕) = { 𝟏;𝟎 ≤ 𝒕 ≤ 𝒂 𝟐 𝒂 𝟐 −𝟏 ; ≤ 𝒕 ≤ 𝒂 given that 𝒇(𝒕 + 𝒂) = 𝒇(𝒕). Solution: The given function is a periodic function with period 𝑇 = 𝑎 𝐿[𝑓(𝑡)] = 1 𝑇 𝑒−𝑠𝑡𝑓(𝑡)𝑑𝑡 1−𝑒−𝑠𝑇 ∫0 = 1 𝑎 𝑒−𝑠𝑡𝑓(𝑡)𝑑𝑡 1−𝑒−𝑎𝑠 ∫0 = 1 1−𝑒−𝑎𝑠 𝑎 0 𝑎 2 [∫2(1)𝑒−𝑠𝑡𝑑𝑡 + ∫𝑎(−1)𝑒−𝑠𝑡𝑑𝑡] = 1 1−𝑒−𝑎𝑠 𝑎 0 𝑎 2 [∫2 𝑒−𝑠𝑡𝑑𝑡 − ∫𝑎 𝑒−𝑠𝑡𝑑𝑡] = 1 1−𝑒−𝑎𝑠 0 𝑎 −𝑠𝑡 −𝑠 −𝑠 𝑎 2 −𝑠𝑡 2𝑎 [[𝑒 ] 2 − [𝑒 ] ] Engineering Maths - IV Laplace Transform Page 30
  • 31. = 1−𝑒−𝑎𝑠 −𝑠𝑎 −𝑠𝑎 1 [𝑒 2 + 1 + 𝑒−𝑎𝑠 − 𝑒 2 ] −𝑠 𝑠 𝑠 𝑠 = 1−𝑒−𝑎𝑠 −𝑠𝑎 𝑠 −𝑎𝑠 1 [1−2𝑒 2 +𝑒 ] = 1 12−(𝑒 2 ) −𝑠𝑎 2 [ −𝑠𝑎 2 (1−𝑒 2 ) 𝑠 ] = 1 −𝑠𝑎 (1−𝑒 2 )(1+𝑒 2 ) −𝑠𝑎 [ −𝑠𝑎 2 (1−𝑒 2 ) 𝑠 ] = 𝑠 −𝑠𝑎 1 (1−𝑒 2 ) −𝑠𝑎 (1+𝑒 2 ) (1−𝑒−2𝑥) [∵ 𝑡𝑎𝑛ℎ𝑥 = ] (1+𝑒−2𝑥) ∴ 𝐿[𝑓(𝑡)] = 1 tanh (𝑎𝑠 ) 𝑠 4 Example: 5.38 Find the Laplace transform of 𝒇(𝒕) = { 𝒕;𝟎 ≤ 𝒕 ≤ 𝒂 𝟐𝒂 − 𝒕 ;𝒂 ≤ 𝒕 ≤ 𝟐𝒂 given that 𝒇(𝒕 + 𝟐𝒂) = 𝒇(𝒕). Solution: The given function is a periodic function with period 𝑇 = 2𝑎 𝐿[𝑓(𝑡)] = 1 𝑇 𝑒−𝑠𝑡𝑓(𝑡)𝑑𝑡 1−𝑒−𝑠𝑇 ∫0 = 1 2𝑎 𝑒−𝑠𝑡𝑓(𝑡)𝑑𝑡 1−𝑒−2𝑎𝑠 ∫0 = 1 1−𝑒−2𝑎𝑠 ∫0 [ 𝑎 𝑡𝑒−𝑠𝑡𝑑𝑡 + ∫ 𝑎 2𝑎 (2𝑎 − 𝑡)𝑒−𝑠𝑡𝑑𝑡] 1 𝑎 𝑒−𝑠𝑡 𝑒−𝑠𝑡 𝑒−𝑠𝑡 𝑒−𝑠𝑡 0 𝑎 2𝑎 = [[𝑡 ( ) − ( )] − [(2𝑎 − 𝑡) ( ) − (−1)( )] ] 1−𝑒−2𝑎𝑠 −𝑠 (−𝑠)2 −𝑠 (−𝑠)2 = 𝑠 𝑠 𝑠2 𝑠2 𝑠2 𝑠2 1 [−𝑎𝑒−𝑎𝑠 − 𝑒−𝑎𝑠 + 1 + 𝑒−2𝑎𝑠 + 𝑎𝑒−𝑎𝑠 − 𝑒−𝑎𝑠 ] = 1−𝑒−2𝑎𝑠 𝑠2 1−𝑒−2𝑎𝑠 1 [1−2𝑒−𝑎𝑠+𝑒−2𝑎𝑠 ] = 12−(𝑒−𝑎𝑠)2 𝑠2 −𝑎𝑠 2 1 [(1−𝑒 ) ] = 1 (1−𝑒−𝑎𝑠)(1+𝑒−𝑎𝑠) 𝑠2 −𝑎𝑠 2 [(1−𝑒 ) ] −𝑎𝑠 = 1 (1−𝑒 ) 𝑠2 (1+𝑒−𝑎 𝑠) 𝑠2 2 = 1 tanh (𝑎𝑠 ) Exercise: 5.6 1. Find the Laplace transform of 𝑓(𝑡) = { 1; 0 ≤ 𝑡 ≤ 𝑎 2 2 −1 ;𝑎 ≤ 𝑡 ≤ 𝑎 given that 𝑓(𝑡 + 𝑎) = 𝑓(𝑡). Ans: 𝑘 tanh (𝑎𝑠 ) 𝑠 2 Engineering Maths - IV Laplace Transform Page 31
  • 32. Engineering Maths - IV 2. Find the Laplace transform of 𝑓(𝑡) = { 𝑡; 0 ≤ 𝑡 ≤ 𝑎 2𝑎 − 𝑡 ;𝑎 ≤ 𝑡 ≤ 2𝑎 given that 𝑓(𝑡 + 2𝑎) = 𝑓(𝑡). Ans: 𝑠2 2 1 tanh (𝑠𝜋 ) 3. Find the Laplace transform of 𝑎 𝑡 ; 0 ≤ 𝑡 ≤ 𝑎 2𝑎−𝑡 ;𝑎 ≤ 𝑡 ≤ 2𝑎 𝑓(𝑡) = { given that 𝑓(𝑡 + 2𝑎) = 𝑓(𝑡). 𝑎 Find the Laplace transform of Ans: 1 tanh (𝑠𝑎 ) 𝑎𝑠2 2 4. 𝑓(𝑡) 𝑠𝑖𝑛𝑡;0 < 𝑡 < 𝜋 = { 0 ;𝜋 < 𝑡 < 2𝜋 𝑓(𝑡 + 2𝜋) = 𝑓(𝑡) Ans: 1 (1−𝑒−𝜋𝑠)(𝑠2+1) 5.6 INVERSE LAPLACE TRANSFORM Definition If the Laplace transform of a function 𝑓(𝑡) is 𝐹(𝑠)𝑖𝑒. , 𝐿[𝑓(𝑡)] = 𝐹(𝑠),then𝑓(𝑡) is called an inverse Laplace transform of 𝐹(𝑠) and we write symbolically𝑓(𝑡) = 𝐿−1[𝐹(𝑠)], where 𝐿−1 is called the inverse Laplace transform operator. Inverse Laplace transform of elementary functions 𝑳[𝒇(𝒕)] = 𝑭(𝒔) 𝑳−𝟏[𝑭(𝒔)] = 𝒇(𝒕) 𝟏 𝑳[𝟏] = 𝒔 𝟏 𝐿−1 [ ] = 𝟏 𝒔 𝟏 𝑳[𝒕] = 𝒔𝟐 𝟏 𝐿−1 [ ] = 𝒕 𝒔𝟐 𝒏! 𝑳[𝒕𝒏] = 𝒊𝒇 𝒏 𝒊𝒔 𝒂𝒏 𝒊𝒏𝒕𝒆𝒈𝒆𝒓 𝒔𝒏+𝟏 𝒏! 𝐿−1 [ ] = 𝒕𝒏 𝒔𝒏+𝟏 𝟏 𝒕𝒏 𝐿−1 [ ] = 𝒔𝒏+𝟏 𝒏! 𝟏 𝑳[𝒆𝒂𝒕] = 𝒔 − 𝒂 𝟏 𝐿−1 [ ] = 𝒆𝒂𝒕 𝒔 − 𝒂 𝟏 𝑳[𝒆−𝒂𝒕] = 𝒔 + 𝒂 𝟏 𝐿−1 [ ] = 𝒆−𝒂𝒕 𝒔 + 𝒂 𝒂 𝑳[𝒔𝒊𝒏𝒂𝒕] = 𝒔𝟐 + 𝒂𝟐 𝟏 𝒔𝒊𝒏𝒂𝒕 𝐿−1 [ ] = 𝒔𝟐 + 𝒂𝟐 𝒂 𝒔 𝑳[𝒄𝒐𝒔𝒂𝒕] = 𝒔𝟐 + 𝒂𝟐 𝒔 𝐿−1 [ ] = 𝒄𝒐𝒔𝒂𝒕 𝒔𝟐 + 𝒂𝟐 Laplace Transform Page 32
  • 33. 𝒂 𝑳[𝒔𝒊𝒏𝒉𝒂𝒕] = 𝒔𝟐 − 𝒂𝟐 𝟏 𝒔𝒊𝒏𝒉𝒂𝒕 𝐿−1 [ ] = 𝒔𝟐 − 𝒂𝟐 𝒂 𝒔 𝑳[𝒄𝒐𝒔𝒂𝒕] = 𝒔𝟐 − 𝒂𝟐 𝒔 𝐿−1 [ ] = 𝒄𝒐𝒔𝒉𝒂𝒕 𝒔𝟐 − 𝒂𝟐 Result on inverse Laplace transform Result: 1 Linear property 𝑳[𝒇(𝒕)] = 𝑭(𝒔) and 𝑳[𝒈(𝒕)] = 𝑮(𝒔) ,then 𝑳−𝟏[𝒂𝑭(𝒔) ± 𝒃𝑮(𝒔)] = 𝒂𝑳−𝟏[𝑭(𝒔)] ± 𝒃𝑳−𝟏[𝑮(𝒔)] Where a and b are constants. Proof: We know that 𝐿[𝑎𝐹(𝑠) ± 𝑏𝐺(𝑠)] = 𝑎𝐿[𝐹(𝑠)] ± 𝑏𝐿[𝐺(𝑠)] = 𝑎 𝐹(𝑠) ± 𝑏 𝐺(𝑠) (𝑖.𝑒. )𝑎 𝐹(𝑠) ± 𝑏 𝐺(𝑠) = 𝐿[𝑎𝑓(𝑡) ± 𝑏𝑔(𝑡)] Operating 𝐿−1 on both sides, we get 𝐿−1[𝑎𝐹(𝑠) ± 𝑏𝐺(𝑠)] = 𝑎𝑓(𝑡) ± 𝑏𝑔(𝑡) 𝐿−1[𝑎𝐹(𝑠) ± 𝑏𝐺(𝑠)] = 𝑎𝐿−1[𝐹(𝑠)] ± 𝑏𝐿−1[𝐺(𝑠)] Result: 2 First shifting property (i) 𝑳−𝟏 [𝑭(𝒔 + 𝒂) = 𝒆−𝒂𝒕𝑳−𝟏[𝑭(𝒔)]] (ii) 𝑳−𝟏 [𝑭(𝒔 − 𝒂) = 𝒆𝒂𝒕𝑳−𝟏[𝑭(𝒔)]] Proof: Let 𝐿[𝑒−𝑎𝑡𝑓(𝑡)] = 𝐹[𝑠 + 𝑎] Operating 𝐿−1 on both sides, we get 𝑒−𝑎𝑡𝑓(𝑡) = 𝐿−1[𝐹[𝑠 + 𝑎]] 𝐿−1[𝐹[𝑠 + 𝑎]] = 𝑒−𝑎𝑡𝐿−1[𝐹(𝑠)] Result: 3 Multiplication by s. 𝒅𝒕 If 𝑳−𝟏[𝑭(𝒔)] = 𝒇(𝒕) and 𝒇(𝟎) = 𝟎, then 𝑳−𝟏[𝒔𝑭(𝒔)] = 𝒅 𝑳−𝟏[𝑭(𝒔)] Proof: We know that 𝐿[𝑓′(𝑡)] = 𝑠𝐿[𝑓(𝑡)] − 𝑓(0) = 𝑠𝐹(𝑠) Operating 𝐿−1 on both sides, we get 𝑓′(𝑡) = 𝐿−1[𝑠𝐹(𝑠)] 𝑑𝑡 𝑑 𝑓(𝑡) = 𝐿−1[𝑠𝐹(𝑠)] 𝑑𝑡 𝑑 𝐿−1[𝐹(𝑠)] = 𝐿−1[𝑠𝐹(𝑠)] 𝑑𝑡 ∴ 𝐿−1[𝑠𝐹(𝑠)] = 𝑑 𝐿−1[𝐹(𝑠)] ∵ 𝑓(𝑡) = 𝐿−1[𝐹(𝑠)] ∵ 𝑔(𝑡) = 𝐿−1[𝐺(𝑠)] Engineering Maths - IV Laplace Transform Page 33
  • 34. Engineering Maths - IV Result: 4 Division by s. 𝑳−𝟏 [𝑭(𝒔) ] = 𝒔 𝒕 𝐿−1[𝐹(𝑠)]𝑑𝑡 ∫𝟎 Proof: We know that 𝐿 [∫ 𝑡 𝑓 (𝑡)𝑑𝑡] = 1 𝐿[𝑓(𝑡)] = 1 𝐹(𝑠) 0 𝑠 𝑠 Operating 𝐿−1 on both sides ,we get ∫0 𝑠 𝑡 1 𝑓 (𝑡)𝑑𝑡 = 𝐿−1 [ 𝐹(𝑠)] 𝑠 𝑡 −1 −1 1 ∫0 𝐿 [𝐹(𝑠)] 𝑑𝑡 = 𝐿 [ 𝐹(𝑠)] 𝑠 ∴ 𝐿−1 [𝐹(𝑠) ] = ∫ 0 𝑡 𝐿−1[𝐹(𝑠)] 𝑑𝑡 Result: 5 Inverse Laplace transform of derivative 𝑳−𝟏[𝑭(𝒔)] = −𝟏 𝑳−𝟏 [ 𝒅 𝑭(𝒔)] 𝒕 𝒅𝒔 Proof: We know that 𝐿[𝑡𝑓(𝑡)] = −𝑑 𝐿[𝑓(𝑡)] = −𝑑 𝐹(𝑠) 𝑑𝑠 𝑑𝑠 Operating 𝐿−1 on both sides ,we get 𝑑𝑠 𝑑 𝑡𝑓(𝑡) = −𝐿−1 [ 𝐹(𝑠)] 𝐿−1[𝐹(𝑠)] = −1 𝐿−1 [ 𝑑 𝐹(𝑠)] 𝑡 𝑑𝑠 𝑓(𝑡) = −1 𝐿−1 [ 𝑑 𝐹(𝑠)] 𝑡 𝑑𝑠 𝐿−1[𝐹(𝑠)] = −1 𝐿−1 [ 𝑑 𝐹(𝑠)] 𝑡 𝑑𝑠 ∞ 𝐹(𝑠)𝑑𝑠] Result: 6 Inverse Laplace transform of integral 𝑳−𝟏[𝑭(𝒔)] = 𝒕𝐿−1[∫ 𝑠 Proof: We know that 𝐿 [𝑓(𝑡) ] = 𝑡 ∫𝑠 ∞ 𝐿(𝑓(𝑡)) 𝑑𝑠 = ∫𝑠 ∞ 𝐹(𝑠) 𝑑𝑠 Operating 𝐿−1 on both sides, we get 𝑡 𝑓(𝑡) = 𝐿−1[∫𝑠 ∞ 𝐹(𝑠) 𝑑𝑠] 𝑓(𝑡) = 𝑡𝐿−1[∫ 𝑠 ∞ 𝐹(𝑠) 𝑑𝑠] 𝐿−1[𝐹(𝑠)] = 𝑡𝐿−1[∫ 𝑠 ∞ 𝐹(𝑠) 𝑑𝑠] Problems under inverse Laplace transform of elementary functions Example: 5.39 Find the inverse Laplace for the following (i) 𝟏 𝟏 𝟐𝒔+𝟑 𝟒𝒔𝟐+𝟗 (ii) (iii) 𝒔𝟑−𝟑𝒔𝟐+𝟕 𝒔𝟒 (iv) 𝟑𝒔+𝟓 𝒔𝟐+𝟑𝟔 Solution: Laplace Transform Page 34
  • 35. (i) 𝐿−1 𝟏 𝟐𝒔+𝟑 [ ] = 𝐿−1 [ 𝟏 𝟑 𝟐 𝟐[𝒔+ ] ] = 𝟏 𝟐 −𝟑𝒕 𝑒− 𝟐 (ii) 𝐿−1 [ 𝟏 𝟒𝒔𝟐+𝟗 ] = 𝐿−1 [ 𝟏 𝟐 𝟗 𝟒[𝒔 +𝟒 ] ] 𝟒 = 𝟏 𝐿−1 [ 𝟏 𝟐 𝟗 𝟒 [𝒔 + ] ] 4 3⁄2 2 = 1 1 𝑠𝑖𝑛 3 𝑡 6 2 = 1 𝑠𝑖𝑛 3 𝑡 3 2 3 2 (iii) 𝐿−1 [𝑠 −3𝑠 +7 ] = 𝐿−1 [𝑠 − 3𝑠 + 7 ] 𝑠4 𝑠4 𝑠4 𝑠4 𝑠 𝑠2 𝑠4 1 1 = 𝐿−1 [1 ] − 3𝐿−1 [ ] + 7𝐿−1 [ ] 𝑠4 3 2 3 𝐿−1 [𝑠 −3𝑠 +7 ] = 1 − 3𝑡 + 7𝑡 3! 3𝑠+5 𝑠 1 (iv) 𝐿−1 [ ] = 3𝐿−1 [ ] + 5𝐿−1 [ ] 𝑠2+36 𝑠2+36 𝑠2+36 𝐿−1 [ 3𝑠+5 ] = 3𝑐𝑜𝑠6𝑡 + 5𝑠𝑖𝑛6𝑡 𝑠2+36 6 Inverse Laplace transform using First shifting theorem 𝑳−𝟏[𝑭(𝒔 + 𝒂)] = 𝒆−𝒂𝒕𝑳−𝟏[𝑭(𝒔)] Example: 5.40 Find the inverse Laplace transform for the following: (i) 𝟏 𝟏 (𝒔+𝟐)𝟐 𝟏 (ii) (iii) 𝟏 (iv) (𝒔−𝟑)𝟒 𝒔+𝟐 𝟏 𝒔𝟐−𝟐𝒔+𝟐 𝒔 (v) 𝒔𝟐−𝟒𝒔+𝟏𝟑 𝒔 (vi) (vii) (𝒔+𝟑)𝟐+𝟗 𝒔+𝟐 (viii) (𝒔+𝟐)𝟐+𝟐𝟓 𝒔 𝒔𝟐+𝟒𝒔+𝟐𝟎 𝟐𝒔+𝟑 (𝒔+𝟑)𝟐 𝒔 (ix) (𝒔−𝟒)𝟑 (x) 𝒔𝟐−𝟐𝒔+𝟐 (xi) 𝒔𝟐+𝟔𝒔+𝟐𝟓 (xii) 𝒔𝟐+𝟔𝒔−𝟕 Solution: (i) 𝐿−1 [ 1 (𝑠+2)2 𝑠2 ] = 𝑒−2𝑡 𝐿−1 [ 1 ] = 𝑒−2𝑡 𝑡 (ii) 𝐿−1 [ 1 𝑠4 3 ] = 𝑒3𝑡𝐿−1 [ 1 ] = 𝑒−2𝑡 𝑡 3! (iii) 𝐿−1 [ (𝑠−3)4 1 (𝑠+3)2+9 ] = 𝑒−3𝑡𝐿−1 [ 𝑠2+9 1 ] = 𝑒−3𝑡 𝑠𝑖𝑛3𝑡 3 (iv) 𝐿−1 [ 1 ] = 𝐿−1 [ 1 (𝑠−1)2+1 1 𝑠2+1 ] = 𝑒𝑡𝐿−1 [ ] = 𝑒𝑡𝑠𝑖𝑛𝑡 (v) 𝐿−1 [ 𝑠2−2𝑠+2 1 ] = 𝐿−1 [ 1 (𝑠−2)2+9 ] = 𝑒2𝑡𝐿−1 [ 𝑠2+9 1 ] = 𝑒2𝑡 𝑠𝑖𝑛3𝑡 3 (vi) 𝐿−1 [ 𝑠2−4𝑠+13 𝑠+2 (𝑠+2)2+25 ] = 𝑒−2𝑡𝐿−1 [ 𝑠 𝑠2+25 ] = 𝑒−2𝑡𝑐𝑜𝑠5𝑡 (vii) 𝐿−1 [ 𝑠+2 𝑠2+4𝑠+20 ] = 𝐿−1 [ 𝑠+2 (𝑠+2)2+16 ] Engineering Maths - IV Laplace Transform Page 35
  • 36. 𝑠2+16 𝑠 = 𝑒−2𝑡𝐿−1 [ ] = 𝑒−2𝑡𝑐𝑜𝑠4𝑡 (viii) 𝐿−1 [ 𝑠 ] = 𝐿−1 [𝑠+3−3 ] (𝑠+3)2 (𝑠+3)2 𝑠2 𝑠+3 3 = 𝐿−1 [ ] − 𝐿−1 [ ] (𝑠+3)2 (𝑠+3)2 = 𝐿−1 1 1 [ ] − 3𝐿−1 [ ] 𝑠+3 (𝑠+3)2 = 𝑒−3𝑡 − 3𝑒−3𝑡𝐿−1 1 [ ] = 𝑒−3𝑡 − 3𝑒−3𝑡𝑡 (ix) 𝐿−1 [ 𝑠 ] = 𝐿−1 [𝑠−4+4 ] (𝑠−4)3 (𝑠−4)3 𝑠−4 4 = 𝐿−1 [ ] + 𝐿−1 [ ] (𝑠−4)3 (𝑠−4)3 = 𝐿−1 1 1 [ ] + 4𝐿−1 [ ] (𝑠−4)2 (𝑠−4)3 = 𝑒4𝑡𝐿−1 1 1 [ ] + 4𝑒4𝑡𝐿−1 [ ] 𝑠2 𝑠3 2 = 𝑒4𝑡𝑡 + 4𝑒4𝑡 𝑡 2! = 𝑒4𝑡𝑡 + 2𝑒4𝑡𝑡2 (x) 𝐿−1 [ 𝑠 𝑠2−2𝑠+2 ] = 𝐿−1 [ (𝑠−1)2+1 (𝑠−1)2+1 𝑠 ] = 𝐿−1 [ 𝑠−1+1 ] 𝑠−1 1 = 𝐿−1 [ ] + 𝐿−1 [ ] (𝑠−1)2+1 (𝑠−1)2+1 𝐿−1 [ 𝑠 𝑠 1 = 𝑒𝑡𝐿−1 [ ] + 𝑒𝑡𝐿−1 [ ] 𝑠2+1 𝑠2+1 ] = 𝑒𝑡𝑐𝑜𝑠𝑡 + 𝑒𝑡𝑠𝑖𝑛𝑡 (xi) 𝐿−1 [ 𝑠2−2𝑠+2 2𝑠+3 𝑠2+6𝑠+25 ] = 𝐿−1 [ 2𝑠+3 ] = 𝐿−1 [2(𝑠+3−3)+3 ] (𝑠+3)2+16 (𝑠+3)2+16 𝑠2+16 = 𝐿−1 [2(𝑠+3)−6+3 ] (𝑠+3)2+16 = 𝑒−3𝑡𝐿−1 2𝑠−3 [ ] 𝑠 1 = 𝑒−3𝑡 [2𝐿−1 [ ] − 3𝐿−1 [ ]] 𝑠2+16 𝑠2+16 𝐿−1 [ 2𝑠+3 𝑠2+6𝑠+25 4 ] = 𝑒−3𝑡 (2𝑐𝑜𝑠4𝑡 − 3𝑠𝑖𝑛4𝑡 ) (xii) 𝐿−1 [ 𝑠 𝑠2+6𝑠−7 ] = 𝐿−1 [ 𝑠 (𝑠+3)2−16 ] = 𝐿−1 [ 𝑠+3−3 (𝑠+3)2−16 ] 𝑠−3 = 𝑒−3𝑡𝐿−1 [ ] 𝑠2−16 𝑠2−16 𝑠 = 𝑒−3𝑡𝐿−1 [ ] − 3𝑒−3𝑡𝐿−1 [ 1 𝑠2−16 ] 𝐿−1 [ 𝑠 𝑠2+6𝑠−7 4 ] = 𝑒−3𝑡 [𝑐𝑜𝑠ℎ4𝑡 − 3𝑠𝑖𝑛ℎ4𝑡 ] Exercise: 5.7 Find the inverse Laplace transform for the following: Engineering Maths - IV Laplace Transform Page 36
  • 37. 1. 2𝑠−3 𝑠2+52 Ans: 2𝑐𝑜𝑠5𝑡 − 3𝑠𝑖𝑛5𝑡 5 2. 3𝑠+5 Ans: 3𝑐𝑜𝑠4𝑡 + 5𝑠𝑖𝑛4𝑡 4 3. 𝑠2+16 1 6 2 Ans: 1 𝑠𝑖𝑛 3 𝑡 4. 4𝑠2+9 1 (𝑠+4)5 Ans: 4 𝑒−4𝑡 𝑡 4! 5. 1 𝑠2−4𝑠+13 3 𝟐𝒕 Ans: 𝒆 𝑠𝑖𝑛3𝑡 Inverse using the formula 𝑳−𝟏[𝑭(𝒔)] = −𝟏 𝑳−𝟏 [ 𝒅 𝑭(𝒔)] 𝒕 𝒅𝒔 Note: This formula is used when 𝐹(𝑠) is cot−1 ∅(𝑠) or tan−1 ∅(𝑠) or 𝑙𝑜𝑔∅(𝑠) Example: 5.41 Find the inverse Laplace transform for the following (i) 𝒄𝒐𝒕−𝟏 (𝒔 ) (ii) 𝒕𝒂𝒏−𝟏 (𝒂 ) (iii) 𝒄𝒐𝒕−𝟏𝒂𝒔 𝒂 𝒔 (iv) 𝒕𝒂𝒏−𝟏(𝒔 + 𝒂) (v) 𝒍𝒐𝒈(𝒔+𝒂 ) (vi) 𝒄𝒐𝒕−𝟏 ( 𝟐 ) 𝒔+𝒃 𝒔+𝟏 𝒔𝟐 (vii) 𝒕𝒂𝒏−𝟏 ( 𝟐 ) Solution: (i) 𝐿−1 [𝑐𝑜𝑡−1 (𝑠 )] = −1 𝐿−1 [ 𝑑 (𝑐𝑜𝑡−1 (𝑠 ))] 𝑎 𝑡 𝑑𝑠 𝑎 1+𝑠2 𝑎2 𝑡 𝑎 𝑡 𝑎2+𝑠2 𝑎2 𝑎 = −1 𝐿−1 [ −1 (1 )] = 1 𝐿−1 [ −1 (1 )] 𝑡 = 1 𝐿−1 [ 𝑎 𝑠2+𝑎2 ] 𝐿−1 [𝑐𝑜𝑡−1 (𝑠 )] = 1 𝑠𝑖𝑛𝑎𝑡 𝑎 𝑡 (ii) 𝐿−1 [𝑡𝑎𝑛−1 (𝑎 )] = −1 𝐿−1 [ 𝑑 (𝑡𝑎𝑛−1 (𝑎 ))] 𝑠 𝑡 𝑑𝑠 𝑠 𝑡 = −1 𝐿−1 [ 1 1+(𝑠 ) 𝑎 2 𝑠2 𝑡 (−𝑎 )] = −1 𝐿−1 [ 1 𝑠2+𝑎2 𝑠2 𝑠2 (−𝑎 )] 𝑡 = 1 𝐿−1 [ 𝑎 𝑠2+𝑎2 ] 𝐿−1 [𝑡𝑎𝑛−1 (𝑎 )] = 1 𝑠𝑖𝑛𝑎𝑡 𝑠 𝑡 (iii) 𝐿−1[𝑐𝑜𝑡−1𝑎𝑠] = −1 𝐿−1 [ 𝑑 (𝑐𝑜𝑡−1(𝑎𝑠))] 𝑡 𝑑𝑠 𝑡 = −1 𝐿−1 [ −1 1+𝑎2𝑠2 𝑡 (𝑎)] = 1 𝐿−1 [ 𝑎 𝑎2 𝑎2(𝑠2+ 1 ) ] 1 𝑎𝑡 𝑠2+ 1 𝑎2 1 𝑎𝑡 1 𝑎 𝑠𝑖𝑛 𝑡 1 𝑎 = 1 𝐿−1 [ ] = [ ] 𝐿−1[𝑐𝑜𝑡−1𝑎𝑠] = 1 𝑠𝑖𝑛 𝑡 𝑡 𝑎 (iv) 𝐿−1[𝑡𝑎𝑛−1(𝑠 + 𝑎)] = 𝑒−𝑎𝑡𝐿−1[𝑡𝑎𝑛−1𝑠] Engineering Maths - IV Laplace Transform Page 37
  • 38. Engineering Maths - IV = 𝑒−𝑎𝑡 [−1 𝐿−1 [ 𝑑 (𝑡𝑎𝑛−1𝑠)]] 𝑡 𝑑𝑠 1 𝑡 1+𝑠2 = 𝑒−𝑎𝑡 (−1 ) 𝐿−1 [ ] 𝑡 = −1 𝑒−𝑎𝑡𝐿−1 [ 1 1+𝑠2 ] −𝑎𝑡 𝐿−1 [𝑐𝑜𝑡−1 (𝑠 )] = −𝑒 𝑠𝑖𝑛𝑡 𝑎 𝑡 (v) 𝐿−1 [𝑙𝑜𝑔 (𝑠+𝑎 )] = −1 𝐿−1 [ 𝑑 (𝑙𝑜𝑔 (𝑠+𝑎 ))] 𝑠+𝑏 𝑡 𝑑𝑠 𝑠+𝑏 = −1 𝐿−1 [ 𝑑 (log(𝑠 + 𝑎) − log(𝑠 + 𝑏))] 𝑡 𝑑𝑠 − 𝑡 𝑠+𝑎 𝑠+𝑏 = −1 𝐿−1 [ 1 1 ] = −1 [𝑒−𝑎𝑡 − 𝑒−𝑏𝑡] 𝑡 𝐿−1 [𝑙𝑜𝑔 (𝑠+𝑎 )] = −1 [𝑒−𝑎𝑡 − 𝑒−𝑏𝑡] 𝑠+𝑏 𝑡 (vi) 𝐿−1 [𝑐𝑜𝑡−1 ( 2 )] = 𝑒−𝑡𝐿−1 [𝑐𝑜𝑡−1 (2 )] 𝑠+1 𝑠 = 𝑒−𝑡 (−1 ) 𝐿−1 [ 𝑑 (𝑐𝑜𝑡−1 (2 ))] 𝑡 𝑑𝑠 𝑠 𝑡 1+ 4 𝑠2 𝑠2 𝑡 −𝑡 = 𝑒−𝑡 (−1 ) 𝐿−1 [ −1 (−2 )] = − 𝑒 𝐿−1 [ 1 𝑠2+4 𝑠2 𝑠2 ( 2 )] −𝑡 = − 𝑒 𝐿−1 [ 2 𝑡 𝑠2+4 ] −𝑡 𝐿−1 [𝑐𝑜𝑡−1 ( 2 )] = − 𝑒 𝑠𝑖𝑛2𝑡 𝑠+1 𝑡 𝑡 𝑑𝑠 𝑠2 𝑠2 (vii) 𝐿−1 [𝑡𝑎𝑛−1 ( 2 )] = −1 𝐿−1 [ 𝑑 (𝑡𝑎𝑛−1 ( 2 ))] 𝑡 = −1 𝐿−1 [ 1 1+( ) 𝑠2 2 2 𝑠3 𝑡 1 𝑠4+4 𝑠3 𝑠4 (−4 )] = 4 𝐿−1 [ ( 1 )] 𝑠 𝑡 𝑠4+4 = 4 𝐿−1 [ ] 𝑡 = 4 𝐿−1 [ 𝑠 (𝑠2)2+22 ] 𝑡 = 4 𝐿−1 [ 𝑠 (𝑠2+2)2−(2𝑠)2 ] 𝑡 = 4 𝐿−1 [ 𝑠 (𝑠2+2+2𝑠)(𝑠2+2−2𝑠) ] 𝑡 = 4 𝐿−1 [ 𝑠 ( − 1 1 −4𝑠 𝑠2+2+2𝑠 𝑠2+2−2𝑠 )] ∵ { 1 (𝑠2+𝑎𝑥+𝑏)(𝑠2+𝑎𝑥+𝑐) = [ − 1 1 1 𝑐−𝑏 𝑠2+𝑎𝑥+𝑏 𝑠2+𝑎𝑥+𝑐 } ] 𝑡 = −1 𝐿−1 [( − 1 1 𝑠2+2𝑠+2 𝑠2−2𝑠+2 )] 𝑡 = −1 𝐿−1 [ − 1 1 (𝑠+1)2+1 (𝑠−1)2+1 ] ∵ 𝑎2 + 𝑏2 = (𝑎 + 𝑏)2 − 2𝑎𝑏 Laplace Transform Page 38
  • 39. 1 𝑡 𝑠2+1 1 𝑠2+1 = −1 (𝑒−𝑡𝐿−1 [ ] − 𝑒𝑡𝐿−1 [ ]) 𝑡 = −1 (𝑒−𝑡𝑠𝑖𝑛𝑡 − 𝑒𝑡𝑠𝑖𝑛𝑡) 𝑡 = 𝑠𝑖𝑛𝑡 (𝑒−𝑡 − 𝑒𝑡) 𝑡 = 𝑠𝑖𝑛𝑡 2𝑠𝑖𝑛ℎ𝑡 𝑠2 𝐿−1 [𝑡𝑎𝑛−1 ( 2 )] = 2𝑠𝑖𝑛𝑡𝑠𝑖𝑛ℎ𝑡 𝑡 Inverse using the formula 𝒅𝒕 𝑳−𝟏[𝒔𝑭(𝒔)] = 𝒅 𝑳−𝟏[𝑭(𝒔)] 𝒔𝟐+𝒃𝟐 𝟐 𝟐 Example: 5.42 Find 𝑳−𝟏 [𝒔𝒍𝒐𝒈 (𝒔 +𝒂 )] Solution: 2 2 2 2 𝐿−1 [𝑠𝑙𝑜𝑔 (𝑠 +𝑎 )] = 𝑑 𝐿−1 [𝑠𝑙𝑜𝑔 (𝑠 +𝑎 )] ⋯ (1) 𝑠2+𝑏2 𝑑𝑡 𝑠2+𝑏2 2 2 2 2 𝐿−1 [𝑙𝑜𝑔 (𝑠 +𝑎 )] = 𝐿−1 𝑑 [𝑙𝑜𝑔 (𝑠 +𝑎 )] 𝑠2+𝑏2 𝑑𝑠 𝑠2+𝑏2 = −1 𝐿−1 [ 𝑑 (log(𝑠2 + 𝑎2) − log(𝑠2 + 𝑏2))] 𝑡 𝑑𝑠 𝑡 = −1 𝐿−1 [ 𝑠2+𝑎2 1 1 𝑠2+𝑏2 2𝑠 − 2𝑠] 𝑡 = −2 𝐿−1 [ − 𝑠 𝑠 𝑠2+𝑎2 𝑠2+𝑏2 ] 𝑡 = −2 [𝑐𝑜𝑠𝑎𝑡 − 𝑐𝑜𝑠𝑏𝑡] = 2 [𝑐𝑜𝑠𝑏𝑡 − 𝑐𝑜𝑠𝑎𝑡] 𝑡 Substituting in (1), we get 𝑡2 2 2 𝐿−1 [𝑠𝑙𝑜𝑔 (𝑠 +𝑎 )] = 𝑑 [2 [𝑐𝑜𝑠𝑏𝑡 − 𝑐𝑜𝑠𝑎𝑡]] 𝑠2+𝑏2 𝑑𝑡 𝑡 = 2 [𝑡(−𝑏𝑠𝑖𝑛𝑏𝑡+𝑎𝑠𝑖𝑛𝑎𝑡)−(𝑐𝑜𝑠𝑏𝑡−𝑐𝑜𝑠𝑎𝑡 ) ] 𝑡2 2 2 𝐿−1 [𝑠𝑙𝑜𝑔 (𝑠 +𝑎 )] = 2 [𝑡(−𝑏𝑠𝑖𝑛𝑏𝑡+𝑎𝑠𝑖𝑛𝑎𝑡)−(𝑐𝑜𝑠𝑏𝑡−𝑐𝑜𝑠𝑎𝑡) ] 𝑠2+𝑏2 Inverse using the formula 𝑳−𝟏 [𝑭(𝒔) ] = 𝒔 𝒕 −𝟏 ∫𝟎 𝑳 [𝑭(𝒔)]𝒅𝒕 This formula is used when 𝐹(𝑠) = 𝑜𝑛𝑒 𝑡𝑒𝑟𝑚 𝑠(𝑎𝑛𝑜𝑡ℎ𝑒𝑟 𝑡𝑒𝑟𝑚) 𝒔(𝒔𝟐+𝒂𝟐) Example: 5.43 Find 𝑳−𝟏 [ 𝟏 ] Solution: 𝐿−1 [ 1 𝑠(𝑠2+𝑎2) 1 (𝑠2+𝑎2) ] 𝑑𝑡 𝑡 𝐿−1 [ ] = ∫0 𝑎 𝑡 [𝑠𝑖𝑛𝑎𝑡 ] 𝑑𝑡 = ∫0 Engineering Maths - IV Laplace Transform Page 39
  • 40. = [ 1 −𝑐𝑜𝑠𝑎𝑡 𝑎 𝑎 𝑡 ] 0 = −1 [𝑐𝑜𝑠𝑎𝑡]𝑡 𝑎2 0 = −1 (𝑐𝑜𝑠𝑎𝑡 − 𝑐𝑜𝑠0) = −1 (𝑐𝑜𝑠𝑎𝑡 − 1) 𝑎2 𝑎2 𝑠(𝑠2+𝑎2) 1 1−𝑐𝑜𝑠𝑎𝑡 ∴ 𝐿−1 [ ] = 𝑎2 𝒔(𝒔𝟐−𝒂𝟐) Example: 5.44 Find 𝑳−𝟏 [ 𝟏 ] Solution: 𝐿−1 [ 1 𝑠(𝑠2+𝑎2) 1 (𝑠2−𝑎2) ] 𝑑𝑡 𝑡 𝐿−1 [ ] = ∫0 𝑎 𝑡 [𝑠𝑖𝑛ℎ𝑎𝑡 ] 𝑑𝑡 = ∫0 = [ 1 𝑐𝑜𝑠ℎ𝑎𝑡 𝑎 𝑎 𝑡 ] 0 = 1 [𝑐𝑜𝑠ℎ𝑎𝑡]𝑡 𝑎2 0 = 1 (𝑐𝑜𝑠ℎ𝑎𝑡 − 𝑐𝑜𝑠ℎ0) = 1 (𝑐𝑜𝑠ℎ𝑎𝑡 − 1) 𝑎2 𝑎2 𝑠(𝑠2−𝑎2) 1 𝑐𝑜𝑠ℎ𝑎𝑡−1 ∴ 𝐿−1 [ ] = 𝑎2 𝒔(𝒔+𝒂) Example: 5.45 Find 𝑳−𝟏 [ 𝟏 ] Solution: 𝐿−1 [ 1 𝑠(𝑠+𝑎) 1 (𝑠+𝑎) ] 𝑑𝑡 𝑡 𝐿−1 [ ] = ∫0 𝑡 𝑒−𝑎𝑡𝑑𝑡 = ∫0 −𝑎 = [ ] 0 𝑒−𝑎𝑡 𝑡 𝑎 = −1 (𝑒−𝑎𝑡 − 1) ∴ 𝐿−1 [ 1 ] = 1−𝑒−𝑎𝑡 𝑠(𝑠+𝑎) 𝑎 Inverse using Partial Fraction Example: 5.46 Find 𝒔(𝒔+𝟐)(𝒔−𝟏) 𝒔−𝟐 𝑳−𝟏 [ ] Solution: = 𝐴 + + 𝑠−2 𝐵 𝐶 𝑠(𝑠+2)(𝑠−1) 𝑠 𝑠+2 𝑠−1 = 𝐴(𝑠+2)(𝑠−1)+𝐵𝑠(𝑠−1)+𝐶𝑠(𝑠+2) 𝑠(𝑠+2)(𝑠−1) 𝐴(𝑠 + 2)(𝑠 − 1) + 𝐵𝑠(𝑠 − 1) + 𝐶𝑠(𝑠 + 2) = 𝑠 − 2 ⋯ (1) Put 𝑠 = 0 in (1) Put 𝑠 = −2 in (1) Put 𝑠 = 1 in (1) 𝐴(2)(−1) = −2 𝐵(−2)(−3) = −4 3𝐶 = −1 Engineering Maths - IV Laplace Transform Page 40
  • 41. ⇒ 𝐴 = 1 ⇒ 𝐵 = −4 = −2 6 3 ⇒ 𝐶 = −1 3 ∴ 𝑠−2 2 = 1 − − 𝑠(𝑠+2)(𝑠−1) 𝑠 𝑠+2 3(𝑠−1) 1 𝑠−2 1 2 1 1 1 𝐿−1 [ ] = 𝐿−1 [ ] − 𝐿−1 [ ] − 𝐿−1 [ ] 𝑠(𝑠+2)(𝑠−1) 𝑠 3 𝑠+2 3 𝑠−1 𝐿−1 [ 𝑠−2 ] = 1 − 2 𝑒−2𝑡 − 1 𝑒𝑡 𝑠(𝑠+2)(𝑠−1) 3 3 Example: 5.47 Find 𝐿−1 [ 2𝑠−3 (𝑠−1)(𝑠−2)2 ] Solution: 2𝑠−3 𝐴 𝐵 = + + (𝑠−1)(𝑠−2)2 𝑠−1 𝑠−2 (𝑠−2)2 𝐶 2 = 𝐴(𝑠−2) +𝐵(𝑠−1)(𝑠−2)+𝐶(𝑠−1) (𝑠−1)(𝑠−2)2 𝐴(𝑠 − 2)2 + 𝐵(𝑠 − 1)(𝑠 − 2) + 𝐶(𝑠 − 1) = 2𝑠 − 3 ⋯ (1) Put 𝑠 = 1 in (1) Put 𝑠 = 2 in (1) Equating the coefficient of 𝑠2 𝐴 = −1 𝐶 = 1 𝐴 + 𝐵 = 0 𝐵 = −𝐴 ⇒ 𝐵 = 1 ∴ 2𝑠−3 = −1 + 1 + 1 (𝑠−1)(𝑠−2)2 𝑠−1 𝑠−2 (𝑠−2)2 2𝑠−3 1 1 1 𝐿−1 [ ] = −𝐿−1 [ ] + 𝐿−1 [ ] + [ ] (𝑠−1)(𝑠−2)2 𝑠−1 𝑠−2 (𝑠−2)2 𝑠2 = −𝑒𝑡 + 𝑒2𝑡 + 𝑒2𝑡𝐿−1 [ 1 ] (𝑠−1)(𝑠−2)2 2𝑠−3 ∴ 𝐿−1 [ ] = −𝑒𝑡 + 𝑒2𝑡 + 𝑒2𝑡𝑡 𝟐 Example: 5.48 Find the inverse Laplace transform of 𝟓𝒔 −𝟏𝟓𝒔−𝟏𝟏 (𝒔+𝟏)(𝒔−𝟐)𝟑 Solution: 5𝑠2−15𝑠−11 = 𝐴 𝐵 + + + 𝐶 𝐶 (𝑠+1)(𝑠−2)3 𝑠+1 𝑠−2 (𝑠−2)2 (𝑠−2)3 3 2 = 𝐴(𝑠−2) +𝐵(𝑠+1)(𝑠−2) +𝐶(𝑠+1)(𝑠−2)+𝐷(𝑠+1) (𝑠−1)(𝑠−2)3 𝐴(𝑠 − 2)3 + 𝐵(𝑠 + 1)(𝑠 − 2)2 + 𝐶(𝑠 + 1)(𝑠 − 2) + 𝐷(𝑠 + 1) = 5𝑠2 − 15𝑠 − 11⋯ (1) Put 𝑠 = −1 in (1) 𝐴(−27) = 9 Put 𝑠 = 2 in (1) 𝐷(3) = −21 Equating the coefficient of 𝑠3 𝐴 + 𝐵 = 0 𝐴 = 9 −27 ⇒ 𝐴 = −1 3 3 𝐷 = −21 = −7 𝐵 = −𝐴 ⇒ 𝐵 = 1 3 Put 𝑠 = 0 in (1), we get −8𝐴 + 4𝐵 − 2𝐶 + 𝐷 = −11 Engineering Maths - IV Laplace Transform Page 41
  • 42. 8 + 4 − 2𝐶 − 7 = −11 3 3 4 − 2𝐶 = 7 − 11 −2𝐶 = −8 ⇒ 𝐶 = 4 2 ∴ 5𝑠 −15𝑠−11 = −1 + + − 1 4 7 (𝑠+1)(𝑠−2)3 3(𝑠+1) 3(𝑠−2) (𝑠−2)2 (𝑠−2)3 2 𝐿−1 [5𝑠 −15𝑠−11 ] = −1 𝐿−1 [ 1 ] + 1 𝐿−1 [ 1 ] + 4𝐿−1 [ (𝑠+1)(𝑠−2)3 3 𝑠+1 3 𝑠−2 1 (𝑠−2)2 ] − 7𝐿−1 [ 1 (𝑠−2)3 ] 3 3 𝑠2 𝑠3 = −1 𝑒−𝑡 + 1 𝑒2𝑡 + 4𝑒2𝑡 𝐿−1 [ 1 ] − 7𝑒2𝑡 𝐿−1 [ 1 ] 𝑠2 2 2 𝐿−1 [5𝑠 −15𝑠−11 ] = −1 𝑒−𝑡 + 1 𝑒2𝑡 + 4𝑒2𝑡 𝐿−1 [ 1 ] − 7𝑒2𝑡 𝑡 (𝑠+1)(𝑠−2)3 3 3 2 Example: 5.49 Find the inverse Laplace transform of 𝟒𝒔+𝟓 (𝒔+𝟏)(𝒔𝟐+𝟒) Solution: 4𝑠+5 = 𝐴 + 𝐵𝑠+𝑐 (𝑠+1)(𝑠2+4) 𝑠+1 𝑠2+4 2 = 𝐴(𝑠 +4)+(𝐵𝑠+𝑐)(𝑠+1) (𝑠+1)(𝑠2+4) 𝐴(𝑠2 + 4) + (𝐵𝑠 + 𝑐)(𝑠 + 1) = 4𝑠 + 5 ⋯ ⋯ (1) Put 𝑠 = −1in (1) 𝐴(1 + 4) + 0 = 4(−1) + 5 Equating coefficients of 𝑠2term in (1) 𝐴 + 𝐵 = 0 Put 𝑠 = 0in (1) 𝐴(4) + 𝐶 = 5 𝐴(5) = 1 ⇒ 𝐴 = 1 5 𝐵 = −𝐴 ⇒ 𝐵 = −1 5 𝐶 = 5 − 4𝐴 = 5 − 4 5 = 25−4 = 21 5 5 ∴ 4𝑠+5 = (𝑠+1)(𝑠2+4) 𝑠+1 −1 21 1 𝑠+ 5 + 5 5 𝑠2+4 = − 1 𝑠 + 21 1 5(𝑠+1) 5(𝑠2+4) 5 (𝑠2+4) 𝐿−1 [ 4𝑠+5 𝑠 (𝑠+1)(𝑠2+4) 5 𝑠+1 5 𝑠2+4 5 1 𝑠2+4 ] = 1 𝐿−1 [ 1 ] − 1 𝐿−1 [ ] + 21 𝐿−1 [ ] = 1 𝑒−𝑡 − 1 𝑐𝑜𝑠2𝑡 + 21 𝑠𝑖𝑛2𝑡 5 5 5 2 (𝑠+1)(𝑠2+4) 5 5 10 4𝑠+5 1 1 21 𝐿−1 [ ] = 𝑒−𝑡 − 𝑐𝑜𝑠2𝑡 + 𝑠𝑖𝑛2𝑡 Exercise: 5.8 Find the Inverse Laplace transforms using partial fraction for the following 1. 1 (𝑠+1)(𝑠+3) Ans: 2 1 (𝑒−𝑡 − 𝑒−3𝑡) 2. 1 𝑠(𝑠+1)(𝑠+2) Ans: 2 1 (𝑒−2𝑡 − 2𝑒−𝑡 + 1) 3. 54−3𝑠−5 (𝑠+1)(𝑠2−3𝑠+2) Ans: 2𝑒−𝑡 3𝑡 3𝑡 𝑡 𝑡 + 2 𝑒 2 𝑐𝑜𝑠ℎ 2 +8𝑒 2 𝑠𝑖𝑛ℎ 2 2 5. 7 INITIALAND FINAL VALUE THEOREMS Initial value theorem Engineering Maths - IV Laplace Transform Page 42
  • 43. 𝑡→0 𝑠→∞ Proof: We know that 𝐿[𝑓′(𝑡)] = 𝑠 𝐿[𝑓(𝑡)] − 𝑓(0) = 𝑠𝐹(𝑠) − 𝑓(0) ∴ 𝑠𝐹(𝑠) = 𝐿[𝑓′(𝑡)] + 𝑓(0) = ∫ ∞ 𝑒−𝑠𝑡𝑓′(𝑡)𝑑𝑡 + 𝑓(0) 0 Taking limit as 𝑠 → ∞ on both sides, we have Engineering Maths - IV Statement: If𝐿[𝑓(𝑡)] = 𝐹(𝑠), then lim 𝑓(𝑡) = lim𝑠𝐹(𝑠) 𝑠→∞ 𝑠→∞ lim𝑠𝐹(𝑠) = lim[∫0 ∞ 𝑒−𝑠𝑡𝑓′(𝑡)𝑑𝑡 + 𝑓(0)] 𝑠→∞ 0 ∞ = lim[∫ 𝑒−𝑠𝑡 𝑓′ (𝑡)𝑑𝑡] + 𝑓(0) 𝑠→∞ ∞ lim[𝑒−𝑠𝑡𝑓′(𝑡)]𝑑𝑡 + 𝑓(0) = ∫0 ∵ 𝑒−∞ = 0 𝑠→∞ 𝑡→0 = 0 + 𝑓(0) = 𝑓(0) = lim𝑓(𝑡) 𝑡→0 ∴ lim𝑠𝐹(𝑠) = lim𝑓(𝑡) 𝑡→∞ 𝑠→0 Final value theorem Statement: If the Laplace transforms of 𝑓(𝑡) and 𝑓′(𝑡) exist and 𝐿[𝑓(𝑡)] = 𝐹(𝑠),then lim𝑓(𝑡) = lim𝑠𝐹(𝑠) Proof: We know that 𝐿[𝑓′(𝑡)] = 𝑠 𝐿[𝑓(𝑡)] − 𝑓(0) = 𝑠𝐹(𝑠) − 𝑓(0) ∴ 𝑠𝐹(𝑠) = 𝐿[𝑓′(𝑡)] + 𝑓(0) = ∫ ∞ 𝑒−𝑠𝑡𝑓′(𝑡)𝑑𝑡 + 𝑓(0) 0 Taking limit as 𝑠 → 0 on both sides, we have 𝑠→0 𝑠→0 0 lim𝑠𝐹(𝑠) = lim[∫ ∞ 𝑒−𝑠𝑡𝑓′(𝑡)𝑑𝑡 + 𝑓(0)] = lim[ 𝑠→0 0 ∞ ∫ 𝑒−𝑠𝑡𝑓′(𝑡)𝑑𝑡] + 𝑓(0) 𝑠→0 ∞ lim[𝑒−𝑠𝑡𝑓′(𝑡)]𝑑𝑡 + 𝑓(0) = ∫0 ∞ 𝑓′(𝑡)𝑑𝑡 + 𝑓(0) = ∫0 = [𝑓(𝑡)]∞ + 𝑓(0) 𝑡→∞ 𝑠→0 0 = 𝑓(∞) − 𝑓(0) + 𝑓(0) = 𝑓(∞) = lim𝑓(𝑡) 𝑡→∞ ∴ lim𝑓(𝑡) = lim𝑠𝐹(𝑠) Example: 5.50 Verify the initial value theorem for the function 𝒇(𝒕) = 𝒂𝒆−𝒃𝒕 Laplace Transform Page43
  • 44. Engineering Maths - IV Solution: Given 𝑓(𝑡) = 𝑎𝑒−𝑏𝑡 𝐹(𝑠) = 𝐿[𝑓(𝑡)] = 𝐿[𝑎𝑒−𝑏𝑡] = 𝑎 1 𝑠+𝑏 𝑠𝐹(𝑠) = 𝑎𝑠 𝑠+𝑏 Initial value theorem is lim𝑓(𝑡) = lim𝑠𝐹(𝑠) 𝑡→0 𝑠→∞ 𝑡→0 lim𝑓(𝑡) = lim 𝑎𝑒−𝑏𝑡 𝑡→0 = 𝑎 ⋯ ⋯ ⋯ (1) 𝑠→∞ 𝑠→∞ 𝑠+𝑏 𝑎𝑠 lim𝑠𝐹(𝑠) = lim [ ] = lim [ 𝑎𝑠 𝑏 𝑠→∞ 𝑠(1+𝑠 ) 𝑎 𝑏 𝑠→∞ (1+𝑠 ) ] == lim [ ] 𝑡→0 𝑠→∞ = 𝑎 ⋯ ⋯ ⋯ (2) From (1) and (2), lim𝑓(𝑡) = lim𝑠𝐹(𝑠) ∴ Initial value theorem is verified Example: 5.51Verify the initial value theorem and Final value theorem for the function 𝒇(𝒕) = 𝟏 + 𝒆−𝒕[𝒔𝒊𝒏𝒕 + 𝒄𝒐𝒔𝒕]. Solution: Given 𝑓(𝑡) = 1 + 𝑒−𝑡[𝑠𝑖𝑛𝑡 + 𝑐𝑜𝑠𝑡] 𝐹(𝑠) = 𝐿[𝑓(𝑡)] = 𝐿[1 + 𝑒−𝑡[𝑠𝑖𝑛𝑡 + 𝑐𝑜𝑠𝑡]] = 𝐿[1] + 𝐿[𝑒−𝑡[𝑠𝑖𝑛𝑡 + 𝑐𝑜𝑠𝑡]] = 𝐿[1] + 𝐿[𝑠𝑖𝑛𝑡 + 𝑐𝑜𝑠𝑡]𝑠→𝑠+1 𝑠 𝑠2+1 1 𝑠 = 1 + [ + ] 𝑠2+1 𝑠→𝑠+1 = 1 + + 1 𝑠+1 𝑠 (𝑠+1)2+1 (𝑠+1)2+1 𝐹(𝑠) = 1 + + 1 𝑠+1 𝑠 𝑠2+2𝑠+2 𝑠2+2𝑠+2 𝑠𝐹(𝑠) = 1 + + 𝑠 𝑠2+𝑠 𝑠2+2𝑠+2 𝑠2+2𝑠+2 Initial value theorem is lim𝑓(𝑡) = lim𝑠𝐹(𝑠) 𝑡→0 𝑠→∞ lim𝑓(𝑡) = lim[1 + 𝑒−𝑡[𝑠𝑖𝑛𝑡 + 𝑐𝑜𝑠𝑡]] 𝑡→0 𝑡→0 = 1 + 0 + 1 = 2 ⋯ ⋯ ⋯ (1) lim𝑠𝐹(𝑠) = lim [1 + 𝑠→∞ 𝑠→∞ + 𝑠 𝑠2+𝑠 𝑠2+2𝑠+2 𝑠2+2𝑠+2 ] Laplace Transform Page44
  • 45. = 1 + lim [ 1 1 (1+𝑠 ) 2 2 2 2 𝑠→∞ 𝑠(1+𝑠+𝑠2) (1+𝑠+𝑠2) + ] 𝑡→0 𝑠→∞ = 1 + 0 + 1 = 2 ⋯ ⋯ ⋯ (2) From (1) and (2), lim𝑓(𝑡) = lim𝑠𝐹(𝑠) 𝑡→∞ 𝑠→0 ∴ Initial value theorem is verified Final value theorem is lim𝑓(𝑡) = lim𝑠𝐹(𝑠) lim𝑓(𝑡) = lim(1 + 𝑒−𝑡[𝑠𝑖𝑛𝑡 + 𝑐𝑜𝑠𝑡]) 𝑡→∞ 𝑡→∞ = 1 + 0 = 1 ⋯ ⋯ ⋯ (3) 𝑠→0 𝑠→0 lim𝑠𝐹(𝑠) = lim [1 + + 𝑠 𝑠2+𝑠 𝑠2+2𝑠+2 𝑠2+2𝑠+2 ] 𝑡→∞ 𝑠→0 = 1 + 0 + 0 = 1 ⋯ ⋯ ⋯ (4) From (3) and (4), lim𝑓(𝑡) = lim𝑠𝐹(𝑠) ∴ Final value theorem is verified. Example: 5.52 Verify the initial value theorem and Final value theorem for the function 𝒔(𝒔+𝟐)𝟐 𝟏 𝒇(𝒕) = 𝑳−𝟏 [ ] Solution: 𝑠(𝑠+2)2 1 Given 𝑓(𝑡) = 𝐿−1 [ ] ⋯ (1) (𝑠+2)2 𝑡 −1 1 = ∫ 0 𝐿 [ ] 𝑑𝑡 = 𝑠2 𝑡 −2𝑡 −1 1 ∫ 0 𝑒 𝐿 [ ] 𝑑𝑡 𝑡 −2𝑡 = ∫ 0 𝑒 𝑡 𝑑𝑡 = ∫ 𝑡 𝑡𝑒−2𝑡 𝑑𝑡 0 𝑒−2𝑡 = [𝑡 ( ) − ] −2 (−2)2 0 (1)𝑒−2𝑡 𝑡 −2𝑡 −2𝑡 = −𝑡 𝑒 − 𝑒 − 0 + 1 2 4 4 −2𝑡 −2𝑡 ∴ 𝑓(𝑡) = 1 − 𝑡𝑒 − 𝑒 4 2 4 From (1), 𝐹(𝑠) = 1 𝑠(𝑠+2)2 1 𝑠𝐹(𝑠) = (𝑠+2)2 Initial value theorem is lim𝑓(𝑡) = lim𝑠𝐹(𝑠) 𝑡→0 𝑠→∞ 𝑡→0 𝑡→0 4 2 4 −2𝑡 −2𝑡 lim𝑓(𝑡) = lim [1 − 𝑡𝑒 − 𝑒 ] = 1 − 0 − 1 = 0 4 4 ∴ lim𝑓(𝑡) = 0 ⋯ (2) 𝑡→0 𝑠→∞ lim𝑠𝐹(𝑠) = lim 𝑠→∞ (𝑠+2)2 1 = 0 Engineering Maths - IV Laplace Transform Page45
  • 46. 𝑡→0 𝑠→∞ 𝑡→∞ 𝑠→0 ∴ Initial value theorem is verified Final value theorem is lim𝑓(𝑡) = lim𝑠𝐹(𝑠) Engineering Maths - IV ∴ lim𝑠𝐹(𝑠) = 0 ⋯ (3) 𝑠→∞ From (2) and (3), lim𝑓(𝑡) = lim𝑠𝐹(𝑠) 𝑡→∞ 𝑡→∞ 4 2 4 −2𝑡 −2𝑡 lim𝑓(𝑡) = lim [1 − 𝑡𝑒 − 𝑒 ] = 1 − 0 − 0 = 1 ⋯ (4) 4 4 𝑠→0 𝑠→0 (𝑠+2)2 lim𝑠𝐹(𝑠) = lim [ 1 ] = 1 ⋯ (5) 𝑡→∞ 𝑠→0 4 From (4) and (5), lim𝑓(𝑡) = lim𝑠𝐹(𝑠) ∴ Final value theorem is verified Example: 5.53 Verify the initial value theorem and Final value theorem for the function 𝒇(𝒕) = 𝒆−𝒕(𝒕 + 𝟐)𝟐 Solution: Given 𝑓(𝑡) = 𝑒−𝑡(𝑡 + 2)2 = 𝑒−𝑡(𝑡2 + 4𝑡 + 4) 𝐹(𝑠) = 𝐿[𝑓(𝑡)] = 𝐿[𝑒−𝑡(𝑡2 + 4𝑡 + 4)] = 𝐿[𝑡2 + 4𝑡 + 4]𝑠→𝑠+1 = [𝐿(𝑡2) + 4𝐿(𝑡) + 4𝐿(1)]𝑠→𝑠+1 𝑠3 𝑠2 = [2! + 4 1 + 4 1 ] 𝑠 𝑠→𝑠+1 = 2 1 + 4 + 4 (𝑠+1)3 (𝑠+1)2 𝑠+1 1 𝑠𝐹(𝑠) = + 2𝑠 4𝑠 + 4𝑠 (𝑠+1)3 (𝑠+1)2 𝑠+1 Initial value theorem is lim𝑓(𝑡) = lim𝑠𝐹(𝑠) 𝑡→0 𝑡→0 𝑠→∞ lim𝑓(𝑡) = lim[𝑒−𝑡(𝑡2 + 4𝑡 + 4)] 𝑡→0 = 4 ⋯ (1) 𝑠→∞ 2𝑠 lim𝑠𝐹(𝑠) = lim [ + 𝑠→∞ (𝑠+1)3 (𝑠+1)2 𝑠+1 4𝑠 + 4𝑠 ] 𝑠→∞ 3 𝑠 𝑠 (1+ ) 1 3 2𝑠 4𝑠 = lim [ + 2 𝑠 (1+𝑠 ) 1 2 + 4𝑠 1 𝑠(1+𝑠 ) ] 𝑠→∞ 2 𝑠 𝑠 (1+ ) 1 3 2 = lim [ + 4 𝑠(1+𝑠 ) 1 2 + 4 1 (1+𝑠 ) ] = 0 + 0 + 4 Laplace Transform Page46
  • 47. 𝑡→0 𝑠→∞ 𝑡→∞ 𝑠→0 ∴ Initial value theorem is verified Final value theorem is lim𝑓(𝑡) = lim𝑠𝐹(𝑠) Engineering Maths - IV = 4 ⋯ (2) From (1) and (2), lim𝑓(𝑡) = lim𝑠𝐹(𝑠) lim𝑓(𝑡) = lim[𝑒−𝑡(𝑡2 + 4𝑡 + 4)] 𝑡→∞ 𝑡→∞ 𝑠→0 𝑠→0 = 0 ⋯ (3) 2𝑠 lim𝑠𝐹(𝑠) = lim [ + (𝑠+1)3 (𝑠+1)2 𝑠+1 4𝑠 + 4𝑠 ] = 0 ⋯ (4) From (3) and (4), lim𝑓(𝑡) = lim𝑠𝐹(𝑠) 𝑡→∞ 𝑠→0 ∴ Final value theorem is verified. Example: 5.54 If 𝑳[𝒇(𝒕)] = 𝟏 𝒔(𝒔+𝟏) , find the 𝐥𝐢𝐦𝒇(𝒕) and 𝐥𝐢𝐦𝒇(𝒕) using initial and final value 𝒕→𝟎 𝒕→∞ theorems. Solution: Given 𝐿[𝑓(𝑡)] = 1 𝑠(𝑠+1) ⋯ (1) 𝑖𝑒. , 𝐹(𝑠) = 1 => 𝑠𝐹(𝑠) = 𝑠(𝑠+1) (𝑠+1) 1 Initial value theorem is lim𝑓(𝑡) = lim𝑠𝐹(𝑠) 𝑡→0 𝑠→∞ = lim 1 𝑠→∞ (𝑠+1) = 0 Final value theorem is lim𝑓(𝑡) = lim𝑠𝐹(𝑠) 𝑡→∞ 𝑠→0 = lim 1 𝑠→0 (𝑠+1) = 1 Exercise: 5.9 1. Verify the initial value theorem for the function 2. Verify the initial value theorem for the function 3. Verify the initial value theorem for the function 𝑓(𝑡) = 𝑒−𝑡𝑠𝑖𝑛𝑡 𝑓(𝑡) = sin2 𝑡 𝑓(𝑡) = 1 + 𝑒−𝑡 + 𝑡2 4. Verify the Final value theorem for the function 𝑓(𝑡) = 1 − 𝑒−𝑎𝑡 5. Verify the Final value theorem for the function 𝑓(𝑡) = 𝑡2𝑒−3𝑡 5.8 CONVOLUTION THEOREM Definition: Convolution of two functions The convolution of two functions 𝑓(𝑡) and 𝑔(𝑡) is denoted by 𝑓(𝑡) ∗ 𝑔(𝑡) and defined by 𝑡 𝑓(𝑢)𝑔(𝑡 − 𝑢)𝑑𝑢. 𝑓(𝑡) ∗ 𝑔(𝑡) = ∫0 State and prove Convolution theorem Statement: If 𝐿[𝑓(𝑡)] = 𝐹(𝑠) and 𝐿[𝑔(𝑡)] = 𝐺(𝑠), then 𝐿[𝑓(𝑡)] ∗ 𝐿[𝑔(𝑡)] = 𝐹(𝑠)𝐺(𝑠) Laplace Transform Page47
  • 48. Engineering Maths - IV Proof: 𝑡 𝑓(𝑢)𝑔(𝑡 − 𝑢)𝑑𝑢 We have 𝑓(𝑡) ∗ 𝑔(𝑡) = ∫0 𝐿[𝑓(𝑡) ∗ 𝑔(𝑡)] = ∫ ∞ [𝑓(𝑡) ∗ 𝑔(𝑡)] 𝑒−𝑠𝑡𝑑𝑡 0 ∞ 𝑡 𝑓(𝑢)𝑔(𝑡 − 𝑢)𝑑𝑢𝑒−𝑠𝑡𝑑𝑡 = ∫0 ∫0 = ∫0 ∫0 ∞ 𝑡 𝑓(𝑢)𝑔(𝑡 − 𝑢)𝑒−𝑠𝑡 𝑑𝑢𝑑𝑡 ⋯ (1) Now we have no change the order of integration. 𝑢 = 0, 𝑢 = 𝑡;𝑡 = 0, 𝑡 = ∞ Change of order is . Draw horizontal strip PQ At P, 𝑡 = 𝑢, At A 𝑢 = ∞ ∞ ∞ 𝑓(𝑢)𝑔(𝑡 − 𝑢)𝑒−𝑠𝑡𝑑𝑡𝑑𝑢 𝐿[𝑓(𝑡) ∗ 𝑔(𝑡)] = ∫0 ∫𝑢 ∞ = ∫ 0 𝑓(𝑢) [∫ 𝑢 ∞ 𝑔(𝑡 − 𝑢)𝑒−𝑠𝑡 𝑑𝑡]𝑑𝑢 ⋯ (2) Put 𝑡 − 𝑢 = 𝑥 ⋯ (3) 𝑡 = 𝑢 + 𝑥 ⇒ 𝑑𝑡 = 𝑑𝑥 When 𝑡 = 𝑢;(3) ⇒ 𝑥 = 0 When 𝑡 = ∞; (3) ⇒ 𝑥 = ∞ (2) ⇒ 𝐿[𝑓(𝑡) ∗ 𝑔(𝑡)] = ∫0 ∞ 𝑓(𝑢) [∫0 ∞ 𝑔(𝑥)𝑒−𝑠(𝑢+𝑥)𝑑𝑥]𝑑𝑢 ∞ = ∫ 0 𝑓(𝑢) [∫ 0 ∞ 𝑔(𝑥)𝑒−𝑠𝑢𝑒−𝑠𝑥𝑑𝑥]𝑑𝑢 ∞ 𝑔(𝑥)𝑒−𝑠𝑥𝑑𝑥 ∞ −𝑠𝑢 = ∫ 0 𝑓(𝑢)𝑒 𝑑𝑢 ∫ 0 = 𝐿[𝑓(𝑢)]𝐿[𝑔(𝑥)] ∴ 𝐿[𝑓(𝑡) ∗ 𝑔(𝑡)] = 𝐹(𝑠)𝐺(𝑠) Note: Convolution theorem is very useful to compute inverse Laplace transform of product of two terms Convolution theorem is 𝐿[𝑓(𝑡) ∗ 𝑔(𝑡)] = 𝐹(𝑠)𝐺(𝑠) 𝐿−1[𝐹(𝑠)𝐺(𝑠)] = 𝑓(𝑡) ∗ 𝑔(𝑡) 𝐿−1[𝐹(𝑠)𝐺(𝑠)] = 𝐿−1[𝐹(𝑠)] ∗ 𝐿−1[𝐺(𝑠)] Problems under Convolution theorem Example: 5.55 Find 𝑳−𝟏 [ 𝟏 (𝒔+𝒂)(𝒔+𝒃) ] using convolution theorem. Solution: 𝐿−1 1 1 1 [ ] = 𝐿−1 [ ] ∗ 𝐿−1 [ ] (𝑠+𝑎)(𝑠+𝑏) (𝑠+𝑎) (𝑠+𝑏) = 𝑒−𝑎𝑡 ∗ 𝑒−𝑏𝑡 = ∫0 𝑡 𝑒−𝑎𝑢𝑒−𝑏(𝑡−𝑢) 𝑑𝑢 ∫0 = 𝑒−𝑏𝑡 𝑡 𝑒−𝑎𝑢 𝑒𝑏𝑢𝑑𝑢 ∫0 = 𝑒−𝑏𝑡 𝑡 𝑒(𝑏−𝑎)𝑢 𝑑𝑢 Laplace Transform Page48
  • 49. −𝑏𝑡 = 𝑒 [ 𝑏−𝑎 ] 0 𝑒(𝑏−𝑎)𝑢 𝑡 −𝑏𝑡 = 𝑒 [𝑒(𝑏−𝑎)𝑡 − 1 ] 𝑏−𝑎 𝑏−𝑎 −𝑏𝑡 = 𝑒 [𝑒𝑏𝑡−𝑎𝑡 − 1 ] = 1 𝑏−𝑎 [𝑒−𝑏𝑡+𝑏𝑡−𝑎𝑡 − 𝑒−𝑏𝑡 ] ∴ 𝐿−1 [ ] = 1 1 (𝑠+𝑎)(𝑠+𝑏) 𝑏−𝑎 [𝑒−𝑎𝑡 − 𝑒−𝑏𝑡 ] Example: 5.56 Find the inverse Laplace transform 𝒔𝟐 (𝒔𝟐+𝒂𝟐)(𝒔𝟐+𝒃𝟐) by using convolution theorem. Solution: 𝐿−1 [ 𝑠2 (𝑠2+𝑎2)(𝑠2+𝑏2) 𝑠 𝑠 ] = 𝐿−1 [ (𝑠2+𝑎2) (𝑠2+𝑏2) ] 𝑠 𝑠 = 𝐿−1 [ ] ∗ 𝐿−1 [ ] (𝑠2+𝑎2) (𝑠2+𝑏2) = 𝑐𝑜𝑠𝑎𝑡 ∗ 𝑐𝑜𝑠𝑏𝑡 𝑡 = ∫0𝑐𝑜𝑠𝑎𝑢 𝑐𝑜𝑠𝑏(𝑡 − 𝑢)𝑑𝑢 = ∫0 𝑡 cos(𝑎𝑢+𝑏𝑡−𝑏𝑢)+cos(𝑎𝑢−𝑏𝑡+𝑏𝑢) 𝑑𝑢 = 1 ∫ 2 0 2 𝑡 (cos(𝑎𝑢 + 𝑏𝑡 − 𝑏𝑢) + cos(𝑎𝑢 − 𝑏𝑡 + 𝑏𝑢)) 𝑑𝑢 = 1 𝑡 [cos(𝑎 − 𝑏) 𝑢 + 𝑏𝑡 + cos(𝑎 + 𝑏) 𝑢 − 𝑏𝑡]𝑑𝑢 ∫ 2 0 = 2 𝑎 −𝑏 𝑎 +𝑏 + ] 0 𝑡 1 [𝑠𝑖𝑛[(𝑎−𝑏)𝑢+𝑏𝑡] 𝑠𝑖𝑛[(𝑎+𝑏)𝑢+𝑏𝑡] = 1 [sin(𝑎𝑡−𝑏𝑡+𝑏𝑡) + sin(𝑎𝑡−𝑏𝑡+𝑏𝑡) − 𝑠𝑖𝑛𝑏𝑡 + 𝑠𝑖𝑛𝑏𝑡 ] 2 𝑎−𝑏 𝑎+𝑏 𝑎−𝑏 𝑎+𝑏 = 1 [sin 𝑎𝑡 + sin 𝑎𝑡 − 𝑠𝑖𝑛𝑏𝑡 + 𝑠𝑖𝑛𝑏𝑡 ] 2 𝑎−𝑏 𝑎+𝑏 𝑎−𝑏 𝑎+𝑏 = 1 [(𝑎+𝑏)𝑠𝑖𝑛𝑎𝑡+(𝑎−𝑏)𝑠𝑖𝑛𝑎𝑡−(𝑎+𝑏)𝑠𝑖𝑛𝑏𝑡+(𝑎−𝑏)𝑠𝑖𝑛𝑏𝑡 ] 2 𝑎2−𝑏2 = 1 [2𝑎𝑠𝑖𝑛𝑎𝑡−2𝑏𝑠𝑖𝑛𝑏𝑡 ] 2 𝑎2−𝑏2 = 1 [2(𝑎𝑠𝑖𝑛𝑎𝑡−𝑏𝑠𝑖𝑛𝑏𝑡) ] 2 𝑎2−𝑏2 ∴ 𝐿−1 [ 𝑠2 ] = 𝑎𝑠𝑖𝑛𝑎𝑡−𝑏𝑠𝑖𝑛𝑏𝑡 (𝑠2+𝑎2)(𝑠2+𝑏2) 𝑎2−𝑏2 Example: 5.57 Find the inverse Laplace transform 𝟏 (𝒔𝟐+𝒂𝟐)(𝒔𝟐+𝒃𝟐) by using convolution theorem. Solution: 1 1 𝐿−1 [ 1 ] = 𝐿−1 [ (𝑠2+𝑎2)(𝑠2+𝑏2) (𝑠2+𝑎2) (𝑠2+𝑏2) ] 1 1 = 𝐿−1 [ ] ∗ 𝐿−1 [ ] (𝑠2+𝑎2) (𝑠2+𝑏2) = 1 𝑠𝑖𝑛𝑎𝑡 ∗ 1 𝑠𝑖𝑛𝑏𝑡 𝑎 𝑏 Engineering Maths - IV Laplace Transform Page49
  • 50. = 1 𝑡 ∫ 𝑠𝑖𝑛𝑎𝑢 𝑠𝑖𝑛𝑏(𝑡 − 𝑢)𝑑𝑢 𝑎𝑏 0 ∫ 𝑎𝑏 0 = 1 𝑡 cos(𝑎𝑢−𝑏𝑡+𝑏𝑢)−cos(𝑎𝑢+𝑏𝑡−𝑏𝑢) 𝑑𝑢 = 1 ∫ 2𝑎𝑏 0 2 𝑡 (cos(𝑎𝑢 − 𝑏𝑡 + 𝑏𝑢) − cos(𝑎𝑢 + 𝑏𝑡 − 𝑏𝑢)) 𝑑𝑢 = 1 𝑡 ∫ [cos[(a + b)u − bt] − cos[(a − b)u + bt]]𝑑𝑢 2 0 = 2𝑎𝑏 𝑎+𝑏 𝑎−𝑏 − ] 0 𝑡 1 [𝑠𝑖𝑛[(𝑎+𝑏)𝑢−𝑏𝑡] 𝑠𝑖𝑛[(𝑎−𝑏)𝑢+𝑏𝑡] = = 2𝑎𝑏 𝑎+𝑏 𝑎−𝑏 𝑎 +𝑏 𝑎 −𝑏 1 [sin(𝑎𝑡+𝑏𝑡−𝑏𝑡) − sin(𝑎𝑡−𝑏𝑡+𝑏𝑡) + 𝑠𝑖𝑛𝑏𝑡 + 𝑠𝑖𝑛𝑏𝑡 ] 2𝑎𝑏 𝑎+𝑏 𝑎−𝑏 𝑎+𝑏 𝑎−𝑏 1 [sin 𝑎𝑡 − sin 𝑎𝑡 − 𝑠𝑖𝑛𝑏𝑡 + 𝑠𝑖𝑛𝑏𝑡 ] = 1 [(𝑎−𝑏)𝑠𝑖𝑛𝑎𝑡−(𝑎+𝑏)𝑠𝑖𝑛𝑎𝑡+(𝑎−𝑏)𝑠𝑖𝑛𝑏𝑡+(𝑎+𝑏)𝑠𝑖𝑛𝑏𝑡 ] 2𝑎𝑏 𝑎2−𝑏2 = 1 [−2𝑏𝑠𝑖𝑛𝑎𝑡+2𝑎𝑠𝑖𝑛𝑏𝑡 ] 2𝑎𝑏 𝑎2−𝑏2 = 1 [2(𝑎𝑠𝑖𝑛𝑏𝑡−𝑏𝑠𝑖𝑛𝑎𝑡) ] 2𝑎𝑏 𝑎2−𝑏2 1 𝑎𝑠𝑖𝑛𝑏𝑡−𝑏𝑠𝑖𝑛𝑎𝑡 ∴ 𝐿−1 [ ] = (𝑠2+𝑎2)(𝑠2+𝑏2) 𝑎𝑏(𝑎2−𝑏2) Example: 5.58 Find the inverse Laplace transform 𝒔 (𝒔𝟐+𝟒)(𝒔𝟐+𝟗) by using convolution theorem. Solution: 𝑠 1 𝑠 𝐿−1 [ ] = 𝐿−1 [ (𝑠2+4)(𝑠2+9) (𝑠2+4) (𝑠2+9) ] 1 𝑠 = 𝐿−1 [ ] ∗ 𝐿−1 [ ] (𝑠2+4) (𝑠2+9) 2 = 1 𝑠𝑖𝑛2𝑡 ∗ 𝑐𝑜𝑠3𝑡 = 1 𝑡 ∫ 𝑠𝑖𝑛2𝑢 𝑐𝑜𝑠3(𝑡 − 𝑢)𝑑𝑢 2 0 2 ∫ 2 0 = 1 𝑡 sin(2𝑢+3𝑡−3𝑢)+sin(2𝑢−3𝑡+3𝑢) 𝑑𝑢 = 1 𝑡 [sin(3t − 𝑢) + sin(5u − 3t)]𝑑𝑢 ∫ 4 0 = 4 −1 5 − ] 0 𝑡 1 [−cos(3𝑡−𝑢) cos(5𝑢−3𝑡) = 1 [cos(3t−t) − cos(5t−3𝑡) − 𝑐𝑜𝑠3𝑡 + 𝑐𝑜𝑠3𝑡 ] 4 1 5 1 5 = 1 [𝑐𝑜𝑠2𝑡 − cos2𝑡 − 𝑐𝑜𝑠3𝑡 + 𝑐𝑜𝑠3𝑡 ] 4 5 5 = 1 [5𝑐𝑜𝑠2𝑡−𝑐𝑜𝑠2𝑡−5𝑐𝑜𝑠3𝑡+𝑐𝑜𝑠3𝑡 ] 4 5 20 = 1 [4𝑐𝑜𝑠2𝑡 − 4𝑐𝑜𝑠3𝑡] ∴ 𝐿−1 [ 𝑠 ] = 𝑐𝑜𝑠2𝑡−𝑐𝑜𝑠3𝑡 (𝑠2+4)(𝑠2+9) 5 Example: 5.59 Find 𝑳−𝟏 [ 𝒔 ] by using convolution theorem. (𝒔𝟐+𝒂𝟐) 𝟐 Engineering Maths - IV Laplace Transform Page50
  • 51. Engineering Maths - IV Solution: 𝑠 1 𝐿−1 [ ] = 𝐿−1 [ (𝑠2+𝑎2)2 (𝑠2+𝑎2) (𝑠2+𝑎2) 𝑠 ] 1 𝑠 = 𝐿−1 [ ] ∗ 𝐿−1 [ ] (𝑠2+𝑎2) (𝑠2+𝑎2) 𝑎 = 1 𝑠𝑖𝑛𝑎𝑡 ∗ 𝑐𝑜𝑠𝑎𝑡 = 1 𝑡 ∫ 𝑠𝑖𝑛𝑎𝑢 𝑐𝑜𝑠𝑎(𝑡 − 𝑢)𝑑𝑢 𝑎 0 ∫ 𝑎 0 = 1 𝑡 sin(𝑎𝑢+𝑎𝑡−𝑎𝑢)+sin(𝑎𝑢−𝑎𝑡+𝑎𝑢) 𝑑𝑢 = 1 2 𝑡 ∫ [𝑠𝑖𝑛𝑎𝑡 + sin(2𝑎𝑢 − 𝑎𝑡)]𝑑𝑢 2𝑎 0 = 1 [∫ 𝑡 𝑠𝑖𝑛𝑎𝑡𝑑𝑢 + 2𝑎 0 𝑡 ∫0 sin(2𝑎𝑢 − 𝑎𝑡) 𝑑𝑢] 2𝑎 = 1 [𝑠𝑖𝑛𝑎𝑡 ∫ 0 𝑡 𝑑𝑢 + ∫0 𝑡 sin(2𝑎𝑢 − 𝑎𝑡) 𝑑𝑢] 1 0 𝑡 = [𝑠𝑖𝑛𝑎𝑡(𝑢) − ( cos(2𝑎𝑢−𝑎𝑡) 2𝑎 2𝑎 𝑡 ) ] 0 = 1 [𝑡𝑠𝑖𝑛𝑎𝑡 − cos(2𝑎𝑡−𝑎𝑡) + 𝑐𝑜𝑠𝑎𝑡 ] 2 𝑎 2𝑎 2𝑎 = 1 [𝑡𝑠𝑖𝑛𝑎𝑡 − cos 𝑎𝑡 + 𝑐𝑜𝑠𝑎𝑡 ] 2𝑎 2𝑎 2𝑎 = 1 2𝑎 𝑡𝑠𝑖𝑛𝑎𝑡 𝑠 𝑡𝑠𝑖𝑛𝑎𝑡 ∴ 𝐿−1 [ ] = (𝑠2+𝑎2)2 2𝑎 Example: 5.60 Find 𝑳−𝟏 [ 𝟏 ] by using convolution theorem. (𝒔𝟐+𝒂𝟐) 𝟐 Solution: 1 1 𝐿−1 [ ] = 𝐿−1 [ 1 (𝑠2+𝑎2)2 (𝑠2+𝑎2) (𝑠2+𝑎2) ] 1 1 = 𝐿−1 [ ] ∗ 𝐿−1 [ ] (𝑠2+𝑎2) (𝑠2+𝑎2) 𝑎 = 1 𝑠𝑖𝑛𝑎𝑡 ∗ 1 𝑠𝑖𝑛𝑎𝑡 𝑎 = 1 𝑡 𝑠𝑖𝑛𝑎𝑢 𝑠𝑖𝑛𝑎(𝑡 − 𝑢)𝑑𝑢 𝑎2 ∫0 𝑎2 ∫0 = 1 𝑡 cos(𝑎𝑢−𝑎𝑡+𝑎𝑢)−cos(𝑎𝑢+𝑎𝑡−𝑎𝑢) 𝑑𝑢 = 1 2 𝑡 [cos(2𝑎𝑢 − 𝑎𝑡) − 𝑐𝑜𝑠𝑎𝑡]𝑑𝑢 2𝑎2 ∫0 1 2𝑎2 ∫0 𝑡 = [ cos(2𝑎𝑢 − 𝑎𝑡)𝑑𝑢 − ∫0 𝑡 𝑐𝑜𝑠𝑎𝑡 𝑑𝑢] 1 2𝑎2 ∫0 𝑡 = [ cos(2𝑎𝑢 − 𝑎𝑡)𝑑𝑢 − 𝑐𝑜𝑠𝑎𝑡 ∫0 𝑡 𝑑𝑢] = 2𝑎2 2𝑎 0 𝑡 1 [(sin(2𝑎𝑢−𝑎𝑡) 0 )𝑡 ) − 𝑐𝑜𝑠𝑎𝑡(𝑢 ] = 1 [sin(2𝑎𝑡−𝑎𝑡) − sin(−𝑎𝑡) − 𝑡𝑐𝑜𝑠𝑎𝑡] 2𝑎2 2𝑎 2𝑎 Laplace Transform Page51
  • 52. = 1 [sin𝑎𝑡 + 𝑠𝑖𝑛𝑎𝑡 − 𝑡𝑐𝑜𝑠𝑎𝑡] 2𝑎2 2𝑎 2𝑎 1 2𝑎2 2𝑎 = [2sin𝑎𝑡 − 𝑡𝑐𝑜𝑠𝑎𝑡] (𝑠2+𝑎2)2 2𝑎2 𝑎 1 1 sin𝑎𝑡 ∴ 𝐿−1 [ ] = [ − 𝑡𝑐𝑜𝑠𝑎𝑡] Example: 5.61 Find 𝑳−𝟏 [ 𝒔𝟐 (𝒔𝟐+𝒂𝟐) 𝟐] by using convolution theorem. Solution: 𝐿−1 [ 𝑠2 (𝑠2+𝑎2)2 𝑠 𝑠 ] = 𝐿−1 [ (𝑠2+𝑎2) (𝑠2+𝑎2) ] 𝑠 𝑠 = 𝐿−1 [ ] ∗ 𝐿−1 [ ] (𝑠2+𝑎2) (𝑠2+𝑎2) = 𝑐𝑜𝑠𝑎𝑡 ∗ 𝑐𝑜𝑠𝑎𝑡 𝑡 𝑐𝑜𝑠𝑎𝑢 𝑐𝑜𝑠𝑎(𝑡 − 𝑢)𝑑𝑢 = ∫0 = ∫0 𝑡 cos(𝑎𝑢+𝑎𝑡−𝑎𝑢)+cos(𝑎𝑢−𝑎𝑡+𝑎𝑢) 𝑑𝑢 = 1 2 𝑡 ∫ [𝑐𝑜𝑠𝑎𝑡 + cos(2𝑎𝑢 − 𝑎𝑡)]𝑑𝑢 2 0 ∫ 2 0 = 1 [ 𝑡 𝑐𝑜𝑠𝑎𝑡𝑑𝑢 + ∫0 𝑡 cos(2𝑎𝑢 − 𝑎𝑡) 𝑑𝑢] 2 = 1 [𝑐𝑜𝑠𝑎𝑡 ∫ 0 𝑡 𝑡 𝑑𝑢 + ∫0 cos(2𝑎𝑢 − 𝑎𝑡) 𝑑𝑢] 1 0 𝑡 = [𝑐𝑜𝑠𝑎𝑡(𝑢) + ( sin(2𝑎𝑢−𝑎𝑡) 2 2𝑎 𝑡 ) ] 0 = 1 [𝑡𝑐𝑜𝑠𝑎𝑡 + sin(2𝑎𝑡−𝑎𝑡) + 𝑠𝑖𝑛𝑎𝑡 ] 2 2𝑎 2𝑎 = 1 [𝑡𝑐𝑜𝑠𝑎𝑡 + sin𝑎𝑡 + 𝑠𝑖𝑛𝑎𝑡 ] 2 2𝑎 2𝑎 = 1 [𝑡𝑐𝑜𝑠𝑎𝑡 + 2sin𝑎𝑡 ] 2 2𝑎 ∴ 𝐿−1 [ 𝑠2 (𝑠2+𝑎2)2 2 𝑎 ] = 1 [𝑡𝑐𝑜𝑠𝑎𝑡 + sin𝑎𝑡 ] Example: 5.62 Find 𝑳−𝟏 [ 𝒔𝟐 (𝒔𝟐+𝟒) 𝟐] by using convolution theorem. Solution: 𝐿−1 [ 𝑠2 ] = 𝐿−1 [ 𝑠 𝑠 (𝑠2+22)2 (𝑠2+22) (𝑠2+22) ] 𝑠 𝑠 = 𝐿−1 [ ] ∗ 𝐿−1 [ ] (𝑠2+22) (𝑠2+22) = 𝑐𝑜𝑠2𝑡 ∗ 𝑐𝑜𝑠2𝑡 𝑡 = ∫0𝑐𝑜𝑠2𝑢 𝑐𝑜𝑠2(𝑡 − 𝑢)𝑑𝑢 = ∫0 𝑡 cos(2𝑢+2𝑡−2𝑢)+cos(2𝑢−2𝑡+2𝑢) 𝑑𝑢 = 1 2 𝑡 [𝑐𝑜𝑠2𝑡 + cos(4𝑢 − 2𝑡)]𝑑𝑢 ∫ 2 0 ∫ 2 0 = 1 [ 𝑡 𝑐𝑜𝑠2𝑡𝑑𝑢 + ∫0 𝑡 cos(4𝑢 − 2𝑡) 𝑑𝑢] Engineering Maths - IV Laplace Transform Page52
  • 53. 2 = 1 [𝑐𝑜𝑠2𝑡 ∫ 0 𝑡 𝑡 𝑑𝑢 + ∫0 cos(4𝑢 − 2𝑡) 𝑑𝑢] 1 0 𝑡 = [𝑐𝑜𝑠2𝑡(𝑢) + ( sin(4𝑢−2𝑡) 2 4 𝑡 ) ] 0 = 1 [𝑡𝑐𝑜𝑠2𝑡 + sin(4𝑡−2𝑡) − sin(−2𝑡) ] 2 4 4 = 1 [𝑡𝑐𝑜𝑠2𝑡 + sin2𝑡 + 𝑠𝑖𝑛2𝑡 ] 2 4 4 = 1 [𝑡𝑐𝑜𝑠2𝑡 + 2sin2𝑡 ] 2 4 ∴ 𝐿−1 [ 𝑠2 (𝑠2+𝑎2)2 2 2 ] = 1 [𝑡𝑐𝑜𝑠2𝑡 + sin2𝑡 ] Example: 5.63 Find 𝑳−𝟏 [ 𝟏 𝒔(𝒔𝟐+𝟒) ] by using convolution theorem. Solution: 1 1 1 𝐿−1 [ ] = 𝐿−1 [ 𝑠(𝑠2+4) 𝑠 𝑠2+4 ] = 𝐿−1 [1 ] ∗ 𝐿−1 [ 1 ] 𝑠 𝑠2+4 = 1 ∗ sin2𝑡 2 = sin2𝑡 ∗ 1 2 2 = ∫0 𝑡 sin2𝑢 (1) 𝑑𝑢 = [ −𝑐𝑜𝑠2 𝑢 4 0 𝑡 1 4 ] = (−𝑐𝑜𝑠2𝑡 + 1) 4 = 1 (1 − 𝑐𝑜𝑠2𝑡) Example: 5.64 Find the inverse Laplace transform 𝒔+𝟐 (𝒔𝟐+𝟒𝒔+𝟏𝟑) 𝟐 by using convolution theorem. Solution: 𝑠+2 1 𝐿−1 [ 𝑠+2 ] = 𝐿−1 [ (𝑠2+4𝑠+13)2 𝑠2+4𝑠+13 𝑠2+4𝑠+13 ] 𝑠+2 1 = 𝐿−1 [ ] ∗ 𝐿−1 [ ] 𝑠2+4𝑠+13 𝑠2+4𝑠+13 = 𝐿−1 𝑠+2 1 [ ] ∗ 𝐿−1 [ ] (𝑠+2)2+9 (𝑠+2)2+9 = 𝑒−2𝑡𝐿−1 𝑠 −2𝑡 −1 1 [ ] ∗ 𝑒 𝐿 [ ] 𝑠2+9 𝑠2+9 −2𝑡 = 𝑒−2𝑡𝑐𝑜𝑠3𝑡 ∗ 𝑒 𝑠𝑖𝑛3𝑡 3 = ∫0 3 𝑡 𝑠𝑖𝑛3(𝑡−𝑢) 𝑒−2𝑢𝑐𝑜𝑠3𝑢 𝑒−2(𝑡−𝑢) 𝑑𝑢 = ∫0 𝑡 𝑠𝑖𝑛(3𝑡−3𝑢) 𝑒−2𝑢𝑐𝑜𝑠3𝑢 𝑒−2𝑡+2𝑢 𝑑𝑢 = 1 3 𝑡 ∫ 𝑒−2𝑢 −2𝑡+2𝑢 𝑐𝑜𝑠3𝑢 sin(3𝑡 − 3𝑢) 𝑑𝑢 3 0 3 2 ∫0 −2𝑡 = 𝑒 𝑡 sin(3𝑢+3𝑡−3𝑢)−sin(3𝑢−3𝑡+3𝑢) 𝑑𝑢 Engineering Maths - IV Laplace Transform Page53
  • 54. −2𝑡 = 𝑒 6 𝑡 ∫ 0 [𝑠𝑖𝑛3𝑡 − sin(6𝑢 − 3𝑡)]𝑑𝑢 6 −2𝑡 = 𝑒 [∫ 𝑡 𝑠𝑖𝑛3𝑡𝑑𝑢 − ∫ 0 0 𝑡 sin(6𝑢 − 3𝑡) 𝑑𝑢] 6 −2𝑡 = 𝑒 [𝑠𝑖𝑛3𝑡 ∫ 0 𝑡 𝑡 𝑑𝑢 − ∫ 0 sin(6𝑢 − 3𝑡) 𝑑𝑢] 𝑒−2𝑡 0 𝑡 = [𝑠𝑖𝑛3𝑡(𝑢) + ( cos(6𝑢 −3𝑡) 6 6 𝑡 ) ] 0 6 −2𝑡 = 𝑒 [𝑡𝑠𝑖𝑛3𝑡 + cos(6𝑡−3𝑡) − cos(−3𝑡) ] 6 6 −2𝑡 = 𝑒 [𝑡𝑠𝑖𝑛3𝑡 + cos3𝑡 − 𝑐𝑜𝑠3𝑡 ] 6 6 6 6 −2𝑡 = 𝑒 𝑡𝑠𝑖𝑛3𝑡 𝑠+2 𝑒−2𝑡 ∴ 𝐿−1 [ ] = 𝑡𝑠𝑖𝑛3𝑡 (𝑠2+4𝑠+13)2 6 Example: 5.65 Find the inverse Laplace transform 𝟏 (𝒔+𝟏)(𝒔𝟐+𝟒) by using convolution theorem. Solution: 𝐿−1 [ 1 (𝑠2+4)(𝑠+1) ] = 𝐿−1 [ 1 𝑠+1 𝑠2+4 1 ] 1 1 = 𝐿−1 [ ] ∗ 𝐿−1 [ ] 𝑠+1 𝑠2+4 = 𝑒−𝑡 ∗ 𝑐𝑜𝑠2𝑡 𝑡 𝑒−(𝑡−𝑢)𝑐𝑜𝑠2𝑢 𝑑𝑢 = ∫0 −𝑡 𝑢 𝑡 = 𝑒 ∫0𝑒 𝑐𝑜𝑠2𝑢 𝑑𝑢 −𝑡 [ = 𝑒 (𝑐𝑜𝑠2𝑢 + 2𝑠𝑖𝑛2𝑢)] 12+22 0 𝑒𝑢 𝑡 5 −𝑡 = 𝑒 [𝑒𝑡(𝑐𝑜𝑠2𝑡 + 2𝑠𝑖𝑛2𝑡) − 𝑒0(𝑐𝑜𝑠0 − 0)] 5 −𝑡 = 𝑒 [𝑒𝑡(𝑐𝑜𝑠2𝑡 + 2𝑠𝑖𝑛2𝑡) − 1] ∴ 𝐿−1 [ 1 (𝑠2+4)(𝑠+1) 5 −𝑡 ] = 𝑒 [𝑒𝑡(𝑐𝑜𝑠2𝑡 + 2𝑠𝑖𝑛2𝑡) − 1] Exercise: 5.10 Find the inverse Laplace transforms using convolution theorem for the following 1. 1 Ans: 1 − 𝑐𝑜𝑠𝑡 2. 𝑠(𝑠2+1) 𝑠 8 2 Ans: 1 [sin2𝑡 − 𝑡𝑐𝑜𝑠2𝑡] 3. (𝑠2+4)2 𝑠2 Ans: 1 [𝑡𝑐𝑜𝑠2𝑡 + sin2𝑡 ] 2 2 4. (𝑠2+4)2 1 2 Ans: 1 [𝑒−𝑡 + 𝑠𝑖𝑛𝑡 − 𝑐𝑜𝑠𝑡] 5. (𝑠+1)(𝑠2+1) 1 (𝑠+1)(𝑠2+4) Ans: − 1 𝑒−𝑡 + 1 𝑐𝑜𝑠2𝑡 − 1 𝑠𝑖𝑛2𝑡 5 5 10 ∵ ∫ 𝑒𝑎𝑡 𝑐𝑜𝑠𝑏𝑡𝑑𝑡 = 𝑒𝑎𝑡 𝑎2 + 𝑏2 (𝑎𝑐𝑜𝑠𝑏𝑡 + 𝑏𝑠𝑖𝑛𝑏𝑡) Engineering Maths - IV Laplace Transform Page54